Download as pdf or txt
Download as pdf or txt
You are on page 1of 36

JAIIB MADE EASY : PAPER - 2

ACCOUNTING & FINANCE FOR BANKERS


( ONLY 24 HOUR BOOK - BEFORE JAIIB- 2 PAPER EXAMS )

[THIS BOOK IS MADE FOR LAST MINUTE FINAL REVISION


HAVING ALL FEATURES OF NORMAL BOOK CONTAINS ALL
TOPICS INCLUDING LATEST CHANGES & RECALLED QUESTION–
A 36 PAGE BOOKLET ]

INDEX

PARTICULARS PAGE NO.

SYLLABUS 02-02

MODULE – A ( BASICS OF BUSINESS MATHEMATICS ) 03-07

MODULE - B ( PRINCIPLES OF BOOK KEEPING ) 07-09

MODULE - C ( SPECIAL ACCOUNTS ) 09-12

MODULE –D ( FINAL ACCOUNTS ) 12-19

QUESTIONS BANK ( RECALLED QUESTIONS ) 19-29

PRACTICE SET PAPERS ( BASED ON PREVIOUS PAPERS ) 29-36

Compiled by Mr. Sanjay Kumar Trivedy, Sr. Mgr., RSTC, mumbai


1
SYLLABUS : PAPER 2 : ACCOUNTING & FINANCE FOR BANKERS

Objective: The candidate would be able to acquire the knowledge of the basics of financial
mathematics and basics of accountancy and develop an understanding of the basic financial
concepts.

Module A - Basics of Business Mathematics


Calculation of Simple Interest and Compound Interest -Fixed and Floating Interest Rates - Calculation
of EMIs - Calculation of front end and back end interest -Calculation of Annuities - Calculation of
provisions for NPA and risk weights for Basel II - Interest calculation using products / balances
Bonds - Calculation of YTM - Duration - Bond Pricing - Premium and Discount - Bond valuation
rules preliminary method, definition of debt, rules on compounding in respect of loan accounts,
penal interest etc.
Capital Budgeting - Discounted cash flow - net present value - pay back methods Depreciation -
different types - methods of calculation
Foreign Exchange Arithmetic for beginners

Module B - Accounting in Banks / Branches


Definition & Scope and Accounting Standards-Nature and purpose of accounting; historical
perspectives-Origins of accounting principles-accounting standards and its definition and Scope.
Generally Accepted Accounting Principles – USA,Transfer Price Mechanism
Basic Accountancy Procedures - Concepts of accountancy - entity going concern - double entry
systems, Principles of conservatism - revenue recognition and realization - accrual and cash
basis. Record keeping basics - Account Categories - Debit and Credit Concepts - Journalizing -
Maintenance of Cash / Subsidiary Books and Ledger -Trial Balance - Adjusting and Closing Entries -
Day Book and General Ledger Posting.

Module C - Bank Accounting and Balance Sheet


Rules for bank accounts, cash / clearing / transfer vouchers / system - subsidiary book and main
day book - General Ledger - Branch v/s Bank Accounts
Bank Balance Sheet Structure - accounts - categories - Assets, Liabilities and Net Worth
Components. Accounting for NPA / Provisioning / Suit Filed Accounts.
Preparation of Final Accounts - Final Accounts of Banking Companies.
Disclosure requirements.

Module D - Other Accounts


Partnership accounts - partner’s fixed capital accounts - Current accounts - loan accounts - treatment
of intangibles like goodwill - admission / retirement / death of partner.
Company accounts - classes of Share Capital - issue/ forfeiture of Shares - issue of Bonus Shares.
Bank Reconciliation Statement - Capital & Revenue Expenditure / Depreciation / Inventory Valuation /
Bill of Exchange / Consignment Account / Joint Venture - Special Accounts - Leasing and Hire -
Purchase Company accounts - Accounts of Non - Trading Concerns - Accounting from incomplete
records - Receipts and Payments Account - Income and Expenditure Account, Ratio Analysis.

Module E - Computerized Accounting


Accounting in electronic environment - methods - procedures - security - rectification.
Core banking environment is to be highlighted. Standard books maintained for different accounts
are to be shown as model.

Compiled by Mr. Sanjay Kumar Trivedy, Sr. Mgr., RSTC, mumbai


2
MODULE —A: BASICS OF BUSINESS MATHEMATICS
CALCULATION OF INTEREST
Simple Interest: When money is lent, the borrower usually pays a fee to the lender. This fee is called 'interest'— 'simple' interest or 'flat rate' interest. The
amount of simple interest paid each year is a fixed percentage of the amount borrowed or lent at the start.
Compound Interest: When interest is added to the account against returning it immediately to the customer, the interest itself earns interest during the next time
period for computing interest. This is compounding of interest or more simply stated compound interest.
Compounding Period: The time interval, between the moment at which interest is added to the account is called compounding period.
The Rule of 72: The rule allows us to determine the number of years it takes your money to double whether in debt or investment. Here is how to do it.
Divide the number 72 by percentage rate you are paying on your debt (or earning on your investment)
Annuities: They are essentially a series of fixed payments required from you or paid to you at a specified frequency over the course of a fixed period of time. The most
common payment frequencies are yearly (once a year), semi-annually (twice a year), quarterly (four times a year), and monthly (once a month). There are two
basic types of annuities: ordinary annuities and annuities due.
Sinking Fund: When there is a need for a specified amount of money at a specified future date, it is a good practice to accumulate systematically a fund by means of
equal periodic deposits. Such a fund is called a sinking fund. Sinking funds are used to pay-off debts, to redeem bond issues, to replace worn-out equipment, to buy
new equipment, or in one of the depreciation methods.
(I) About Simple & Compound Interest : Formula for calculation of simple interest is: Interest = PxRxT/100), where 'P' stands for
principal amount, 'R' stands for rate of interest per annum as a fraction (e.g. 8% as 0.08) and 'T' stands for time in years.
(i) Formula for calculation of amount due after a certain period on compound rate of interest is: A= P (1+0)nwhere 'A' is total
amount due after n years., 'P' is the principal amount and 'R' is rate of interest per annum expressed as fraction.
Formula for half yearly compounding will be modified by reducing rate of interest to half its original value and multiplying time by 2.
Likewise for compounding of interest at quarterly rests, the rate of interest will,be divided by 4 and time period multiplied by 4. So the
formulae under suth.dispensation will be:- A=P(1+r/2)2n for half yearly compounding and A=P(1+r/4)4n, for quarterly compounding.
(iv)For monthly compounding, the annual rate will be_divided by 12 and time period multiplied by 12 making the formula as
A=P(1+r/12)12n
(v) Compound interest will be CI =A-P where CI stands for compound interest, A for total amount due and P for principal amount.
(vi) Rule of '72' enables us to calculate the period during which our deposit or loan will become double. It is to divide'72' by annual
rate of interest and the result will be the period during which the amount will become double. For example if you availed a personal
loan @12% as per rule of '72' it will double in 6 years (72/12). Likewise if you have placed deposit with a bank at 8% rate of
interest, the amount of deposit will be double in 9 years (72/8).
(vii) There is a modified version of rule of '72' which is referred to as rule of '69'.It says that period during which the amount will
double will be calculated by dividing 69 by the rate of interest +0.35. To illustrate with 9% rate of interest the period will be
69/9+0.35 i.e. 7.67+0.35 years i.e. around 8.02 years.
(2) Calculation of Equaled, Monthly Instalment (EMI) : EMI is fixed based on the loan amount, rate Of interest and repayment
tenure. The formula for its calculation is as under:
EMI=P X r{(1+r)n/(1+r)n-1}. where 'P' is the principal , 'r' is the rate of interest per instalment period, 'n' is the number of
instalments in the tenure. Rate of interst per instalment period is calculated from the
the value of r will be 12/12 i.e 1% or 0.01.
(3-)Fixed & Floating Interest Rates: Fixed rate means rate of interest is fixed for the-entire period of loan or deposit. To illustrate on
housing loan@ 10% fixed rate the rate of interest will not change for the entire tenure of the loan. In practice banks in India don't offer
fixed interest rates housing loans for periods beyond 3 or 5 years. Likewise, if rate of interest is 8.5% on deposit for period of 5 years it
will not undergo change with the change in the rate of interest in future.
Floating or varying rate of interest is more popular with the banks as it takes care of market risk automatically. Banks prefer offering
loans at varying rates of interest. RBI has advised banks that they can even design deposit products based on floating rate of
interest. In case of varying rate of interest EMI does not undergo change. It is the number of instalments that changes.
(4) Something about Annuities
MARANO'S LAKSSHYA INSTITUTE FOR RANKERS, C-414, 2" FLOOR, ARDEE CITY, SECTOR 52, GURGAON -
° Annuities are series of fixed payments required from the
122003 Contact: 09312219818, 09818546499, 09899692772, 0124-4144687
NANAND'S LAKSSHYA DisTiTurE FOR BANKERS, C-41,4, 2""" FLOOR, ARDEE CITY. SECTOR 52. GURGAON - 1220133
customer or paid to him at specified frequency over the course
of fixed period of time. The payment frequency may be yearly,
half yearly, quarterly or even monthly.
Ordinary Annuity: Under it payments are required or made at the end of each period. In case of straight bonds, for example,
interest payments are effected at half yearly intervals at the end of each period.
Annuity Due:. Under it payments are required or made at the beginning of each period. Payment of monthly rent by the tenant is an
apt example of annuity due.
(5) Calculation of Future Value of Annuities
Future Value (FV) for Ordinary Annuity=C X [{(1+i)n-1}/i] where 'C' stands for cash flow per period,'I is the rate of interest , 'n'
stands for number .of payments. Since in case of annuity due each payment is received, one period sooner, the formula stands
modified.
Future Value of Annuity Due will be = C X [{(1+i) n-1}-1} X (1+i)
(6) Calculation of Present Value of Annuities
Present Value (PV) for Ordinary Annuity= C X [{1- (1+i)n}/i]
Present Value (PV) for Annuity Due = C X [{1- (1+i)n}/i] x (1+i)
(7) Amortization of a Debt is practical application of annuities. When a debt e.g. a housing loan is repaid by equal periodical
instalments , the payments besides recovering the interest on the amount of loan also recover a part of the principal. With the
passage of time more and more amount goes to reduce the principal. Under this method the debt becomes the discounted value of
an annuity. The total period is called term of the annuity and regular periods at which repayments are made is called payment
periods.
(8) Sinking Fund: When a specified amount is required at predetermined future date and a fund is creates for the purpose by means
of equal periodical deposits it is called sinking funds.
(9) Two companies having identical operating income but having different levels of debt can have different taxable income and after tax
income. Does it not sound strange? But it is true because interest payments on debt are tax deductible which means that interest
payments on debt are deducted from income to arrive at taxable income— But it is not so with dividend which is not tax deductible.
BASEL II & III
Basel: A standing committee of Bank Supervisors Committee on Banking Regulation and Supervisory Practices (now known as Basel Committee on Banking
Supervision) set up under the auspices of the Bank for International Settlements, in 1975.
Basel II: It is based on three mutually reinforcing pillars. These are : PILLAR1: (1) Risk based capital requirement or minimum capital
for banks:- While Basel II requires banks to maintain minimum capital adequacy of 8% of their risk weighted assets (RWAs) RBI has
fixed it at minimum 9%. For the purpose of capital requirement banks are required to take into account all types of risk viz. credit risk,
market risk and operational risk. While under Basel I banks were required to maintain minimum capital for credit and market risk
there was no capital requirement for operational risk. Provision of capital for operational risk has been introduced for the first time in
Basel II.
(ii) Credit risk arises out of default of the borrower of the counter party. Market risk arises as a result of adverse movement in
foreign exchange rates or interest rates or commodity or security prices. Operational risk arises on account of system failure or
human failure.
(iii) Banks are required to adopt Standardised Approach for credit risk w.e.f 31.03.2008 / 31.03.2009. The other approaches are
Internal rating based approach-foundation and advanced which banks will be adopting in due course of time.
(iv) Banks are required to adopt Basic indicator Approach for operational risk by 31.03.2008/ 31.03.2019, They are required to
switch over to other approaches viz Standardised Approach and Advanced Measurement Approach in due course of time.
Compiled by Mr. Sanjay Kumar Trivedy, Sr. Mgr., RSTC, mumbai
3
(v) For Market risk, banks have been advised by RBI to continue following the Standardised Duratuion Approach for computing capital
charge for market risk.
(vi) Capital of the banks has been divided into three categories viz. Tier I , Tier II and Tier III. Under Tier I come banks' paid up
capital, free reserves, profit on safe of assets and innovative perpetual debt instruments and Perpetual Non-Cumulative Preference
Shares (PNCPS). Revaluation Reserves, hybrid debt, subordinate debt, provision on standard assets etc fall under Tier II capital. Tier
III capital have not yet been introduced in India.
PILLAR 2 :-Risk based supervision or Supervisory Review and Evaluation Process.Under this Pillar Central banks of countries are
required to ensure that banks have appropriate mechanism to evaluate the risk and provide for the same. RBI is required to
effectively monitor the implementation of its guidelines and to take remedial measures in case of banks who are not able to achieve
minimum capital requirement.
Pillar 3: Risk based Disclosure or Market Discipline. Under it the banks are required to make disclosures regarding their positions to
ensure transparency of their operations.
CAPITAL ADEQUACY RATIO / CAR/CRAR
The objective is to strengthen the capital base of banks with reference to their risk weighted assets, expressed
as a ratio as under. Capital Fund / Risk Weighted Assets x 100
Minimum CAR as per Basel II recommendations 08 %
Minimum CAR in India as per RBI guidelines 09%
Out of this 6% should be Tier I by 31.3.2010, if already not so. Tier II cannot be more than 50% of the total capital as per Basel I.
Tier I TierII
paid-up capital
statutory reserves
other disclosed free reserves
Un-disclosed reserves and cumulative perpetual
capital reserves representing surplus arising out of
preference shares:
sale proceeds of assets.
Revaluation Reserves (at a discount of 55 percent While
Investment Fluctuation Reserve.
determining their value for inclusion in Tier II
Innovative Perpetual Debt Instruments
capital)
Perpetual Non-Cumulative Preference
General Provisions and Loss Reserves upto a
shares.
maximum of 1.25% of weighted risk assets:
(PNCPS)
Hybrid debt capital Instruments (say bonds):
Both not to be more than 40% of Tier I (IPDI alone
Subordinated debt (long term unsecured loans:
max 15%). There is no maturity period. There is call
Debt capital instruments min maturity 15 years
option after 10 years.
Redeemable cumulative preference shares.
Minus
Redeemable non-cumulative preference shares.
equity investments in subsidiaries,
Perpetual cumulative preference shares.
intangible assets, and
losses in the current period and those brought
forward from previous periods

Risk weighted assets — Fund Based


Risk weighted assets mean fund based assets such as cash, loans, investments and other assets. Degrees of credit risk expressed as percentage
weights have been assigned by RBI to each such assets.
Risk Weights for Important Assets
Cash, balance with RBI, 0%
Balances with other banks with CRAR of 9% & above 20%
Secured loans to Staff Members 20%
Other loans to staff members 75%
Housing Loan if LTV.ratio more than 75% 100%
Housing loan if LTV Ratio is up to 75%
Loan up to Rs.30 lac to individual secured by Mortgage 50%
Loan more than Rs.30 lac to individual secured by Mortgage 75%
All Housing Loan of Rs 75 lakh and above 125%
Forex and gold open position 100%
Exposure to Central/State Govt 0%
Central Govt guaranteed advance 0%
State Govt guaranteed advances 20%
Loans to PStis (not guaranteed by Govt) 100%
Claims on unrated corporates 100%
Exposure to DICGC/CGFT. 0%
Exposure to ECGC 20%
Loans against FOR, LIC policy, NSCs with margin 0%
Education loan (under Basel I — 100%) But under Basel-II 75%
Loans against gold/silver jewellery up to Rs.1 lac 50%
Consumer credit / credit cards/Personal Loan 125%
Exposure to capital market 125%
Commercial real estate . 100%
Commercial Real Estate — Residential Housing (CRE-RH) 75%
Venture Capital invstmt as part of Capital market exposure 150%
Loans to non-deposit taking NBFCs for on-lending 100%
Retail Loan & Loan to SME (Retail Loan means limits up to Rs 5 crore and sale in 75%
FY less than Rs 50 crore)
NPA if specific provision is less than 20% 150 %
If specific provision is 20% and above but less than 50% 100%
If specific provision is 50% and above• 150%
Bills purchased under LC issued by another bank 20%

BASEL II: Three Pillars : 1st Pillar: Minimum Capital Standard (to be complied with by bank)
2nd Pillar: Supervisory Review (to be carried by RBI based on ICAAP &SREP)
3rd Pillar: Market Discipline.

Approaches for risk calculation


Standard Approach*, Internal rating Based approach (comprise foundation
Credit Risk
approach & advance approach)
Market Risk Standard Approach* (comprising maturity method & duration method), Internal
risk based approach
Operational Risk Basic Indicator Approach*, Standard Approach, Advance Measurement Approach
Compiled by Mr. Sanjay Kumar Trivedy, Sr. Mgr., RSTC, mumbai
4
*Immediate implementation. Other approaches to be implemented later on.

Introduction of Advanced Approaches of Basel II Framework in India — Time Schedule


As part of Basel II, Banks have migrated to the Standardised Approach for credit risk, Basic indicator Approach for operational risk and the
Standardised Duration Approach for market risk. RBI has now laid down a timeframe for implementation of the advanced approaches in India
as given below.
S. No. Approach Earliest' date of Likely date of
making approval by the RBI
application to the RBI
Internal Models Approach (IMA) for Market Risk April 1, 2010 March 31, 2011
The Standardised Approach (TSA) for Operational April 1, 2010 September 30, 2010
Risk
Advanced Measurement Approach (AMA) for April 1, 2012 March 31, 2014
Operational Risk
'Internal Ratings-Based (IRB) Approaches for Credit Risk April 1, 2012 March 31, 2014
Foundation- as well as Advanced IRB)

BASEL III
The Basel Committee on Banking Supervision (BCBS) has issued comprehensive reform packages entitled "Basel III: A global regulatory framework for more
resilient banks and banking systems" and "Basel III: International framework for liquidity risk measurement, standard and monitoring" in December 2010. A
summary of Basel III capital requirements is given below:
Improving the Quality, Consistency and Transparency of the Capital Base: Presently, a bank's capital comprises Tier 1 and Tier 2 capital with a
restriction that Tier 2 capital cannot be more than 100% of Tier 1 capital. Within Tier 1 capital, innovative instruments are limited to 15% of Tier 1
capital. Further, Perpetual Non-Cumulative Preference Shares along with Innovative Ter 1 instruments should not exceed 40% of total Tier 1. capital at
any point of lime. Within Tier 2 capital, subordinated debt is limited to a maximum of 50% of Tier 1 capital. However, under Basel III, with a view to
improving the quality of capital, the Tier 1 capital will predominantly consist of Common Equity. At present, the regulatory adjustments (i.e. deductions
and prudential filters) to capital vary across jurisdictions. These adjustments are currently generally applied to total Tier 1 capital or to a combination of
Tier 1 and Tier 2 capital. They are not generally applied to the Common Equity component of Ter 1 capital. Most of the adjustments under Basel III will
be made from Common Equity. The important modifications include the following:
(I) deduction from capital in respect of shortfall in provisions to expected losses under Internal Ratings Based (IRB) approach for computing capital for
credit risk should be made from Common Equity component of Tier 1 capital;
cumulative unrealized gains or losses due to change in own credit risk on fair valued financial liabilities, if recognized, should be filtered out
from Common Equity;
shortfall in defined benefit pension fund should be deducted from Common Equity;
certain regulatory adjustments which are currently required to be deducted 50% from Tier 1 and 50% from Tier 2 capital, instead will receive
1250% risk weight; and
limited recognition will be granted in regard to minority interest in banking subsidiaries and investments in capital of certain other financial
entities.
The transparency. of capital base will be improved, with all elements of capital required to be disclosed along with a detailed reconciliation to
the published accounts.
Enhancing Risk Coverage: At present, the counterparty credit risk in the trading book covers only the risk of default of the counterparty. The reform
package includes an additional capital charge for Credit Value Adjustment (CVA) risk which captures risk of mark-to-market losses due to deterioration in
the credit worthiness of a counterparty. The risk of interconnectedness among larger financial firms (defined as having total assets greater than or equal to
$100 billion) will be better captured through a prescription of 25% adjustment to the asset value correlation (AVC) under IRB approaches to credit risk. In
addition, the guidelines on counterparty credit risk management with regard to collateral, margin period of risk and central counterparties and counterparty
credit risk management requirements have been strengthened.
Enhancing the Total Capital Requirement and Phase-in Period: The minimum Common Equity, Tier 1 and Total Capital requirement will
be phased in between Jan 1, 2013 and Jan 1, 2015 as indicated below:
April 1, 2013 Jan 1, 2014 Jan 1, 2015
Min Common Equity 3.50% 4.00% 4.50%
Min Tier I 4.50% 5.50% 6.00%
Min Total Capital 8.00% 8.00% 8.00%

Capital Conservation Buffer: The capital conservation buffer (CCB) is designed to ensure that banks build up capital buffers during normal times (i.e.
outside periods of stress) which can be drawn down as fosses are incurred during a stressed period. The requirement is based on simple capital conservation
rules designed to avoid breaches of minimum capital requirements. Therefore; in addition to the minimum total of 8% as indicated above, banks will be
required to hold a capital conservation buffer of 2.5% of RWAs in the form of Common Equity to withstand future periods of stress bringing
G-99-A
the total Common Equity requirement of 7% of RWAs and total capital to RWAs to 10.5%. The capital conservation buffer in the form of
Common Equity will be phased-in over a period of four years in a uniform manner of 0.625% per year, commencing from January 1, 2016.
Countercyclical Capital Buffer: further, a counter cyclical buffer within a range of 0 - 2.5% of Common Equity or other fully loss absorbing capital will
be implemented according to national circumstances. The purpose of counter cyclical capital buffer is to achieve the broader macro-prudential goal of
protecting the banking sector from periods of excess aggregate credit growth. For any given country, this buffer will only be in effect when there is excess
credit growth that results in a system-wide build up of risk. The countercyclical capital buffer, when in effect, would be introduced as an extension of the
capital conservation buffer range.
Supplementing the Risk-based Capital Requirement with a Leverage Ratio: One of the underlying features of the crisis was the build-up of excessive
on and off-balance sheet leverage in the banking system. Subsequently, the banking sector was forced to reduce its leverage in a manner that not only
amplified downward pressure on asset prices, but also exacerbated the positive feedback loop between losses, declines in bank capital and contraction in
credit availability. Therefore, under Basel III, a simple, transparent, non-risk based regulatory leverage ratio has been introduced. Thus, the capital
requirements will be supplemented by a non-risk based leverage ratio which is proposed to be calibrated with a Tier 1 leverage ratio of 3% (the Basel
Committee will further explore to track a leverage ratio using total capital and tangible common equity). The ratio will be captured with all assets and off
balance'sheet (OBS) items at their credit conversion factors and derivatives with Basel II netting rules and a simple measure of potential future exposure
(using Current Exposure Method under Basel II framework).
Guidelines on Implementation of Basel III Capital Regulations in India: RBI has rescheduled the start date of implementation of Basel HI capital
regulations to April 1, 2013 from January 1, 2013. In view of the shift in the start date of Basel III implementation, all instructions applicable as on January 1,
2013, except those relating to Credit Valuation Adjustment (CVA) risk capital charge for OTC derivatives, would become effective from April 1, 2013 with
banks disclosing Basel III capital ratios from the quarter ending June 30, 2013. As the introduction of mandatory forex forward guaranteed settlement
through a central counterparty has been deferred pending resolution of certain issues such as exposure norms, etc., the CVA risk capital charges would

Compiled by Mr. Sanjay Kumar Trivedy, Sr. Mgr., RSTC, mumbai


5
become effective as on January 1, 2014. The other transitional arrangements would remain unchanged and Basel III will be fully implemented as on March
31, 2018.
CALCULATION OF YTM :
Face Value: It is also known as par value and stated on the face of the bond. It represents the amount borrowed by the firm, which it promise to repay after a
specified period of time. Coupon Rate: A bond carries a specific rate of interest which is also called as the coupon rate. Maturity: A bond is issued for a specified
period of time. It is repaid on maturity. Redemption Value: The value which bondholder gets on maturity is called redemption value. A bond may be redeemed at
par, at premium (more than par value) or at discount (less than par value).
Market Value: A bond may be traded in a stock exchange. Market value is the price at which the bond is usually bought or sold in the market. Market value may be
different from par value or redemption value.
Intrinsic Value: It is quite clear that the holder of a bond receives a fixed annual interest payment for a certain value (equal to par value) at the time of maturity.
Therefore, the intrinsic value is the present value of the cash flow over the redemption period.
Duration of Bond: The holding period for which interest rate risk disappears knows as the duration of the bond
Terms associated with bonds:
(i) Face Value of the bond is the amount of debt raised by the company from the bond-holder. It is also referred to as 'par value'
(ii) Redemption yalue is the amount of money that bond-holder gets on maturity of the bond. A bond may be redeemed at par
(when par value= redemption value-) at a premium (when redemption value> par value) or at a discount (when redemption value <
par value)
(iii),Coupon rate is the rate of interest mentioned on the bond which is payable to the holder of the bond.
iv) Market Value is the price at which the bond is sold or purchased in the stock market. It may be different from par value or
redemption value. If market rate of interest is higher than the coupon rate, the market value of the bond will be less than when
market rate of interest is lower than the coupon rate.
(v) Current yield on bond is defined as the rate of return that the investor earns if he purchases the bond on the current market
price and coupon interest is received.
Current Yield= (Coupon interest / current market price) X100 : To illustrate a bond of Rs 1000 at coupon rate of 9% is
quoted at 1125 in the market, the current yield will be= (90 /1125) X100= 8%
(vi) Yield to Maturity is the discount rate which makes the present value of proposed cash flows equal to the current market price
or purchase price.
Some important facts about Bond Values
(i) The market value is equal to the par value when the required rate and coupon rate are equal.
(ii) The market value of the bond is lower than the par value when the required rate of return is more than the coupon rate.
(iii) The market value of the bond is higher than the par value when the required rate of return is less than the coupon rate.
(iv) Bond price is inversely proportional to its yield to maturity (YTM).
(v) When the required rate of return is more than the coupon rate, the-bond is quoted at a discount to face value. This discount is
greater for longer maturity period and decreases as the maturity approaches.
(vi) When the required rate of return is less than the coupon rate, the bond is quoted at a premium to face value. This premium is
greater for longer maturity period and decreases as the maturity approaches.
From the above, it is clear that the longer term bond is more sensitive to interest rate change than the short-term bond. •
Duration of Bond ,
A holder of the bond has to face interest rate risk for-two reasons; firstly on account of reinvestment of interest income . from the bond
and secondly capital gain / loss on account of sale of the bond at the end of the holding period. When interest rates rise, there is gain
resulting from reinvestment of interest income from the bond but loss liquidation of the bond. Likewise, when interest rates fall, there is
loss, resulting from reinvestment of interest income from the bond but gain from liquidation of the bond as market price will be higher.
For any bond there will be a holding period when the effect of the two will exactly balance each other. In other words, if there is gain on
reinvestment of interest it will be exactly neutralized by loss on liquidation of the bond and vice-versa. This period is called 'Duration of
the bond' and interest rate risk for this period disappears. The concept was first introduced by F. Macaulay and also referred to as
Macaulay Duration.
CAPITAL BUDGETING
Discounted Cash Flow: Time has value, as the cash received today is worth more than the same cash received after the end of a period, as the cash received
today can earn interest during the said period. Present Value: A discounting factor suitably discounts the cash flows, in order to know the present value.
NPV: It is the net difference between the discounted cash inflows and outflows. IRR: IRR is the value of the discount rate in the NPV equation, that leads to a zero
value for the NPV. Payback: It is the period where the net cash inflow equals the initial cash outflow or investment.
Capital Budgeting: There are two main discounting techniques of investment appraisal namely the Net Present Value (NPV) and
Internal Rate of Return [IRR).
(i) Under the NPV, method a comparison is made between present values of future cash flows of an investment opportunity with
the cash outlay which is required to be made. NPV is difference between the present values of all future cash flows and
the investment outlay. Any project will be worthwhile only if NPV is positive and in case it is negative the project does not merit
investment. In case of mutually exclusive projects, the project with maximum NPV should be undertaken
Mathematically, NPV = C1/(1+r)1+C2/(1+r)2+C3/(1+r)3+ C4/(1+r)4 ----------------------+Cn/(1+r)n-I
Internal Rate of Return (IRR) is that discount rate at which NPV becomes zero. In other words cost of funds is equal to the return
from the
project. If IRR is more than the cost of capital, the project must be undertaken. In case of mutually exclusive projects, the project with
highest IRR
should be preferred. In any case, IRR must be more than cost of funds.
Mathematically NPV = C1/(1+r)1+C2/(1+r)2+C3/(1+r)3+ C4/(1+r)4 ----------------------+Cn/(1+r)n-I=0
Depreciation is defined as a decline in the value of a fixed asset due to wear and tear or obsolescence. It is allocation of the cost of
an asset over a period of time for accounting and tax purposes.
Methods of Depreciation and Accounting Standard:
(1) Straight Line Method: This method assumes that the asset will lose equal value for each period. The annual depreciation is
calculated by reducing the salvage or scrap value of the asset from the purchase price and then apportioning the resultant figure
between the number of years of useful life of the asset. In other words
Depreciation = (Purchase price- Scrap value) / estimated useful life of the asset
(2) Reducing Balance or Declining Balance Method: As against the Straight line method of depreciation where the amount of
depreciation charged each year remains the same, in reducing balance or declining balance method percentage of depreciation
remains the same and is calculated as the:- percentage of outstanding balance. Naturally, it is a faster method of , depreciation in
which amount of depreciation is more in the initiaLyears.
(3) Double Declining Method of Depreciation: Under this method amount of depreciation is calculated as if straight line method is
being used.The rate of depreciation is doubled and applied. In the subsequent years also same percentage is applied. However, it is
ensured that if the. amount of depreciation goes below the amount calculated by Straight line method the double declining method is
abandoned and replaced by straight line rate of depreciations.
(4) Sum of the Years Digits method of depreciation:- under this method total useful life of the asset is ascertained. Then the
totals of the year figures are added. For example is an asset has useful life of 5 years the sum of years will be 5+4+3+2+1 i.e. 15.
In the first year the amount of depreciation will be 5/15 of the value of asset, in the second year it will be 4/15, in the third year
3/15 and so on.
(5) Accounting Standard 9 (AS=9) applies to Depreciation. As per this standard, the method of depreciation charged by a
firm should be consistent. Any change in the method of depreciaiton is treated as a change in an accounting policy and is disclosed
accordingly.
DEPRECIATION
Depreciation is defined as a decline in the value of a fixed asset due to wear and tear or obsolescence. It is allocation of the cost of
an asset over a period of time for accounting and tax purposes.
Methods of Depreciation and Accounting Standard:
Straight Line Method: This method assumes that the asset will lose equal value for each period. The annual depreciation is
calculated by reducing the salvage or scrap value of the asset from the purchase price and then apportioning the resultant figure

Compiled by Mr. Sanjay Kumar Trivedy, Sr. Mgr., RSTC, mumbai


6
between the number of years of useful life of the asset. In other words Depreciation = (Purchase price- Scrap value) / estimated
useful life of the asset
Reducing Balance or Declining Balance Method: As against the Straight line method of depreciation where the amount of
depreciation charged each year remains the same, in reducing balance or declining balance method percentage of depreciation
remains the same and is calculated as the:- percentage of outstanding balance. Naturally, it is a faster method of , depreciation in
which amount of depreciation is more in the initiaLyears.
Double Declining Method of Depreciation: Under this method amount of depreciation is calculated as if straight line method is
being used.The rate of depreciation is doubled and applied. In the subsequent years also same percentage is applied. However, it is
ensured that if the. amount of depreciation goes below the amount calculated by Straight line method the double declining method is
abandoned and replaced by straight line rate of depreciations.

Sum of the Years Digits method of depreciation:- under this method total useful life of the asset is ascertained. Then the totals
of the year figures are added. For example is an asset has useful life of 5 years the sum of years will be 5+4+3+2+1 i.e. 15. In the
first year the amount of depreciation will be 5/15 of the value of asset, in the second year it will be 4/15, in the third year 3/15 and
so on.
Accounting Standard 9 (AS=9) applies to Depreciation. As per this standard, the method of depreciation charged by a firm
should be consistent. Any change in the method of depreciaiton is treated as a change in an accounting policy and is disclosed
accordingly.
FOREIGN EXCHANGE ARITHMETIC
Direct and Indirect Quote: A direct quote is the home currency price of one unit of the foreign currency, e.g. US$ 1 = Rs. 42.8450. An
indirect quote is the foreign currency price of one unit of the home currency, e.g. Re.l = US$ 0.0227.
Cross Rate: If rate of currency A is known in terms of currency B and rate of currency B is known in terms of currecy C,we can derive
the rate of currency A in terms of currency C by cross-multiplication.
Chain Rule: The above concept of cross rate is called Chain Rule and can be used in finding the rate of a currency A in terms of other
currencies, through cross-multiplications, eventhough the quote of currency A in terms of that currency is not available in the market.
Value date: The value date is a date on which the exchange of currencies actually takes place.
Types of Rates: Depending on the value date, the exchange rates can be; Cash/Ready, TOM, SPOT, or Forward.
Premium: In case of direct rates, if the forward rate is more than the spot rate, the base currency is called as being at a premium.
Discount: In case of direct rates, if the forward rate is less than the spot rate, the base currency is called as being at a discount.
Forward Points: The forward premium or discount, expressed in percentage points, is called Forward Points, e.g. a forward premium of
0.0150 is referred to as premium of 150 points.
Arbitrage: Arbitrage is an operation by which one can make risk free profits by undertaking offsetting transactions.
(i) Foreign exchange transactions in India are governed under Foreign Exchange Management (FEMA), 1999. Rates are quoted in
two ways. First method is called Direct' while the other is referred to as 'Indirect'. When unit of foreign currency is fixed and the rate
is expressed by varying the Indian rupees (INR) it is called Direct Rate e.g. I USD= INR 45.20-45.30.When INR is fixed and the rate
is expressed by varying the amount of foreign currency it is called indirect Rate e.g. INR 100= USD 2.25- 2.20
(ii) In case of Direct Quotation the principle followed is "Buy low, Sell high" While.in Indirect quotation the principle gets reversed
"Buy high, Sell low"
Which in fact stands for buying more foreign currency for INR 100 and selling less the same INR 100.
(iii) Cross rate is derived when rate of one foreign currency in terms of INR is 'known and we have to calculate value of other foreign
currency in terms of INR. For example, if 1 USD= INR 45 and 1 Euro = 1.60 USD, the rate of Euro in terms of INR will be 1 Euro =
1.60 USD or 1.60 X 45 i.e. INR 72.00 This way of calculating rate from rate of another currency is called Cross Rate.
(iv) Value Date Is the date on which exchange of foreign currencies actually takes place. Based on this concept, transaction based
rates are:
(a) Cash or Ready Rate: It is applied when exchange of currencies takes place on the day of the contract. Normally this rate is
applied to transactions between customers and the banks.
(b) Tom Rate: It is applied when settlement or exchange of currencies takes place on the next business day of the contract.
(c) Spot Rate: It is applied when settlement or exchange of currencies takes place on the second business day of the contract.
Most of the transactions between dealers are based on spot rates.
(d) Forward Rates: It is applied when settlement or exchange of currencies takes place beyond second business day of the
contract. In fact, forward transactions are resorted to by banks and parties as insurance cover against wild fluctuations in exchange
rates. When a currency is quoting higher in future than in cash or spot deal, it is said to be trading at premium and when it is quoted
lower in future it is said to be trading at a discount.

MODULE – B : PRINCIPALS OF BOOK KEEPING

DEFINITION, SCOPE AND ACCOUNTING STANDARDS


Accounting: An art of recording, classifying and summarising in a significant manner and in terms of money, transactions and events, that are, in part
at least, of a financial character and interpreting the results thereof. Financial Statement: A set of documents that shows the results of business
operations during a period, how the results were achieved and the position of assets and liabilities on a given date. It normally means the balance
sheet, profit and loss account, statement of changes in the financial position (which may be either a fund flow statement or a cash flow statement),
explanatory statements, notes and relative schedules forming part of financial statement. Accounting Standards: The policy documents issued
by the recognised expert accountancy body relating to the various aspects of measurement, treatment and disclosure of accounting transactions
and events.
Book keeping and Accounting: Book keeping means recording of business transactions in the books of original entry and in the ledgers.
Accounting, on the other hand, is a much wider concept. It includes book keeping. It stands for compilation of accounts in such a way that one
is in a position to know the state of affairs of the business.
Financial Statements normally mean and include balance sheet, profit and loss account, funds flow and cash flow statements. These also include
explanatory statements, notes and schedules forming part of the financial statements:Ther objective of these statements is to provide information
relating to performance and Financial position of the enterprise. The various users of these statements can be government, share-holders,
investors, bankers, financial institutieinsp:business associates etc.
Accounting standards are the rules and criteria for recording financial events.
Accounting involves: (i) systematic classification of,business, transaction for recording them in books of accounts (ii) Recording of events and
transctions in books of account called book keeping (iii) Summarising of the recorded events i.e. preparation of Trial Balance and preparation
of Balance Sheet and Profit & Loss account from the Trial Balance (iv) Interpretation_of 'Profit & Loss account and Balance Sheet.

Purposes and Objectives of Accounting : (I ) Keeping a systematic record of transactions (ii)Ascertaining results of business operations
(iii)Ascertaining financial position of he business (iv)Facilitating rational decision making (v) Satisfying requirements of law.
Origin of Accounting PrincipIes : The Greeks, Egyptians and Babylonians had well developed records and maintained a good system of record
keeping and control. The concept of Double Entry book keeping took birth in Italy in the 13th and 14th centuries. Luca De Bargo Pacioll an
Italian monk published his book 'Summa'. It contained a section on Double Ent-iy Book Keeping which is still in use the world over. In India also
Kautilya a minister of king Chandragupta wrote 'Arthashastra'.

Compiled by Mr. Sanjay Kumar Trivedy, Sr. Mgr., RSTC, mumbai


7
Accounting Standard board constituted by the Institute of Chartered Accountants in Iridi$1ICAL) formulates Accounting Standards for
mandating by ICAI. If a mandatory accounting standard is not followed the auditors are required to qualify the audit reports. Under Section
21(i) of the Companies Act,1956 where financial statements of a company do not comply with the Accounting Standards , such companies will
be required to disclose the deviation from the Accounting Standard Wagons therefore and the effects of such deviation.

Accounting Standards in India


Accounting Standard (AS) 1 Disclosure of Accounting Policies
Accounting Standard (AS) 2 Valuation of Inventories
Accounting Standard (AS) 3 Cash Flow Statements
Accounting Standard (AS) 4 Contingencies and Events Occurring After the Balance Sheet Date
Accounting Standard (AS) 5 Net Profit or Loss for the Period, Prior Period Items and Changes in Accounting Policies
Accounting Standard (AS) 6 Depreciation Accounting
Accounting Standard (AS) 7 Construction Contracts
Accounting Standard (AS) 8 Accounting for Research and Development. Deleted w.e.f. 01.04.2003 and now covered under (AS) 26.
Accounting Standard (AS) 9 Revenue Recognition
Accounting Standard (AS) 10 Accounting for Fixed Assets
Accounting Standard (AS) 11The Effects of Changes in Foreign Exchange Rates
Accounting Standard (AS) 12 Accounting for Government Grants
Accounting Standard (AS) 13 Accounting for Investments
Accounting Standard (A5) 14 Accounting for Amalgamations.
Accounting Standard (AS) 15 Employee Retirement Benefits'
Accounting Standard (AS) 16 Borrowing Costs
Accounting Standard (AS) 17 Segment Reporting
Accounting Standard (AS) 18Related Party Disclosures
Accounting Standard (AS) 19Leases
Accounting Standard (AS) 20 Earnings Per Share
Accounting Standard (AS)21 Consolidated & financial Statements
Accounting Standard (AS) 22 Accounting for taxes on Income
Accounting Standard (AS) 23 Accounting for Investments, Associates in Consolidated Financial statements
Accounting Standard (AS) 24 Acontinuing Operations
Accounting Standard (AS) 25 Interim Financial Reporting
Accounting Standard (AS) 26 Intangible Assets
Accounting Standard (AS) 27 Financial Reporting of Interests in
Accounting Standard(AS) 28 Impairment of Assets
Accounting Standards(AS) 29 Provisions, contingent Assets & Liability
The main Accounting Concept are :
Cost Concept: According to this concept every -transaction is recorded at present value and not at any future value. Any item which has no
money value is not recorded. This concept does not provide for continuing the original value of the asset which has a definite4seft4 life. Such
an asset is reduced in value through process of depreciation. In pr'ky4ditig for depreciation acquisition or written down value and not future
pricejs"i0entInto account.
Money Measurement Concept: According to this concept every transaction which can be expressed
in terms of money is recorded and which cannot be expressed in terms of money, howsoever,
important for the business, is not recorded in the books of account .For example; if the Managing
Director who is behind the success story of the company is not keeping good health, the fact is not
recorded in the books of account.
Business Entity Concept: According to this concept business and businessman are separate. When
businessman contributes capital to the business, it is a liability for the business. Likewise any profit earned by the business is a liability of the
business towards owner whereas loss is an asset owed by the owner to the business. This distinction is easily understandable in case of a limited
company which is treated as a distinct artificial person separate from its owners i.e. shareholders.
Going Concern Concept: According to this concept, the business is a going concern. If some expense is incurred and its utility consumed during

an accounting period it is shown as expenditure otherwise it is recorded as an asset. Under this concept fixed assets are valued at cost and not
market value; current assets are valued at lower of cost or market price; reserves and provisions are created for any future liability; deferred
revenue expenditures are written off over a number of years.
Dual Aspect Concept: According to this concept every transaction has double effect. If a trader starts a business with a capital of Rs 2,00,000
the business acquires assets worth the amount while the business owes Rs 2,00,000 to the trader as liability in the form of capital. Every
transaction results in credit to one account and simultaneous debit to the other account. This concept is the basis for accounting equation Assets
= Capital+ Liabilties
Matching Concept: According to this concept income and expenditure relevant to a particular period must be matched. If some expense has not yet
been incurred but is relevant to the period it must be provided for. Under this concept all adjustments regarding prepaid expenses, outstanding
expenses are made in the final Accounts. Likewise, the concept of deferred revenue expenditure arises due this concept.
Historical Cost Concept: According to this concept transactions4areffecorc6d as and when they take place and it creates historical records of all
transactions.
Accounting Period Concept: According to this concept all businesses have to report the results of their operations after a particular period which is
referred to Accounting period. All expenses whether paid for or not but relevant to the s accounting period must be provided for. Likewise, all
incomes relevant to the period whether received in cash or not must be accounted for

Main Conventions of Accounting are :


Accounting of Full Disclosure: According to this convention the accounts must be prepared honestly and nothing material should remain
undisclosed . Companies Act lays down the contents of profit and loss account and balance sheet of a company thereby providing for disclosure of
all relevant facts to all concerned. There are certain items or facts which do not find place in the accounting statements. These include contingent
liabilities and m4ketEValue of investments which are appended as notes to the balance sheet.
Convention of Materiality : According this convention, each and every material information must be record& What is material and what is
not material will vary from business to business. But still there are certain conventions. For example if a business has a stock of stationary
items worth Rs 2,000 only it may treat the entire amount as expendittifV60:ke relevant period. However, if the stock is worth Rs 5,00,000 the
business may treat the unused stock as asset.
Convention of Conservatism: According to this convention, while recording transactions all possible losses must be taken into account
while all anticipated profits should be ignored. The convention is, therefore, also referred to as convention of cadence. Creation of
provisions for doubtful debts, creation of contingent .serves are some of the practical applications of the concept of conservatism.
Convention of Consistency: According to this convention if a method is selected for recording transactions, it must be followed in future also. For
example if a firm is charging depreciation under straight line method it should continue to do so in future also. In case it wants to switch over to
written down value method of charging depreciation it must indicate the effect it will have on the business results.
Accounting Concepts at the Recording Stage: Business Entity Concep,Money Measurement Concept, Objective Evidence Concept,
Historical Record Concept, Cost Concept, Dual Aspect Concept,
Compiled by Mr. Sanjay Kumar Trivedy, Sr. Mgr., RSTC, mumbai
8
Accounting Concepts at the Reporting Stage : Going Concern Concept, Accounting Period Concept, Matching Concept,
Conservatism Concept, Full Disclosure Concept, Materiality Concept
Journal and Journalising: Accounting process starts with the identification of financial transactions of a business. Such financial
transactions are recorded permanently in the books of accounts systematically in different specialized books. These books of accounts are
called journal. The journal is an important book under the double-entry system. Journal is the first book of systematic record of-the financial
transactions of the business. Journal is called the book of original or prime entry, because its financial transactions are first of all recorded in
this book as and when they take place. Journal is also called a subsidiary book as it is maintained to help prepare the main book called the
ledger. Journalising is the process whereby the day-to-day business transactions are recorded, in the primary
books of Accounts called Joumal. It can be described as the first phase of accounting.
Phases of Accounting Cycle:
Stage 1- Identifying the transactions that can be recorded, -- •
Stage 2- Classifying of business transactions and rec4d them accordingly.
Stage 3- Communicating the information through reports Such as financial statements and variance reports.
Stage 4- Helping management make decisions from the information that is communicated.
Journal and Subsidiary books:
Subsidiary book may be defined as a book of prime entry in which transactions of a particular category are recorded. In other words, in
order to save time and energy, the transactions which are of similar character are recorded in separate books, these are called subsidiary
books or subdivision of journal. A number of subsidiary books are opened to record all business transactions. In practical system of book-
keeping, subsidiary books are:
Cash Book: Transactions held In cash or by cheque are recorded in this book. There are two sides in a cash book. In the left hand side all
cash receipts are recorded and in the right hand side all cash payments are recorded. Cash Book is of five types: single column cash
book, double column cash book, triple column cash book, bank cash book and petty cash book. In the single column cash book only
receipt of cash and payment of cash are recorded.
In the double column cash book, receipt of cash, receipt of cash discount, payment of cash and cash discount allowed are recorded.
In the triple column cash book along with the transactions which are recorded in double column cash book, cheque received and
cheque paid are recorded. In the bank cash book the receipt of cheque, payment of cheque, cash discount allowed and cash discount
received are recorded. In the petty cash book only small payments of cash are recorded by the petty cashier.
Purchase Book: All credit purchase of goods are written in this book. Cash purchase of goods and credit purchase of assets are not
recorded in this book. Other names of purchase book are purchase day book, purchase journal, bought journal, inward invoice book etc.
Sales Book: All sales of goods are written in this book. Cash sale of goods and credit sale of assets are not recorded in this book. Other names
of Sales Book are Sales Day Book, Sales Journal, Sold book, Outward Invoice Book etc.
Purchase Return Book: It may be necessary to return some goods that the firm has bought on credit for a variety of reasons. All
returns of such goods are recorded primarily in Return Outward Book. This book is also known as Purchase Return Book. Sales
Return Book: Goods may be returned by the customers for a variety of reasons. All goods returned from customers are recorded in
Sales Return Book. This book is also known as Return Inward Book.
Bills Receivable Book: When credit sales of goods are made the purchaser gives his guarantee to make payment in future in the form of
bill. When the seller receives such bill, it is Bill Receivable for him as he will receive payment in future against such bill. In case a
business house receives a number of bills, a Bills Receivable Book is maintained to record all such bills.
Bills Payable Book: When credit purchases are made by a firm it gives a guarantee to the seller to make payment in future in the form of
a bill. This bill is said to be Bills Payable for the firm as he will pay for the bill in future. A Bilis Payable Book is opened to record all
such bills.
Journal Proper: It is a subsidiary book maintained to record the transaction which cannot be recorded in other special subsidiary books.
Usually the transactions of infrequent character are recorded in the journal proper. The entries like adjustment entries, opening entries,
closing entries, transfer entries, purchase and sale of assets on credit, interest on capital, interest of drawing etc. are recorded in journal
proper.
Account Categories: Personal Accounts: (i) Natural Personal Accounts, (ii) Artificial Personal Accounts (iii) Representative Personal
Accounts, Real Accounts: (i) Tangible accounts, (ii ) Intangible Accounts
Nominal Accounts: (1) Expenses and losses (ii) incomes and gains
Examples of Various types of Accounts
Personal Accounts: Reliance ,Industries Ltd, Hari Mohan, Capital account, Wages Outstanding, Bank Loan
Real Accounts: Plant & Machinery, Land & Building, Goodwill, Bills Receivable, Investments, Cash
Nominal Accounts: Salaries., wages, interest, carriage, insurance charges, discounts allowed..
Debit and Credit Concept :
Type of Account Debit Credit
Personal The Receiver The Giver
Real What comes In What goes out
Nominal
Expenses or losses Incomes or gains

MODULE —C: SPECIAL ACCOUNTS


BANK RECONCILIATION STATEMENTS :

What is Bank Reconciliation? When a trader maintains a bank account with a bank, he is issued account statement or pass-book. It reflects the position of
the trader's account with the bank as appearing in the books of the banker. Likewise, the trader enters all transactions in the bank column of the cash book
maintained with him. As soon as the trader deposits a cheque received from his client he will debit the bank account in his cash book whereas the
passbook will reflect this entry only when the cheque is realized which may take 2-3 or even more days. Similarly When the trader issues a cheque in
favour of someone he will immediately credit the bank account in his cash book whereas pass-book will reflect this entry only'wlien the cheque is actually
paid by the bank. On account of these factors and'SeveraLother transactions, normally the balance as reflected by the trader's cash book and the one
reflected in the pass-book will show a difference. There is imperative need to tally the two. This process of tallying or reconciling is called Bank reconciliation.
Items which cause differences between cash book and pass books balances: Cheques issued but not presented for payment ,Cheques lodged for
collection but not realized till date of reconciliation, Bank charges for issue of cheque books, cheque returning charges, standing instructions
charges, cheque collection charge which may not have been entered in the cash book, Interest on savings account which may not have been
accounted for by the depositor, Interest on overdraft charged by the bank which may not have been entered in the cash book, Amount
paid/collected as per customers standing instructions entries for which may not have been made in the cash book, Dishonour of a cheque,
Errors
Trial Balance: Trial balance defined as a statement showing debit and credit balances taken from ledger including cash and bank balance as on
a particular date. Its main purpose is to establish arithmetical accuracy of transactions recorded in the books of accounts. It is usually
prepared at the end of the year but can be prepared on monthly quarterly or half yearly basis also. It helps in preparation of final accounts
Non Tallying of Trial Balance and Classification of Errors: Trial balance may not tally because of certain errors committed. These errors may
be of following types :
Error of Omission: When a transaction is omitted to be recorded in the books of account. it is called error of omission. If both debit and
credit entries are not recorded, it is called error of complete omission and it will not affect trial balance. .However, when either credit or
debit is omitted to be recorded it is called error of partial omission and will result in non., tallying of trial balance.
Errors of commission: When an error is committed while recording the transaction with wrong amount or posted to the wrong side it is called
error of commission. Compensating Errors: When one mistake nullifies the wrong effect of another error or errors, they are called compensating
errors and balance each other. These errors are normally arithmetical errors.

Compiled by Mr. Sanjay Kumar Trivedy, Sr. Mgr., RSTC, mumbai


9
Errors of Principle: These errors occur when accounting principles are not correctly observed. For example purchase of plant and
machinery is entered in the purchase register. This error will not affect trial balance.
Rectification of Errors: Errors can be rectified by reversing the wrong entry already passed or passing the entry for the difference
amount. However, when a number of errors of different types are detected they can be rectified through 'suspense account' mechanism.
Capital & Revenue Expenditure: Whether an expenditure is capital or revenue in nature depends on a number of factors like nature of an
expense, effect on revenue earning capacity and benefit of an expenditure.
Distinction between Capita! and Revenue Expenditure:
Capital Expenditure Revenue Expenditure .::
1. It is non-recurring in nature and the amount spent is usually large. 1.It is recurring in nature and the amount spent is relatively small.
2.The purpose of the expenditure is improving or enhancing the productive or earning 2. The purpose is to Maintain the fixed assets in good working
capacity. condition.
3.The benefit of the expenditure can be enjoyed for a long duration 3.The benefit of the expenditure can be enjoyed for a short duration.
4.It is shown on assets side of the balance sheet 4. it is shown in the Trading and Profit & Loss accounts .

Deferred Revenue Expenditure: This is basically revenue expenditure where entire expense is not charged off to profit and to count and
is deferred as the benefit of such expenditure is likely to accrue to the company for more than one year. Heavy advertisement expenditure
for launching a new product, expenditure for raising debentures or capital and expenditure for formation or registration of a Company come
under deferred revenue expenditure.
I n v e n t o r y V a l u a t i o n : I n v e n t o r y m e a n s stock and includes raw material, semi finished goods and finished goods. Ther-e are
several methods of valuation of stock. Important of these are First in first out (FIFO), Last in first out (LIFO), Average cost method. Under FIFO
goods issued to production /sale is usually the earliest lot on hand. The stock on hard hand consists of the latest consignment. Under LIFO goods
issued to production / sale is according to the latest consignments on hand. Under the Average cost method prices of different lots are added and
divided by the number of pieces. The closing stock is valued according to the price ascertained.
Bills of Extchange: Section 5 of the Negotiable Instruments Act,1881 defines a bill of exchange as an instrument in writing containing an
unconditional order signed by the maker and directing another person to pay a certain sum of money only to or to the order of a certain person or
the bearer of the instrument. The person who signs the order is called 'Drawer ; the person who is required to meet the order of the drawer is
called Drawee and the person who is to receive the payment is called 'Payee'. Though as per provisions of NI Act, 1881 bill of exchange can be
drawn payable to bearer yet under Section 31 of RBI Act, 1934 no person other than RBI, Central Govt. or any person/ institution authorized by-
RBI/Govt. can issue bill of exchange payable to bearer. In a trade transaction the supplier of the goods on credit will make an unconditional order
(bill of exchange) directing the receiver of the goods to pay the amount to a person called payee. Alternatively, the buyer of the goods can make
out a promissory note giving unconditional promise to pay a certain sum of money to the supplier of the goods or to his order. Section 4 of NI Act,
1881 defines promissory' note as unconditional undertaking to pay a certain sum of money on demand or after a certain specified period. As the
promissory note Is signed by the buyer of the goods or receiver of the consideration it does not require any acceptance. If dishonoured on due date,
noting of promissory note is not required. In a bill of exchange transaction, the bill must first be accepted by the drawee. As per provisions of the NI Act,
1881, a drawee is allowed 48 hours excluding public holidays to consider whether he will accept the bill or not. In case of usance bills of exchange he will be
required to pay the amount on due date. In calculating the due date three days of grace are allowed as per section 22 of the NI Act. Under section 25 of the
same Act if a bill of exchange matures on a public holiday, it will be deemed to be due on the immediate next preceding business day. In case bill of
exchange is dishonoured it may be required to be noted through notary public. As of now in case of inland bills noting is not compulsory.
Bill of Exchange Promissory Note • _
1.It contains unconditional order Lit contains unconditional prornise
2. Acceptance of bill of exchange is necessary. 2. Since it is signed by.the promisor it needs no acceptance.
3. There are 3 parties involved. 3. Only two Persons are involved.
4. Dishonour may require noting and/or protest 4. Noting or Protest not needed.
t
Accommodation Bill: It is a bill of exchange which is accepted by one party on whom it is drawn without any consideration but only to mutually help each
other it is called accommodation bill. Normally these bills. are for round amount and are drawn by two parties mutually on each other without transacting any
business.
Consignment Account: 1. The dispatch or sending of goods by the owner to an agent for selling is called consignment. The owner who sends the goods is
called Consignor and the agent who sells the goods is called Consignee.
Difference between Sale and Consignments:

S. No. Sale Consignment


NARANG'S LAKSSHYA INSTITUTE FOR BANKERS, C-41.A.. 2m° FLOOR,. ARDEE OJT, SECTOR 57, GURGAON -
122003 Contact: 09312219818, 09818546499, 09899692772, 0124-4144687

1 Property in the goods is transferred to buyer i.e. he becomes Owner of goods The goods remain the property of the sender i.e. consignor.
2 The buyer can sell the goods at his sweet will. The Consignee can sell the goods as per instructions of the consignor.

3 The entire risk goods is transferred to the buyer The consignor being only an agent of the consignor does not bear risk

4 The buyer can not return the goods to the seller The consignee may return the goods to the consignor if he thinks the
goods are not marketable.
The seller sends a statements called sale invoice. The consignor sends a statement giving details of goods etc. It is
5,.: ;. Called Proforma invoice

Acount Sale: The wholesaler or manufacturer sends his goods to consignee for sale1t!,an indicated price. The consignee is reimbursed expenses for
selling the goods and commission for selling the goods. The consignee prepares an account giving details of sales made, expenses incurred a,
commission due to him and the balance payable to the consignor. It is called Account Sale.
Commission payable to Consignee: It is of two types. (I) Ordinary commission which is a fixed percent of total sales effected by the consignee. Any bad
and doubtful debts have to be provided for by the consignor. (ii) Del Credere is an extra commission, over and above the ordinary commission for sales on
credit. All losses on account of bad debts, expenses on sale have to be borne by the consignee. ,
Entries in the books of the consignor : i. For goods sent on consignment at cost or invoice price
Consignment a/c Dr
To Goods sent on consignment
ii For expenses incurred by the consignor-
Consignment a/c Dr
To Bank
(iii) For expenses incurred and commission payable to consignee-
Consignment a/c Dr
To consignee
(iv) On sale of goods by consignee-
Consignee Dr
To consignment a/c
(v) For bad debts where no del credere commission is allowed-
Consignment a/c Dr
To consignee
(vi) For profit or loss on consignment-
Consignment a/c Dr
To P&L a/c (for profit)
P&L a/c Dr
To Consignment a/c (for loss)
Joint Venture: It is a commercial undertaking Or two -more persons distinct from a partnership for a particular project. Its feature are (i)
agreement for a particular job (ii) the agreement is over as soon as the project is completed.

Compiled by Mr. Sanjay Kumar Trivedy, Sr. Mgr., RSTC, mumbai


10
Distinction between partnership and Jantrlitnture:
Es. no. Partnership Joint Venture
1 Common name, .,No common name
2 Continuing is nature' Temporary in nature
3 Joint and Several liability Liability based on mode of contract
4 Separate books of account No separate set of books
5 Accrual basis of accounting Cash basis of accounting
6 Profit & loss ascertained at the end of the accounting year Profit & loss ascertained at the completion of each venture
Lease: It is a an agreement under which the owner of the assets transfers the right of use of the assets to another person for periodical
payment. The person owning the asset is callledl lessor, the person using the assets is called lessee and the payment effected by him for the
purpose is called lease rental. There are advantages to the lessor as well as lessee.
Advantages the Lessor: (i) The lessor can expand his business by offering his assets on the lease basis which involves comparatively low
amount when assets are sold on ownership basis..(ii) If assets are sold on cash basis the seller has to pay tax on the entire profit which gets
substantially reduced as his profit is spread over a long Period.
Advantages to the Lessee: (i) With assets taken on lease basis there is much lower capital investment for acquiring the assets. (ii) The lessee
is safe from the risk of obsolescence and the same is borne by the lessor. (ii) Lease rental by the lessee is treated as revenue expenditure and it
directly reduces his tax liability. (iv) Leasing reduces the dependence of the lessee on financial institutions and banks for financial needs.
(a) Types of Lease: Lease can of mainly of 4 types: Financial or Capital Lease-It is fairly long term in nature. Usually the entire
economic life of the asset is agreed to be transferred for use by the lessee who bears the risk of obsolescence and under utilization
(b) Operating Lease-Under this type of lease the lessor does not transfer the rewards and risks of using the assets and it is short term
in nature and normally does not span over entire economic life of the asset.
(c) Service Lease- This type of lease is popular with manufacturer of capital goods like machinery and consumer durables and lease
payment covers the cost of servicing and not the capital outlay.
(d) Leveraged Lease- In this type of lease, the lessor arranges finance from a financier and thus there are three parties in this type of
lease - the financier the lessor and the lessee. Since the lessor uses funds borrowed at low costs it is called leveraged lease.
Receipts and Payments Account: There are many firms or organizations that carry on some activities not for profit but for common good
of the society. These are called non- trading concerns. Their final accounts consist of:
(a) Receipts and payment account , b. Income and Expenditure account, c.Balance Sheet
The Receipts and Payments account shows the actual cash (including cheques) received and paid. The receipt or payment may relate the
Previous year, current year or future period. It also does not distinguish between Capital or revenue receipts or expenditures. The account begins
with opening balance of cash and ends with Closing balance of cash.
Income and Expenditure account is nothing but Profit and loss account. Since the non-profit organizations are not for profit the name of the
account is changed to Income & Expenditure account. It shows income and expenditures pertaining to the current year only.
Balance Sheet is a statement of assets and liabilities of the concern as on a Particular date

Receipts and Payments & Income and Expenditure Accounts Compared


S.No. Receipts and PaNern- ents. Account Income and Expenditure Account
1. It is a real account It is a nominal account
2. It records actual receipts and payments during the relevant period It records income and expenditure of the relevant period
3. It shows both capital and revenue items. It shows only revenue items
4. It shows all items of receipts and Payments irrespective of the period It shows income and expenditure of the current year only
to which they pertain

Final accounts from receipt & payments account: (a )Deduct receipts pertaining to the previous year or future period from the total receipts
and add amount receivable but not received to arrive at income relevant to the entire period. Likewise, reduce payments pertaining to the
previous year or future period from the total payments and add amount payable but not actually paid to arrive at expenditure relevant to the
current period.
(b) Any amount received in respect of the previous year will be shown as an asset in the opening balance sheet and that received in
advance as a liability in the closing balance sheet.
(c) Revenue payments for the previous period will appear on the liabilities side of the opening balance sheet while any payments for the
future period will appear as prepaid expenses in the closing balance-sheet.

Depreciation Accounting That part of the cost of fixed asset to its owner which is not recoverable when the asset is finally put out of use
by him. Provision against this loss of capital is an integral cost of conducting the business during the effective commercial life of the fixed asset and is
called 'Depreciation'. In other words, depreciation is an operating cost. There are two popular methods of Depreciation Accounting. These are:
Straight Line Method- In this method the amount of depreciation is a fixed percentage of original cost of the asset. It is calculated as
under: Depreciation= (Cost price of the asset- Scrap value) / Estimated life of the asset
Written Down Value Method- In this method , the percentage of depreciation each year is fixed but it applies to the value at which the fixed
asset stands in the books at the beginning of the year. In other words, with each passing year the amount of depreciation charged goes on
-
diminishing or reducing. Hence this rnethod is also called Reducing balance method of depreciation.

Ratio Analysis:
For the purpose of analysis, financial ratios are classified in four broad heads namely liquidity, solvency, activity and
profitability ratios.
Liquidity ratios are calculated to ascertain the capability of the unit to meet current liabilities in the short run out of realisation of current assets. In this
category the following ratios are computed:

a) Current Ratio = Current Assets / C u r r e n t L i a b i l i t i e s


As per 2n'd method of Tandon Committee Repot this ratio- works out to at least 1.33:1. Ideal current ratio is 2:1 but from the.banker’s
Perspective even 1.33:1 is consider as good ratio
Quick Ratio =Quick Assets / Quick Liabilities
Quick Assets are cash and bank balances, bank fixed deposits, current book debts/receivable, marketable govt. securities, quoted shares meant for disposal. Alternatively
this ratio is calculated as under : Current Assets minus inventory & Prepaid expenses / Current liabilities
Solvency ratios indicate the contribution of owners vis a vis that of outsiders and extent of dependence or outside liabilities. In this category, mainly three ratios are
calculated :
a. Debt equity ratio : Long Term liabilities / Tangible net worth(net worth minus intangible assets) This ratio should be generally not more
than 2:1
b. Total indebtedness Ratio = Term Liabilities + current liabilities /Tangible Net Worth
c. Proprietary Ratio = Tangible tangible / tangible assets x 100
Activity Ratio indicate the efficiency of the unit in utilizing the available resources. In this category various items of assets are
compared with sales.
a) Fixed Asset Turnover Ratio = Net Sales / Fixed Assets
b) Working capital Turnover Ratio = Net Sales / Current Assets
c) Stock Turnover Ratio = Sales / Average Inventory
d. Inventory Turnover = Average Inventory/ Stocks
*Average Inventory =Opening Stock + Closing Stock / 2
(If only one year balance sheet is given, use closing stock instead of average stock) ** Sometimes, it is calculated with reference to cost of sales
e. Debtors velocity Ratio= Average Debtors x 12 for months (or 365 for days) /
(Debt collection period) (Credit) Sales
f) Raw material holding = Average raw materials x 12 for months(or 365 for days) / Raw material consumption
Stock in process holding = Average stock in process x 12 for months (or 365 for days) / Cost of production
Compiled by Mr. Sanjay Kumar Trivedy, Sr. Mgr., RSTC, mumbai
11
Finished goods holding =Average finished goods x 12 for months (or 365 far days) / cost of sales
g) Creditors Velocity Ratio = Average Creditors x 12 for months (or 365 for days) / (creditor payment period) Purchases
Profitability ratios indicate profitability of the unit over the years affdr_compared to
other players in the industry. _
a) Gross Profit Ratio = Gross _Profit x 100
Net Sales
b) Operating Profit Ratio= Operating Profit x 100
Net Sales
c) Net Profit Ratio = Net Profit x 11:1
Net Sales
d) Return on Investment = Net Profits ( before interest and tax) x 100
Tangible net worth + long term loans
e) Return on Proprietor's Fund or Equity's = Net Profit after tax x 100
Tangible Net Worth
Debt service coverage ratio (DSCR) :
Profit after tax + depreciation + annual interest on long term Loans
Annual interest on long term Loans + annual instalment payable on long term loans
This ratio is very useful to Term lending Institutions and banks financing term loans. It is used to ascertain the borrower will have sufficient
funds to repay the instalment of the term Loan. The ideal ratio of DSCR Ratio = 2 : 1

MODULE -D: FINAL ACCOUNTS


Balance Sheet Equation: A Balance sheet is a statement of assets and liabilities as on a particular date. Assets are the items that the business
owns and liabilities are the items that the business owes. The business owes money to the owner of the business and also to the outsiders. The
money owed to the owner is called capital while that owed to the outsiders is called liabilities. In a balance sheet the total of assets is always
equal to the total of capital and liabilities. To sum up we can say
Assets=Capital +liabilities or Liabilties= assets- capital or Capital Assets -Liabilties
The above equation is called Balance Sheet Equation.
1. Partnership Accounts: (A) As per section 4 of the Indian Partnership Act, 1932 partnerships the relation between persons who have
agreed to share the profits of a 14sine-ss carried on by all or any of them acting for all. (B) A partnership concern can have 20 partners for any
business other than banking and 10 for banking business. This restriction on number partners is contained in section 11 of Companies Act,
1956. (B)As per section 25 of the Indian Partnership act, 1932 every partner is liable, jointly with all other partners and also severally, for
all acts of the firm while he is a partner. However , as per section 49 of the same Act, where there are joint debts due from the firm, and
also separate debts due from any partner, the separate property of any partner shall be applied first in the payment of his separate debts, and
the surplus (if any) in the payment of the debts of the firm. To be able to recover its debts from assets of the firrn also the Individual assets
of the partner(s), the banks get the documents, executed not only as partners but also as individuals. (C)A partnership firm need not
compulsorily be registered. But registration of partnership firm confers certain rights on the firm which an unregistered firm does not enjoy. A
registered firm can sue others to enforce it rights arising out of contractual obligation. An unregistered firm cannot sue others in its own name
though others can sue its name. (Section 69 of Indian Partnership Act, 1932) (d) As per section 3o of the Partnership Act, 1932, a minor cannot
be a full fledged partner but he can be admitted to the benefits of partnership. (D)As per section 19 of the Partnership Act, 1932, a partner of a
firm has implied authority to act on behalf of the firm for the normal business of the firm and binds the firm. Accordingly, in the absence of
any usage or custom of the trade to the contrary a partner's implied authority does not cover: admission of any liability in a suit against the firm
withdrawal of any suit filed on behalf of the firm, acquire/transfer any immovable property on behalf of the firm - submitting a dispute relating
to the business of the firm to arbitration, opening a banking account on behalf of the firm In his own name, compromising on behalf of a firm,
entering into partnership on behalf of the firm.
(F) As per section 18 of Partnership Act 1932, a partner is the agent of the firm for the purpose of business of the firm. Being an agent, he
can't delegate his authority to an outsider without the written consent of all other partners. (G) A Limited Company can be a partner in the
firm subject to other provisions of Indian Partnership Act, 1932.However, as per RBI current guidelines, a Non-Banking Finance Company
cannot be a partner in a partnership firm.
3. Fixed and Fluctuating Capital of Partners:
The capital accounts of the partners of a partnership firm can be maintained in two ways. Under the Fixed capital method, two accounts are
maintained for each partner. All transactions relating to interest on capital, interest on loan , share of profit etc. are passed through current
account and the capital account shows a fixed amount and hence the name fixed capital account. Under the Fluctuating Capital method all
transactions are routed through the capital account only and hence the balance can be different at different periods and so this method is
called fluctuating capital method.
4. Goodwill and its valuation:
Goodwill is defined as the value of an established business over and above the value represented by its tangible assets. In simple words it is the
reputation-that the firm has created for itself in the market and helps it to earn more than the normal profit or to earn super profits. it is an
intangible asset of the firm. The necessity of valuation of goodwill arises when there is change in the profit sharing ratio, admission of a new
partner of retirement or death of a Partner or on sale of business. Following methods are used for valuation of goodwill of a firm: (A) Average
Profit method- Under this method the average profit of the firm is multiplied by an agreed multiplier called so many number of years'
purchase. (B) Super Profit Method-Under this method goodwill is calculated by multiplying super profit by a multiplier called so many
number of years purchase of Super Profit. Super profit is defined as the difference between the actual profit and the normal profit expected
in the trade. (C) Capitalisation of Profit Method- Under, this method value of goodwill is arrived at after capitalizing the profit at a given
reasonable or normal rate of return. Profit divided by the normal rate of return gives the capital required to earn that profit . This capital is
then compared with the total assets of the firm. The excess of the capitalized value over actual net assets of the firm gives the value of
goodwill.
5. Steps to be taken on the admission of a partner: (a)Revaluation of assets 41 liabilities, (b) Treatment of goodwill (c) Capital to be
brought in by the new partner, (c) Adjustment relating to accumulated losses and reserves, (d) Adjustment of capital accounts of partners.
6. Steps to be taken on the death or retirement of a partner: (a) Treatment of balance in his current and capital accounts, (b) Treatment of share
of his goodwill (c) Treatment of his share of profits and reserves, (d) treatment of his share on revaluation of assets and liabilities,(e) interest on
capital, drawings and treatment of his share of profit till the date of retirement or death.
7. Final Accounts of Banking Companies: (1) Definition of Banking - As per section 5 of the Banking Regulation Act, 1949 "banking" means
the accepting, for the purpose of lending or investment, of deposits of money from the public, repayable on demand or otherwise, and
withdrawable by cheque, draft, order or otherwise. Under Section 6 of the same Act in addition to the business of banking, a banking company
may engage in any one or more of the following forms of business, namely,- (a) the borrowing, raising, or taking up of money; the lending or
advancing of money either upon or without security; and drawing, making, accepting, discounting, buying, selling, collecting and dealing in bills
of exchange, hundies, promissory notes, coupons, drafts, bill of lading, railway receipts, warrants, debentures, certificates, scripts and other
instruments, and securities whether transferable or negotiable or not; the granting and issuing of letters of credit, travellers' cheques and
circular notes; the buying, selling and dealing in bullion and specie; the buying and selling of foreign exchange including foreign bank notes;
the acquiring, holding, issuing on commission, underwriting and dealing in stock, funds, shares, debentures, debenture stock, bonds,
obligations, securities and investments of all kinds; the purchasing and selling of bonds, scripts or other forms of securities on behalf of
constituents or, others; the negotiating of loan and advances; the receiving of all kinds of bonds, scripts or valuables on deposit or for safe
custody or otherwise; the providing of safe deposit vaults; the collecting and transmitting of money and securities; (b) acting as agents for any
government or local authority or any other person or persons; the carrying on of agency business of any description including the clearing and
forwarding of goods, giving of receipts and discharges and otherwise acting as an attorney on behalf of customers, but excluding the business
of a managing agent or Secretary and Treasurer of a company; (c) contracting for public and private loans and negotiating and issuing the same;
(d) the effecting, insuring, guaranteeing, underwriting, participating in managing and carrying out of any issue, public or private, of State,
municipal or other loss or of _shares, stock, debentures or debenture stock of an company, corporation or association and the lending of
money for the purpose of any such issue; (e) carrying on and transacting every kind of guarantee and indemnity business; (f) managing, selling
and realising any property which may come into the possession of the company in satisfaction or part satisfaction of any of its claims; (g)
acquiring and holding and generally dealing with any property or any right, title or interest in any such property which may form the security or
part of the security for any loans or advances or which may be connected with any such security; (h) undertaking and executing trusts; (i)
undertaking the administration of estates as executor, trustee or otherwise; (j) establishing and supporting or aiding in the establishment and
support of associations, institutions, funds, trusts, and conveniences calculated to benefit employees or ex-employees of the company or the
Compiled by Mr. Sanjay Kumar Trivedy, Sr. Mgr., RSTC, mumbai
12
dependents or connections of such persons; granting pensions and allowances and making payments towards insurance; subscribing to or
guaranteeing moneys for charitable or benevolent object or for any exhibition or for any public, general or useful object; (k) the acquisition,
construction, maintenance and alteration of any building or works necessary or convenient for the purpose of the company; (l) selling,
improving, managing, developing, exchanging, leasing, mortgaging, disposing of or turning into account or otherwise dealing with all or any
part of the property aid rights of the company; (m) doing all such things as are incidental or conducive to the promotion or advancement of the
business of the company;(o) any other form of business which the Central Government may, by notification in the Official Gazette, specify as
a form of business in which it is lawful for a banking company to engage.
(2) Legal Requirements of Banking Companies relating to Accounts and Audit: (a)A company registered under the Companies Act,
1956 is required to prepare and present its financial statements in the format as laid down in Schedule VI to the Companies Act, 1956. (b)
However, banking companies are required under section 29 of the Banking Regulation Act, 1949 to prepare and present their balance sheet and
profit and loss account in the format given in third schedule to the Act .As per RBI guidelines banks operating in India are required to present
their performa balance sheet in form A and profit loss account in form B. (c) Section 30 of the Banking Regulation Act, 1949, the balance-
sheet and profit and loss account prepared in accordance with section 29 shall be audited by a person duly qualified under any law for the time
being in force to be an auditor of companies. In addition every banking shall, before appointing re appointing or removing any auditors, obtain
the previous approval of the Reserve Bank. Further where the Reserve Bank is of opinion that it is necessary in the public interest or in the
interest of the banking company or its depositors so do to; it may at any time by order direct that a special audit of the banking company’s
accounts, for any such transaction or class of transactions or for such period or periods as may be specified in the order, shall be conducted and
may by the same or different order either appoint a person duly qualified under any law for the time being in force to be an auditor of
companies or direct the auditor of the banking company himself to conduct such special audit specified in the order made by the Reserve Bank
shall be borne by the banking company. (d) Under section 31 of the Banking Act, 1949 the accounts and balance-sheet referred to in section 29
together with the auditor’s report shall be published in the prescribed manner and three copies thereof shall be furnished as returns to the
Reserve Bank within three months from the end of the period to which they refer. However, the Reserve bank may in any case extend the said
period of three months for the furnishing of such returns by a further period not exceeding three months. Further a regional rural bank shall
furnish such returns also to NABARD. (e) Rule 15 of the Banking Regulation (Companies) Rules, 1949, provides that the balance sheet and
profit and loss account prepared in terms of section 29 of the Act together with the Auditors’s report shall be published within a period of six
months from the end of the period to which they relate in a newspaper which is in circulation at the place where the banking company has its
principal office. For the purpose of this rule, the expression ‘newspaper’ means any newspaper or journal published at least once a week,
butdoes not include a journal other that a banking, commercial, financial or economic journal.
(3) Bank’s Balance Sheet and its schedules: Form A as per third schjedule to the Banking Regulation Act requires liabilities of the bank in
5 schedule as under
Schedule no. Classification of liabilities
1 Capital
2 Reserves and Surilus
3 Deposits
4 Borrowings
5 Other Liabilities and Provisions
Total
Likewise, in the Form A assets of the bank are shown in 6 schedules numbered as 6 to 11 as under:

Schedule No Classification of assets


6 . Cash and balances with RBI
7. Balances with Banks and Money at Call and
Short
8 . Notice
Investments
9 . Advances Schedule 12 shows contingent liabilities
(4) Bank's Profit and loss account and its schedules: Form B as per third schedule to the Banking Regulation Act requires items of profit
and loss account of the bank in 4 schedules numbered schedule 13 to 16 as under:
Income: Interest Earned Other Income Schedule 13 Year ended Rs.
Expenditure Schedule 14
Interest expended Schedule 15
Operating expenses Schedule 16
Provisions and Contingencies Total
Profit/Loss:
Net Profit/Loss of the year
T o t a l
Appropriations:
Transfer to Statutory Reserves Transfer to other
(5)
R eDetails
s e r v e srelating to various schedules of Balance Sheet and Profit and Loss account of the Banking company:
Schedule l - Capital of the banks required to be shown under authorized capital, issued capital, subscribed capital, called up capital less
unpaid calls and to it is added amount of forfeited shares.
Schedule 2 -Reserves and Surplus are further bifurcated Into statutory reserves, capital reserves, share premium,revenue and other reserves
and balance in the profit and loss account.
Schedule 3 –Deposis of the banks are bifurcated into demand deposits (from banks and from others),Saying bank deposits, term deposits
(from banks and from others). Deposits are further bifurcated into deposits of branches in India and deposits of branches outside India.
Schedule 4- total borrowing of the bank are bifurcated into borrowings from RBI, and from other institutions and agencies. Borrowings
outside India are separately shown. Further secured borrowings are Indicated separately.
Schedule 5- Other liabilities and provisions comprise bills payable, net inter office adjustments, interest accrued and others which includes
provisions.
Schedule 7- This schedule includes besides balances with banks money at call and short notice both in India and outside India. 'At call' money
is repayable on demand, whereas 'short notice' money Implies that notice of repayment of up to 14 days will be given. After cash, money at
call and short notice are the banks' most liquid assets. They are usually interest-earning secured loans but their importance lies in providing the
banks with an opportunity to use their surplus funds and to adjust their cash and liquidity requirements.
Schedule 8-Investments in India and outside India are shown separately. Investments in India include government securities, other approved
securities, shares, debentures and bonds, investments in subsidiaries and/or joint ventures and other investments. Likewise investments outside
India are shown under Government securities including local authorities, investments in subsidiaries and/or joint ventures and other
investments.
Schedule 9-Advances of banks in India are bifurcated into bills discounted and purchased; cash credits and overdrafts and loans repayable on
demand; term loans. All the advances have to be shown under (I) secured by tangible assets (ii) Covered by bank/Govt. guarantees and (iii)
Unsecured. Further these advances are bifurcated into advances to priority sectors, to public sector, to banks and others. Likewise , advances of
banks outside India are Shown under dues from banks and dues from others and the latter are broken into bills purchased and discounted,
syndicated loans and others.
Schedule 11-Other assets of the bank include (i) Net inter-office adjustments (ii) Interest accrued (iii) Tax paid in advance/tax deducted at
source (iv) Stationery and stamps (v) Non-banking assets acquired in satisfaction of claims and (vi) others
Schedule 12- Contingent liabilities of the bank are shown under,sub-heads : (i) Claims against the bank not acknowledged as debts (ii)
partly paid investments (iii) Liability on account of outstanding foreign exchange contracts (iv) Guarantees given on
behalf of constituents in and outside,India (v) Acceptances, endorsements and other obligations (vi) Any other items for which
banks contingently liable.
Schedule 14.-Interest earned on loans and advances is the main source of bank for the bank .Other income which is shown in schedule 1A
comprises (I) commission, exchange and brokerage (ii) profit on sale of investments(iii) Profit on revaluation of investments (iv) Profit on sale
of land/building and other assets (v) Profit on exchange transactions (vi) Income earned by way of dividends etc. from subsidiaries and (vii)
Misc income.
Compiled by Mr. Sanjay Kumar Trivedy, Sr. Mgr., RSTC, mumbai
13
Schedule 16.-Interest expended on deposits is "the main expense for the bank and is shown in schedule 15. Operating expenses which are shown in
schedule 16 are (i) Payments to and provisions for employees and includes pay and allowances, bonus, leave fare concession, gratuity and PF etc. (ii)
Rent , taxes and lighting (iii) printing and stationery (iv) Advertisement and publicity (v) Depreciation on bank's property, (vi)Directors' fees,
allowances and expenses (vii) Auditors' fees and expenses (viii) Law charges (ix) Pastages, telegrams, telephones etc. (x) Repairs and
maintenance (xi) Insurance (xii) Other expenditure.
Schedule 17- Under this, schedule banks are required to make disclosures regarding movement of provisions for NPAs, depreciation of
investments and those regarding income recognition, assets classification and provisioning.
(6) Current Income Recognition, Asset classification and Provisioning norms: As per current guidelines of RBI, a non-performing asset
(NPA) shall be a loan or an advance where (i) interest and/ or installment of principal remain overdue for a period of more than 90 days in
respect of a term loan,(ii) the account remains out of order for a period of more than 90 days, in respect of an overdraft/Cash Credit (OD/CC),
(iii) the account remains overdue for a period of more than 90 days in the case of bills purchased and discounted, (iv) interest and/or
installment of principal remains overdue for two crop seasons in the case of an advance granted for short duration crops (which mature within
12 months) and one crop season in case of long duration crops(which mature after 12 months), (v) any amount to be received remains overdue
for a period of more than 90 days in respect of other accounts. Income recognition - Policy
1.The policy of income recognition has to be objective and based on the record of recovery. Internationally income from non-performing assets
(NPA) is not recognized on accrual basis but is booked as income only when it is actually received. Therefore, the banks should not charge and
take to income account interest on any NPA. 2.However, interest on advances against term deposits, NSCs, IVPs, KVPs and Life policies may be
taken to income account on the due date, provided adequate margin is available in the accounts. 3.If Government guaranteed advances become
NPA, the interest on such advances should not be taken to income account unless the interest has been realized.
ASSET CLASSIFICATION
Categories of NPAs Banks are required to classify non-performing assets further into the following three categories based on the period for
which the asset has remained non-performing and the reliability of the dues:
(a) Sub-standard Assets (b) Doubtful Assets (c) Loss Assets
Sub-standard Assets : A sub-standard asset was one, which was classified as NPA for a period not exceeding 12 Months. In such cases, the
current net worth of the borrower/ guarantor or the current market value of the security charged is not enough to ensure recovery of the dues to
the banks in full. In other words ,such an asset will have well defined credit weaknesses that jeopardize the liquidation of the debt and are
characterized by the distinct possibility that the banks will sustain some loss, if deficiencies are not corrected.
Doubtful Assets : A doubtful asset was one, which remained NPA for a period exceeding 12 months. A loan classified as doubtful has all the
weaknesses inherent in assets that were classified as sub-standard with the added characteristic that the weaknesses make collection or
liquidation in full of the basis of currently known facts, conditions and values , highly questionable and improbable.
Loss Assets ; A loss asset is one where loss has been identified by the bank or internal or external auditors or the RBI inspection but the
amount has not been written off wholly. In other words, such an asset is considered uncollectible and of such little value that its continuance
as a bankable asset is not warranted although there may be some salvage or recovery value.
Special Mention Account-These are potential NPA accounts and if timely and proper corrective actiakigk'ralit taken they will become
NPA.When an NPA account should straightway be classified as 'Loss' or' Doubtful'? When realizable value of both primary and
collateral security falls below 10% of the
Outstanding balance & the account is straightway classified 'Loss' and when realizable value is.*bove-I00/0 but less than 50% of the
outstanding balance it is straightway classified as doubtful PROVISIONING NORMS
(a) In respect of Standard Assets provisioning is required to be made @ minimum 0.40°kof outstanding balance on a global basis. However, in
case of direct advances to agriculturell.SME it has been retained at 0.25%. For commercial real estate the provision is 1%. In November, 2010
RBI has advised banks to make provision in Standard housing loans with teaser terms of payment @ 2%.However, provisioning on these assets
would revert to 0.40 per cent after 1 year from the date on which interest rates are reset at higher rates if the accounts remain
'standard'.(b)Restructured accounts classified as standard advances will attract a provision of 2 per cent in the first two years from the date of
restructuring. In cases of moratorium on payment of interest/principal after restructuring, such advances will attract a provision of 2 per cent
for the period covering moratorium and two years thereafter.
(c) Restructured accounts classified as non-performing advances, when upgraded to standard category will attract a provision of 2 per cent in
the first year from the date of up gradation.
(d)In respect of sub standard assets the rate of provision has been revised to 15% of outstanding balance. However, in respect of unsecured
exposures classified as Sub standard the provision will be 25%**. However, "unsecured exposures" in respect of Infrastructure loan accounts
classified as sub-standard, in case of which certain safeguards such as escrow accounts are available, will attract an additional provision of 5 per
cent only i.e. a total of 20 per cent.
(e)In case of doubtful assets 100% provision is made for unsecured portion. In case of secured portion the rate of provision depends on age of
the doubtful asset and the current rate is as under:
Age of Doubtful Asset provision as 0/0 of secured portion
Up to 1 year 25%
> I year & up to 3 years 40%
> 3 years
** Unsecured exposure is defined as an exposure where the fealisaple_value of the security, as assessed by the bank/approved values /Reserve
Banks inspecting officers, is not more than 10 percent, ab-initio, of the outstanding exposures 'Exposure' shall include all funded and non-
funded exposures (including underwriting and similar commitments). 'Security' will mean tangible security properly discharged to the bank and
will not include intangible securities, guarantees, comfort letters etc.
(7) Additional Disclosure Prescribed By RBr4.4-,
In addition to the disclosure to be made in 'the balance sheet and profit and loss account, in pursuance of the requirements Of the Third
Schedule to the
Act, the RBI has directed that the f011oyAng information should be disclosed by way
of notes on accounts:
 Capital adequacy ratio
 Capital adequacy ,II capital
 Percentage of shareholding of the Government of India in nationalized banks
 Amount of subordinated debt raised as tier II capital
 Gross value of investments, etc.
 Provisions made towards depreciation in the value of investments
 Moment of provisions held towards depreciation on investments
 Repo transactions
 Non SLR investment portfolio
 Forward rate agreement/interest rate swap
 Exchange traded interest rate derivatives
 Disclosures on risk exposure m derivatives
 Percentage of net NPAs to net advances
 Movement in NPAs
 Amount of provisions made towards NPAs
 Movement of provisions made towards NPAs
 Details of Loan assets subjected to restructuring
 Restructuring under CDR
 Details of financial assets sold to a SC/RC for asset reconstruction
Provision on standard assets
 Interest income as a percentage to working funds
 Non-interest Income as a percentage to working funds
 Operating profit as a percentage to working funds
 Return on assets
 Business (deposits plus advances) per employee
 Profit per employee
 Maturity pattern of loans and advances
Compiled by Mr. Sanjay Kumar Trivedy, Sr. Mgr., RSTC, mumbai
14
 Maturity pattern of investment securities
 Maturity pattern of deposits
Maturity pattern of borrowings
 Foreign currency assets and liabilities
 Exposure to real estate sector
 Exposure to capital market: investment in equity shares, etc,—
 Bank financing for margin trading
 Exposure to country risk
 Details of single borrower/group borrower limit exceeded by the bank
 Provision made towards income tax during the year
 Disclosure of penalties imposed by RBI
 Consolidated financial statements —AS 21
 Segment reporting —AS 17
 Related party disclosure — AS 18
 Other disclosures as required under the relevant accounting standards
Features of a Joint Stock Company
(0 Legal formation- No single individual or a group of individuals can start a business and call it a joint stock
company. A joint stock company comes into existence only when it has been registered after completion of all
formalities required by the Indian Companies Act-0956.
(ii) Artificial person- Just like an individual, who takes birth, grows, enters into relationships and dies a joint
stock company takes birth, grows, enters into relationships and may,7,li_e.-41itger, it is called an artificial
person as its birth, existence and death are regulated by law and it does not possess physical attributes like that
of a normal person -
(iii) Separate legal entity-Being an artificial person, a joint stock company has its own separate e3Ostence
independent of its members. It means that a joint stock company can own property, enter into contracts and
conduct any lawful business in its own name. It can sue and can be sued by others in the court of law. The
shareholders" are not the owners of the property owned by the company. Also, the shareholders cannot be held
responsible for the acts of the company.
(iV) Common seal- A joint stock company has a seal, which is used while dealing with others or entering into
contracts with outsiders. It is called a common seal as it can use by any officer at any level of the organization
working on behalf of the company. Any document, on which the company's seal is put and is duly signed by any
official of the company, become binding on the company.
(v) Perpetual existence- A joint stock company continues to exist as long as it fulfils the requirements of law. It
is not affected by the death, lunacy, insolvency or retirement of any of its members. For example, In case of a
private limited company having four members, if all of them die in an accident the company will not be closed. It
will continue to exist. The shares of the company will be transferred to the legal heirs of the deceased members.
(vi) Limited liability- In a joint stock company, the liability of a member is limited to the extent of the value of shares
held by him. While repaying debts, for example, if a person owns 1000 shares of Rs.10 each, then he is liable only up to Rs
10,000 towards payment of debts. That is, even if there is liquidation of the company, the personal property of the
shareholder cannot be attached and he will lose only his shares worth Rs. 10,000.
(vii) Democratic management- Joint stock companies have democratic management and control. That is, even
though the shareholders are owners of the company, all of them cannot participate in the management of the
company. Normally, the shareholders elect representatives from among themselves known as 'Directors' to manage the
affairs of the company.
(viii) Transferability of Shares- The shares of a joint stock company held Can be easily transferred to another person. -
(ix) Large Membership- The number of shareholders of a joint stock canpany may swell to millions as is the case with
Reliance Industries Ltd, HDFC-,,SBI .Etc . Types of Companies
A. On the basis of incorporate in
(1) Chartered Companies-Companies which are incorporated Under special charter or proclamation issued by the head of state,
are known as chartered companies. The Bank Of England, The East India Company, CharteredCBSnieet0: are the examples of
Chartered companies. .--
(2)Statutory Companies- Companies which are formed or incorporated by a special act of parliament, are known as
statutory companies. The activities of such companies are governed by their respective acts and are not required to have
any Memorandum or Articles of Association. Example of such companies are SBI, ICI, LIC etc.
(3) Registered Companies- Registered companies are those companies which are formed by registration under the
companies Act, 1956. Registered companies may be divided into two categories 1. Private limited companies and public limited.
companies. Examples of pub Milked companies are Reliance Industries Ltd, Tata Motors Ltd, Infosys Technologies limited
(4)Foreign Companies„-Companies which are incorporated outside India. Examples of such companies are American express
Bank, HSBC, IBM etc.
B. On the basis of ownership
(1) Private Limited company- These companies can be formed by at least two individuals having4-airrtum paid-up capital of
not less than Rupees one lakh. As per the Companies Act 1956 the total membership of these companies cannot exceed 50. The
share allotted to its members is also not freely transferable between them. These companies are not allowed to raise money
from the public through open invitation They, are required to use "Private Limited" after their names. The example of such
companies are Indian publishers and Distributors Private Limited, Oricom Systems Private Limited, etc. (2)-"Public Limited
Company- A minimum of seven members are required to form a public limited company. It must have minimum paid-up
capital of Rs 5 lakhs. There is no restriction on maximum number of members. The shares allotted to the members are freely
transferable. These companies can raise funds from general public through open invitations by selling its shares or accepting
fixed deposits. These companies are required to write either 'public limited' or 'limited' after their names. Examples of such
companies are Hyundai Motors India Limited, Steel Authority of India Limited etc.
(3) Government Company- Government company is a company which either registered as a Private Company or as a Public
company with the Registrar of Companies under the Companies Act, 1956, & the Government has taken over or purchased 51%
or more capital of the company. The remaining capital may be taken over by the public. Bharat Heavy Electricals Limited, Steel
Authority of India Limited, etc are some examples of Government Company.
(4) Holding Company- A company that owns enough voting stock (generally not less than 51%) in another firm to control
management and operations by influencing or electing its board of directors. It is also called parent company.
(5) Subsidiary company- A company that is publicly-traded but has more than half its stock owned by another company, known
as the parent company. As long as the parent company owns more than half the stock, it maintains control of the
subsidiary, though its other stock is still traded. For example, the trucking company Overnight Transportation is a wholly owned
subsidiary of Union Pacific Corporation. If a subsidiary is wholly owned; all its stock is held by the parent company - •
C. On the basis of Liability
Registered companies are divided into two types, namely, companies having limited Liability and companies having unlimited
liability.
(1) Companies Having Limited Liability This liability can be limited in two ways:
(a) Liability Limited By Shares- These are the companies in which the capital is divided into shares and liability of members (share
holders)-_is limited to the extent of face value of shares held by them. This is the most common & popular type of
company.
(b) Liability limited by guarantee promise to pay a fixed amount to meet the liabilities of the company in the case of liquidation.
(2) Companies Having Unlimited Liability
A company not having any limit on the liability of its members as in the case of a partnership or sole trading concern lean
unlimited company. If such a company goes into liquidation, the members can bk9Ire`g upon to pay an unlimited amount even out
of their individual assets to-Vetthe4laim of the creditors of the company. 10. Distinction between Partnership and Limited
Liability Company
The major difference between companies and partnerships may be considered under the following headings:
(a) Formation: A company is created by registration under the Companies Act. A partnership is created agreement which may
be express or implied from the conduct of the partners is subject to the Indian Contract Act and the Indian Partnership Act. No
Compiled by Mr. Sanjay Kumar Trivedy, Sr. Mgr., RSTC, mumbai
15
(b) special form is required, though partnerships deeds are usually written.
(c) Status at law i_:)% company is an artificial legal person with perpetual succession. Thus a company may buy or sell property,
make contracts and sue and be su0.1t,is-an entity distinct from its members. A partnership is not a legal entity thougli4 May sue
and be sued in the firm's name. Thus the partners own the parties often firm and are liable for the contracts of the firm. jointly as
well as severally.
(d) Transfer Of Shares: Shares in a company are freely transferable unless the company's constitution provides otherwise;
restrictions may, of course, appear in the articles of a private company. A partner can transfer his shares in the firm, but the
assignee does not thereby become a partner and is merely entitled to the assigning partner's share of the profits.
(e) Number Of Members: A private company must have at least two members and maximum 50 members whereas a public
limited company must have minimum 7 members with no ceiling on maximum members.. A partnership cannot consist of more
than 10 persons in case of banking business and 20 persons for other than banking.
(f) Management: Members of a company are not entitled to take part in the management of the company
unless they become directors. Partners can take part in the management of the firm by virtue of their implied
authority unless the partnership deed provides otherwise:
(g) Agency: A member of a company is not an agent of the company or that of other members and cannot
bind a company by his acts. In case of a partnership firm, each partner is an agent of the firm and other
partners, and nay bind the firm by his acts.
(h) Liability Of Members: The liability of a member of a company may be limited by shares or by guarantee.
The liability of a partner is unlimited, joint and several.
(i) Powers: The affairs of accompany are closely controlled by the Companies--Act, 1956 and the company can
only operate within the objects laid down in the memorandum of association, though these can be altered to
some extent :by special resolution. Partners may carry on any business as they please so long as not illegal and
make whatever arrangements they wish with regard to the running of the firm from time to time.
(j) Termination: No one member of a company can wind up the company, and the death, bankruptcy or insanity
of a member does lead to tie afrnp9py being wound up. A partnership may be dissolved by any partner at art
time, unless the 6 partnership is entered into for a fixed period of time. A partnership is also dissolved
by the death or bankruptcy of a partner.
Basis Private
- Limited CoVp-any:'- Public Limited Company 1
:
Membership Minimum - 2 . Minimum - 07
Maximum - 5 Maximum - no
Identification Use a suffix "Private restriction
Use a suffix "Limited"
Limited' after its after its name

Transferability of name
Restricted 1 Free
shares required
Capital Not less than Rs. 1 Not less than Rs. 5
Raising of funds lakh
Can not give open lakh
Can raise as much
money as required
,_ invitation but within its
"
` authorized capital
12.Classes of Share capital
Capital refers to the amount invested in the company so that it can carry on its activities. In a company capital
refers to "share capital". The capital clause in Memorandum of association must state the amount of capital
with which company is registered giving details of number of shares and the type of shares of the company. A
company can not issue share capital in excess of the limit specified in the Capital clause Without Altering the
capital clause of the MA.
The following
v. different terms are used to denote different aspects of share capital:-
(1) Nominal, authorized or registered capital means the sum mentioned in the capital cause of
Memorandum of Association. It is the maximum amount which the company can raise by issuing the shares and
on which the registration fee is paid. This limit is cannot be exceeded unless the Memorandum of Association is
altered.
(2) Issued capital means that part of the authorized capital which has been offered for subscription to
members and includes shares allotted to members for. Consideration in kind also.
(3) Subscribed capital means that part of the issued capital at nominal or face value which has been
subscribed or taken up by purchaser of shares in the company and which has been allotted.
(4)Called-up capital means the total amount of called up capital on the shares issued and shareholders. For
example, if the face value of a share is Rs. 10/- but the company requires- only Rs. 2/- at present, it may call
only Rs. 2/- for the present and the balance Rs.8/- at a later date. Rs. 2/ is the called up share capital and Rs.
8/- is the uncalled share capital.
(5) Paid-up capital means the total amount of called up share capital which is actually paid to the company
by the members.
In India, there is the concept of par value of shares. Par value of shares means the face value of the shares. A
share under the Companies Act, can either of Rs10 or Rs100 or any other value which may be the fixed by the
Memorandum of Association of the company. When the shares are issued at the price which is higher than the par
value say, for example Par value is Rs10 and it is issued at Rs15 then Rs5 is the premium amount i.e., Rs10 is the
par value of the shares and Rs5 is the premium. Similarly when a share is issued at an amount lower than the par
value, say Rs:8, in
That case Rs2 is discount on shares and Rs10 will be par value. _-
13. Types of shares: Shares in the company may be similar i.e. they, may carry the same rights and liabilities and
confer on their holders the same rights; liabilities and
Duties. There are two types of shares under Indian Company '=
1. Equity shares means that part of the share capital of the company which are not preference shares.
2. Preference Shares means shares which fulfill the folOwipg.2-..Conditions. Therefore, a share which is does not
fulfill both these condition is an equity share.
(i) It carries Preferential rights in respect of dividend at fixed amount or at fixed rate i.e. dividend payable is
payable on fixed figure on percent and this dividend must paid before the holders of the equity shares can not be
divided
(ii) It also carries preferential right in regard to payment of capital on winding up or otherwise. It means the amount
paid on preference share must be paid back to preference shareholders before anything is paid to the equity
shareholders. In other words, preference share capital has,pabrity bo in repayment of dividend as well as
C a p i t a l
Types of Preference Shares-
1. Cumulative or Non-cumulative: A non-cumulative or simple preference shares gives right to fixed
percentage dividend of profit of each year. In case no dividend thereon is declared in any year because of absence
of profit, the holders of preference shares get nothing nor can they claim unpaid dividend in the subsequent year
or years in respect of that year. Cumulative preference shares, however, give the right to the preference
shareholders to demand the unpaid dividend in any year during the subsequent year or years when the profits are
available for distribution. In this case dividends which are not paid in any year are accumulated and are paid out
when the profits are available.
2. Rededmablkand Non- Redeemable: Redeemable Preference shares are preference shares which have to be
repaid by the company after the term of which for which the preference shares have been issued. Irredeemable
Preference shares mean preference shares need not be repaid by the company except on winding up of that
company. However, under the Indian Companies Act, a company cannot issue irredeemable preference shares.
In fact, a company limited by shares cannot issue preference shares which are redeemable after more- than 10
years from the date of issue. In other words the maximum tenure of preference shares is 10 years. If a company
is unable to redeem any preference shares within the specified period, it may, with consent of the Company Law
Compiled by Mr. Sanjay Kumar Trivedy, Sr. Mgr., RSTC, mumbai
16
Board, issue further redeemable preference shares of equal amount to redeem the old preference shares
including dividend thereon.

Voting Rights of the Members


Every member of a public company limited by shares holding equity shares will have votes in proportion to his share
in paid up equity capital of the company. Generally, preference shareholders do not have any voting rights.
However, they can vote on matters directly relating to the rights attached to the preference share capital. Any
resolution for winding up of the company or for the reduction or repayment of the share capital shall be deemed
to affect directly the rights attached to preference shares. Where the preference shares are cumulative (in
respect of dividend) and the dividend thereon has remained unpaid for an aggregate period of two years before
date of any meeting of the company, the preference shareholders will have right to vote on any resolution. In
case of non-cumulative preference shares, preference shareholders have right to vote on every resolution if
dividend due on their capital remains unpaid, either in respect of period of not less than two years ending with
the expiry of the financial year immediately preceding the commencement of the ' meeting or in respect of
aggregate period of not less than three years comprised in six years ending with the expiry of concerned financial
year. Every equity: shareholder has a right to vote at a general meeting. No company can prohibit any member
from exercising his voting right on any ground including the ground that he has not held his shares for a
minimum period before he becomes eligible to vote. However, a member's voting rights can be revoked if that
member does not make payment of calls or other sums due against him or where the company has exercised the
right of lien on his shares.
Further issue of the capital
(a) Rights Issue of Shares- If, at any time after the expiry of 2 years from the date of incorporation of the
company or after one year from the date of first allotment of shares, whichever is earlier, a public
company limited by shares issues further shares within the limit of authorized capital and directors
must first offer such shares to the existing holders of equity shares in the proportion to the capital
paid up on their shares at the time of further issue. This is commonly known as "Rights Issue of
shares". The company must give notice each of the equity shareholders giving him the option to buy
the shares offe4echo.tlirrn. The shareholders must be informed of the number of shares he has the
option to buy. He must be given at least 15 days to decide for exercising his option. The directors
must state in the notice of the offer the fact that the shareholders also have the right to renounce
the offer in whole or part in favour of some other person. This is commonly known as "Renunciation
of Rights".
If the shareholder does not inform the company of his decision to take the shares, it is deemed that he has
declined the offer. In case where the rights shares are not taken by the shareholders, the directors of the
company may dispose of the shares in the manner they think fit.
A company may by special resolution in the general meeting decide that the directors need not offer the shares
to the existing shareholders of the equity shares and that they may dispose them off in a manner thought fit by
them. This is known as "preferential or offer of shares" where third parties or only certain shareholders are giveii-
81kr,es in priority over the other shareholders.
(b) Issue of shares at discount-A company may issue shares at a discount i.e. at a value below its par value.
The following conditions must be satisfied in connection with the issue of shares at a discount:-

1. The shares must be of a class already issued.


2. Issue of the shares at discount must be authorized by resolution passed in the general meeting of company
and sanctioned by the company law board.
3. The resolution must also specify the maximum rate of discount at which the Shares are to be issued
4. Not less than one year has elapsed from the date on which the company was entitled to commence the
business.
5. The shares to be issued at discount must issued within 2 months after the date on which issue is sanctioned by the
company law board or within extended as may be allowed by the Company Law Board.
6. The discount must not exceed 10 percent unless the Company Law Board is of
the opinion that the higher percentage of discount may be allowed in special circumstances of case.
(c) Issue of shares at premium
A company may issue shares at a premium i.e. at a value above its par value. The following conditions must be satisfied in
connection with the issue of shares at a premium:-
1. The amount of premium must be transferred to an account to be called share_ premium account. The provisions of this Act
relating to the reduction of share .capital of the company will apply as if the share account premium account were paid up share
capital of the company.
2. Share premium account can be used only for the following purpose 1.- (a) In issuing fully paid bonus shares to members.
(b) In writing off preliminary expenses of the company. (C) In writing off public issue expenses such as underwriting
commission, advertisement expenses, etc (d) In providing for the premium payable paid on redemption of any
redeemab1elme6tei-ice shares or debentures. (e) In buying back its shares
(d) Issue of bonus shares- Bonus shares are issued by converting the reserves of the company into share capital. It is nothing but
capitalization of the reserves of the company. Bonus shares can be issued by a company only if the Articles of Association of the
company authorizes a bolus shares. Where there is no provision in this regard in the articles, they must be amerilde,d7ty passing
special resolution act at the general meeting of the company care must be taken that issue of bonus shares does not lead to total
share capital in Excess of the authorized share capital. Otherwise, the authorized capital is increased by amending the capital
clause of the Memorandum of association If the company has availed of any loan from the financial institutions, prior *mission is to
be obtained from the institutions for issue of bonus shares. If the company is listed on the stock exchange, the stock exchange
must be informed of the –decision of the board to issue bonus shares immediately after the board meeting where the bonus shares
are to be issued to the non-
resident members2e-iolkconsent of the Reserve Bank should be obtained. Only fully paid up bonus shares can be issued. Partly paid
up bonus shares cannot be issued since the share holders become liable to pay the uncalled amount on those shares.
(e) Sweat equity and Employee Stock Options
Sweat Equity Shares mean equity shares issued by the company to its directors and/ or employees at a discount or for
consideration other than cash for providing know how or making available the rights in the nature of intellectual property rights or
value additions.
A company may issue sweat equity shares of a class of shares already issued if the following conditions are fulfilled: -
(I) A special resolution to the effect is passed at a general meeting of the company (ii) The resolution specifies the number of
shares, the current market price, consideration, if any, and the class of employees to whom the shares are to be issued (iii) At
least 1 year has passed since the date on which the company became eligible to commence business
(f) Forfeiture and Reissue of shares:
Shares are forfeited because only a part of the due amount of such shares is received and the balance remains unpaid. On
forfeiture the membership of the original allottee is cancelled. He/she cannot be asked to make payment of the remaining
amount. Such shares become the property of the company. Therefore company may sell these shares. Such sale of shares is
called 'reissue of shares'. Thus reissue of shares means issue of forfeited shares.
Once the Board of directors has forfeited the shares, the defaulting share holder is asked to return the share certificate which is
cancelled thereafter. The board of directors passes a resolution allotting the forfeited shares to the new purchaser/purchasers of
such shares.
In case of reissue of shares neither a prospectus is issued nor is any offer otherwise made to the general public. Though the
amount of such shares may be called in more than one installment but usually the entire amount is called in one installment
I.e. lump sum.
The board of directors of the company while reissuing the shares decides the price of reissue. These shares can be reissued at
Compiled by Mr. Sanjay Kumar Trivedy, Sr. Mgr., RSTC, mumbai
17
par, at premium or at discount. Generally, these shares are reissued at a discount i.e. at a price which is less than its-nominal
value. The amount of discount allowed at the time of reissue in no case should be more than the amount forfeited on such shares.
Question arises at what price the forfeited shares can be reissued?--There is no limit of the price at which it can be reissued if
price charged is more than the price of issue at the time of their forfeiture. But then there is a limit below which price cannot be
charged or we can say that there is a minimum price below which the company cannot reissue its forfeited shares. We can look at
it from another angle i.e. the company cannot give discount more than a particular amount while reissuing the forfeited shares. The
maximum permissible discount at the time of reissue of forfeited shares is ascertained in different situation in the following
manner: (I) Shares originally issued at par : When the shares are originally issued at par, the maximum permissible discount for
reissue of shares is equal to the amount forfeited on such shares
(ii) Shares originally issued at premium: In case of shares originally issued at premium, there can be two situations: cal premium
has not been received on the forfeited shares, and (b) premium has been received on such shares. The amount forfeited is the
amount thaghas4been received including the amount of premium if it has been received and the maximum discount that can be
allowed on reissue of such shares is the amount so forfeited
(iii) Shares originally issued at discount: In this case the actual amount received becomes the forfeited amount. But the maximum
permissible discount on reissue of shares will be equal amount forfeited plus the amount of discount initially
allowed on these shares at the time of their original issue.
Final Accounts of Limited Companies —General and Legal Requirements
(1) Genera! requirements of the Companies Act
The joint stock companies are legally required to prepare a set of financial statement, to periodically assess the profits earned and
to know the financial position of the company as on a specified date. In the case of companies registered under the Companies
Act, the Act specified the books of accounts to be maintained and also prescribes the format and content of the financial
statements. The legal requirement laid down by the Companies Act, therefore, assumes a great
Importance in the preparation of the financial statements of a joint stock company. In addition, the accounts must be statutorily
audited by an external person called the auditor and it is the duty of the auditor to submit a report in the prescribed format to the
shareholders
Books of Accounts :Section 209 of the Companies Act specifies the books of accounts to be maintained by a company in respect of
—(1) All sums of money received and expended by the company and the matters in respect of which the receipts and expenditure
take place Land sidings Buildings Lease hold Railway Plant and machinaries. Furniture iridliltingt Develop-merit fitproperty Patents,
trademarks and designs etc.
Tangible worth noting here that goodwill, patents, trademarks and designs which are normally shown as intangible assets in
financial analysis, are shown as fixed assets in the case of a company.
(a) Current assets will cover- Interest accrued on investment,Stores and spare parts,Loose tools,Stock-in-trade,Work-in-progress,
Sundry debtors, Cash balance on hand .
N.B. Sundry debtors are broadly classified into debtors outstanding for a period exceeding 6 months and other
debtors. The provision for doubtful debits should be shown as deduction from the debtors.
(c) Miscellaneous expenditure will cover Preliminary expenses Expenses including commission or broker-age on
un writing or subscription of share or debentures Discount allowed on the issue of share or debentures Interest
paid out of capital during construction Development expenditure (not adjusted)
Other items
Liabilities of the companies will cover (i) Share capital (ii) Reserves and surplus (iii) Secured loans (iv)
Unsecured loans (v) Current liabilities (vi) Provisions Share capital will be shown as Authorized capital Issued
capital; Subscribed capital; Called up capital; Calls in arrears;Any forfeited shares
Classes of preferences; Paid up capital; Bonus shares issued;
Reserves and surplus will include :Capital reserves,Capital redemption reserve,Share premium account,Other
reserves,Surplus, balance after providing Tor diQerid, bonus or reserves;Proposed additions to reserves;Sinking
fun
(a) Secured loans will_cover : Debentures , Loans and advanes from the banks Loans and advances from
subsidiaries, Other loans and advances
Unsecured loans will comprise : Fixed deposits ,Short term Loans from banks ,Loans from others
Current Liabilities will be : Bills payable (accounts payable), Sundry creditors, Amount due to subsidiary
companies,Advance payment received
Unclaimed dividend Other's liabilities;Interest accrued but not
due on loans
Notes to the Balance Sheet
(i) Claims against the company not acknowledged as debts
(ii) Uncalled liability on shares partly paid
(iii) Arrears of fixed cumulative dividends
(iv) Estimated amount of contracts remaining to be executed on capital account and not provided for
(v) Other money for which the company is contingently liable.
(7) Requirement of the Companies Act with Respect to Profit and Loss Account Part II of schedule VI of the Companies
Act, 1956 does not prescribe any format for the profit and loss account but only outlines the information to be included. The
various items of receipts and expenses should be arranged under the most convenient
heads.
(a) Revenues : The following are the revenues which must be disclosed in the P&L account
(i) The turnover or the aggregate amount of sales effected by the company
(ii) The gross income derived from services rendered or supplied.
(iii) Incomes from trade investments and other investments
(iv) Incomes by way of interests
(v) Profit on investments
(vi) Profits of non recurring nature
(vii) Miscellaneous income
(viii) Dividends from subsidiary companies
(b) Expenses : The following are the expenses which must be disclosed in the profit and loss account.
(i) The value of raw material consumes, item wise breakup and quantity consumed.
(ii) Opening & closing stock of raw material
(ii) Opening & closing work-in-progress
(iv) The amount provided for depreciation
(v) Consumption of stores and spare parts
(vi) Power and fuel
(vii) Rent;
(viii) Repairs to buildings
(ix) Repairs to machinery
(x) Salaries and wages and bonus
(xi) Miscellaneous expenditure
(xii)Contribution to provident fund and other funds (xiii)Staff welfare experises
(xiv)Insurance
(xv) Income tax payable
(xvi) Dividend Paid;'
(xvii) Provisions for losses of subsidiary companies (xviiiI ) Amounts reserved for repayment of share
capital
(xviii) Amount set aside for reserves
(xix ) Amounts set aside for provisions
(xx) Expenses related to sales, trade discounts
(8) Notes to profit and Loss Account
According to part II schedule VI, certain information has to be provided by way of
notes to Profit and Loss account. It includes details of

Compiled by Mr. Sanjay Kumar Trivedy, Sr. Mgr., RSTC, mumbai


18
(i)Payments made to directors
(ii) Managerial remuneration paid or payable under section 198
(iii) Other allowances & commission
(iv) Any other perquisites or benefits in cash or in kind
(v) Pensions
(vi) Gratuities
(vii) Payment to provident fund
(viii) Compensation for loss of office
(ix) Retirement benefits
(x) Auditors fees
(xi) Value of imports calculated on C.I.F basis
(xii) Expenditure in foreign currency on account of royalty, etc.
(xiii) Value of export of goods calculated on F.O.B basis
(xiv) Other incomes indicating the nature thereof
Profit and Loss Appropriation Account
It shows in detail the appropriations made from the profits in respect of dividends and transfer to reserves etc.,
The balance in the P&L appropriation account ,if it is a credit balance will be shown on the liability side of the
balance sheet heading' "reserves and surplus". If there is a closing debit balance in the profit and loss
appropriation account (that is, a loss), then such balance will be shown am the asset side under the heading
"miscellaneous expenditure"
Some Items of Final Accounts of the Company Explained
(a) Sinking Fund- Reserves created by periodically setting aside a certain sums in a custodial account as cash
or investment in marketable securities for future replacement of an asset or repayment of a liability.
(b) Difference Between Provision & Reserve: The Provisions are charged on profit and loss account; even
though there were no profits for the current financial year. It is mandatory to create-the Provisions according
to conservatism concept. On the contrary the reserves are apportioned from the profits only. If there are no
profits, the managemt cannot create Reserves. They are not mandatory. Reserves can be used to write off
capital losses. Later they will form the part of owners net worth.
(c) Share Premium Account : Share Premium account is shown on liability side of balance sheet under the
head reserves and surplus. Section 78 of Companies Act restricts the use of the amount collected as premium
onectiritir0 for the following purposes alone. (i)Issuing fully paid bonus shares to the4'Members; ii. Writing off
preliminary expenses ; iii. Writing off the expenses of issue or the commission paid or discount allowed on any
issue of shares or debenture of the company;iv. Providing for the Premium payable on the redemption of any
redeemable preference shares of any debentures of the company;v.In purchasing its own shares (buy back)
[section 77A)
(d) Dividend : Dividends may defined as the share of profits that is payable to each share holder of the
company.The companies act lays down that dividends can be paid out of profits only and prohibits the payment
of any dividend out of capital. Further dividend should be paid in cash only. A company may pay dividends from
any or all of the following three sources : Profits of the current year ,Undistributed profits of previous year,Money
provided by the central or any state government for the payment of dividends
(e) Something more about dividend
(i) The dividend recommended by the directors is termed as proposed dividend till such time it is adopted by
the share holders in the annual general meeting. The proposed dividend will be classified as a provision and
shown on the liability side of the•balance sheet .
(ii) The dividend finally decided by the share holders in the annual general meeting as payable Is termed as
declared dividend.
(iii) Any dividend declared must be paid within 42 days from the date of declaration. Hence a declared dividend must be classified
current liability in the balance sheet of the company.
(iv) Though the dividends can be declared only by a resolution of the share holders , if the articles of the company permit , the
directors can declare and interim dividend between two annual general meetings.
(v) Any dividend declared by the company reaming unpaid beyond 30 days of declaration , shall be transferred by the company
to a special account within 7 days of the expiry of the aforesaid period of 30days.
If the dividend is not claimed for a period of 7 years from the date of transfer to the special bank account , the unclaimed
amount must be transferred by the-2- company to the fund established under sec 205C. After such transfer, any share holder
entitled to claim such dividend may claim it from the government.

ACCOUNTING IN COMPUTERISED ENVIRONMENT


A computer is an electronic information-processing device that receives inputs, processes them with the help of set of instructions called
programme and, gives the desired results called the output. Computerised accounting is the performing of the various accounting functions
on a computer.A computerised accounting system has several advantages over the traditional manual accounting system such as speed,
accuracy, low cost, etc. Various accounting software are available in the market which can perform variety of functions. Banks are on the move
to fully computerised environment with centralised database. Core banking solutions are used for the same.
Data: Data mean any facts, observations, assumptions or occurrences. In accounts they mean accounting entries to be passed in books of
account to prepare financial tatement.
Software: A computer is run on the basis of a set of instructions called the software programme developed by a computer professional called the
programmer.
Computerised Accounting: Computerised accounting means maintaining books of account and preparing financial statement using a
computer.
Internet: Internet is the inter-connection between several computers of different types belonging to various networks all over the globe.
World Wide Web (WWW): WWW is a series of servers that are interconnected through hypertext.
Hypertext is a method of presenting information in which certain text is highlighted that, when selected,
displays more information on the particular topic. These highlighted items are called hyperlinks and allow the users to navigate from one
document to another that may be located on different servers.

MEMORY BASED QUESTIONS OF PREVIOUS YEARS ON AFFB – JAIIB PAPER II


PAPER SECOND – AFFB JAIIB JUNE 2012
1) The Capital contribution of RRBs should be in the ratio of---- by Centre,---- % by State and-----% by Sponsoring bank. a)50,15;35
b)50;25;25 c) 50; 10; 40 d) 40;20;40
2) A Bonds Redemption value (par value) is Rs. 1,000 years and annual coupon rate of 12% and has a term to maturity of 3 years. The going
market rate for similar, new investments is 10%. What is the price of this bond in secondary markets? a) Rs.1049.44 b) Rs.1059.55
c) Rs.1409.44 d) Rs.999.55
3) What is true about the Duration of a Bond?a)Duration is expressed in terms of years. b) Duration of a coupon-paying bond is always less
than its maturity.c)In Zero-coupon bonds where periodical interests are not paid out,duration will be equal to its maturity. d) All of the above
4) The duration of a Perpetual bond is equal to 1 + rI r, where r = current yield of the bond. a) True b) False c) Partially true
5) Zero Coupon bonds are those a)Which do not make a periodical coupon payment b)These bonds are bought for less than their face value
(at a discount). c) Are mostly issued in auctions by Treasury. d) All of the above
6) If the Coupon rate and the Discount rate (Market based) or the expected rates of return are same: a) The bond will be trading at par b)
Compiled by Mr. Sanjay Kumar Trivedy, Sr. Mgr., RSTC, mumbai
19
Bond will trade at a discount. c) Bond will trade at a premium. D)Coupon rate & discount rate have no connection with each other.
7) Depositories hold shares in form: a) Physical b) Dematerialized c) Either a or bd) Neither
8) Decrease of CRR & SLR has the effect of: a) Decreasing Credit creation b) Increasing Credit creation c) Increasing purchasing power of
money. d) None of the above.
9) The objectives of SLR are:a)To ensure that Banks remain solvent. B)To increase volumes in Government securities.c)To regulate expansion
of Credit d) All of the above
10) The loan values in Retail banking generally range between- a) Rs.20,000 Rs.One crore b) Rs. One crore - Rs. Ten crore c) Depends on
each Bank d) Up to Rs.100,000 only
11) Participatory Notes are contract notes issued by Foreign Institutional Investors (FII) to other entities that want to invest in Indian
markets but are not interested in registering themselves with: a) IRDA b) RBI c) SEBI d) All of the above
12) The Grand Bazaar super store sells its very famous soft bread at Rs,71- as against Rs 8.50 elsewhere. The shop is actually making a loss
in this transaction. However many people in the area go to Grand Bazaar to buy bread. While there, they normally buy other household
requirements also. What is the pricing strategy here? a) Special event pricing b) Geographical pricing c) Cash rebate d) Loss leader pricing
13) The monetary ceiling for referring cases for compromise settlement of dues of banks using the forum of Lok Adatat from Rs. 10 lac to
a) Rs. 12.5 lac b) Rs. 15 lac c) Rs. 17,5 lacd) Rs, 20 lac
14) Manufacturing Trading account is prepared by:
a) Trading concern b) Manufacturing concern c) Company d) Sole Trader
15) Left side of asset account is for: a) Recording decrease b) Recording increase c) Recording depreciation d) Recording sale
16) Which of the following have a credit balance? a) A/c payable b) A/c receivable c) Current a/c d) Discount a/c
17) Bill receivable endorsed is debited to: a) Debtor a/c b) Creditor a/c c) Bill payable a/c d) Bill receivable a/c
18) Straight-line method charges.depreciation: a) Same amount b) Amount is reduced every year c) Same rate d) Amount is increased
every year
19) Amount of depreciation provided in Sinking fund method when compared to annuity method is: a) Higher b) Lower c) Equal
d) Unpredictable
20) Capital expenditure relate to: a) Wages for construction b) Repair of machinery c) Payment of wages to Mr. X d) Both a & b
21) Revenue expenditure is: a) Day to day expenditure b) Monthly c) Yearly expenditure d) Half yearly
22) Drawings do not include: a) Payment of rent for proprietor residence b) Goods taken by properitor c)Amount withdrawn from bank
d)Amount withdrawn from bank for domestic use.
23) Nominal Capital is:a) Part of authorized capital issued. B)Maximum amount of share capital which company is authorized to issue c)
Amount of capital actually applied for
d) Amount of capital actually paid
24) Profit & loss a/c shows: a) Nominal a/c b) Real a/c c) Personal a/cd) Both a & b
25) Bad debts are_ for business: a) Loss b) Revenue c) Expense d) Expenditure
26) Value of asset can become zero in method:a) Straight line b) Written down c) Sinking fund d) Insurance policy
27) Depreciation is calculated onof asset.a) Cost price b) Market price c) Book value d) Invoice price
28) The amount realised from sale of obsolete asset is: a) Scrap value b) Residual value c) Both a & b d) Neither a nor b
29) If last installment is not paid, seller can the goods: a) Take away b) Not retain c) Sale d) Return
30) Interest suspense alc is shown in: a) Trading a/c b) Balance sheet c) Profit & loan a/c d) Venture a/c
31) Account sale statement shows: a) Gross sales b) Expenses incurred c) Bills accepted d) All above
32) Net worth of business means:a) Equity capital b) Total asset c) Fixed asset d) Total asset — total liability
33) Amount received from Insurance Company on maturity of joint life policy is distributed among partners: a) Equallyb) In profit sharing ratio
c) Capital ratiod)Not mentioned
34) At time of dissolution, an asset was taken by partner should be debited to: a) Partner capital a/c b) Realisation a/c c) Asset a/c d)
None
35) Which of the following is a Fixed Asset? a) Goodwill b) Land & Building c) Cash d) Capital
36) Scrap value of asset is deducted from original cost while calculating depreciation under:
a) Straight Line Method b) Reducing Balance Method c) Sinking Fund Method. d) Written Down value
37) The Book Value of machinery remains the same under which method of calculating depreciation: a) Straight Line Method b)
Written Down value c) Sinking Fund
38) Under Sinking Fund Method, at the end of the period funds are available for replacement of machinery: a) True b) False c)
Partially true
39) If there is no liability then: a) Asset > Capital b) Asset < Capital c) Asset = Capital d) None
40) Purchase a/c is credited for: a) Goods withdrawn b) Goods lost c} Good purchased d) Both a & b
41) Credit side of cash book (Triple column) consists of: a) Discount column b) Bank column c) Cash column d) All above
42) Preference shareholder have a right of: a) Dividend b) Interest c) Claim d) Commission
43) Which of following is not an asset? a) Fixed Asset b} Current Asset c) Capital d) Discount on share
44) Which of following has debit balance? a) Purchase a/c b) Sales a/c c) Capital a/c d) Loan a/c
ANSWER JAIIB SPECIAL
1 C 2 A 3 D 4 A 5 D 6 A 7 B
8 B 9 D 10 A 11 C 12 D 13 D 14 B
15 B 16 A 17 B 18 A 19 B 20 A 21 A
22 C 23 B 24 A 25 A 26 A 27 C 28 C
29 A 30 B 31 D 32 D 33 B 34 A 35 B
36 A 37 C 38 A 39 C 40 D 41 D 42 A
43 C 44 A

PAPER SECOND – AFFB JAIIB DEC 2010


5) A Bonds Redemption value (Par value) is Rs. 1,000 bears a annual coupon rate of 12% and has a term to maturity of 3 years. The
going market rate for similar. new investments is 10%. What is the price of this bond in secondary markets?
a) Rs.1049.44 b) Rs.1059.55
c) Rs.1409.44 d) Rs 999_55
9) A Banking company can hold shares in a company as pledgee, mortgagee or absolute owner, of an amount not
exceeding _______
a) 30% of its own paid up capital
b) 30% of its reserve
c) 30 % of paid up capital of the company
d) 30% of its paid up capital plus reserve or 30% paid up capital of that company, whichever is less
10) The Grand Bazaar super store sells its very famous soft bread at Rs.7/- as against Rs 8.50 elsewhere. The shop is actually making
a loss in this transaction. However many people in the area go to Grand bazaar to buy bread. While there, they normally buy other
household requirements also. What is the pricing strategy here?
a) Special event pricing b) Geographical pricing
c) Cash rebate d) Loss leader pricing
18) What is true about the Duration of a Bond?
a) Duration is expressed in terms of years
b) Duration of a coupon-paying bond is always less than its maturity.
c) In Zero-coupon bonds where periodical interests are not paid out, duration will be equal to its maturity
d) All of the above
19) The duration of a Perpetual bond is equal to 1 + r/ r, where r = current yield of the bond.
a) True b) False
20) Zero Coupon bonds are those ___
a) Which do not make a periodical coupon payment.
b) These bonds are bought for less than their face value (at a discount).
c) Are mostly issued in auctions by Treasuries
d) All of the above
Compiled by Mr. Sanjay Kumar Trivedy, Sr. Mgr., RSTC, mumbai
20
21) Debentures are_____
a) Normal types of bonds issued by Corporates
b) It is unsecured debt. backed only by the name and goodwill of the Company.
c) In the event of the liquidation of the corporation, holders of debentures are repaid before stockholders, but after other
secured creditors. d) All of the above
22) If the Coupon rate and the Discount rate (Market based) or the expected rates of return are same:
a) The bond will be trading at par
b) Bond will trade at a discount
c) Bond will trade at a premium
d) Coupon rate and discount rate have no connection with each other
e) 24) Cash Reserve Ratio to be held by Banks is
f) a) Voluntary b) Compulsory
g) c) Depends on the type of the Bankd) None of the above
h) 25) Depositories hold shares in form:
i) a) Physical b) Dematerialized
j) c) Either a or b d) Neither
28) Forecasting demand means:
a) Assessing whether the inventories are sufficient
b) Transporting the goods to the final destination
c) Assessment of the demand for the products
d) None of the above
29) _ provide Clearing House facilities for netting of
Payments and Security delivery:
a) Primary dealers b) Banks
c) Stock exchanges d) All of the above
30) Leasing, Hire purchase and Bill discounting are the main functional domains of:
a) NBFCs b) Banks c) Mutual fundsd) none
31) _ is the regulatory authority for Mutual funds:
a) IRDA b) SEBI
c) RBI d) Ministry of Corporate affairs
32) Primary dealers deal in ____
a) Shares b) Insurance
c) Government securities d) None of the above
33) Decrease of CRR & SLR has the effect of:
a) Decreasing Credit creation
b) Increasing Credit creation
c) None of the above.
34) The objectives of SLR are:
a) To restrict expansion of credit
b) To ensure that Banks remain solvent
c) To increase volumes in Government securities
d) All of the above
35) The Central Board of RBI consists of:
a) One Governor b) Maximum four Dy. Governors
c) 14 Directors d) All of the above
36) The loan values in Retail banking generally range
between _______
a) Rs20,000 - Rs. One croreb) Rs One crore - Rs. Ten crore
c) Depends on each Bank d) up to Rs.100,000 only
37) Participatory notes are Contract notes issued by Foreign Institutional Investors (FII) to other entities that want to invest in
Indian markets but are not interested in registering
themselves with ______
a) IRDA b) RBI c) SEBI d) All of the above
38) Initial Public Offering means that an existing or a new
Company is offering _____ to public / QIBs:
a) Existing equity (shares) b) New Equity
c) Either d) Neither
39) The aim of a Balanced Mutual Fund is ___
a) Growth or Price appreciation
b) Regular Income (dividends)
c) Both (a and b above) in a balanced manner
d) Investing both in shares & Bank deposits

41) On how many pillars do Basel- II norms rest?


a) One b) Two c) Three d) Four
42) In Call money markets, _ are allowed to trade:
a) Only Banks b) Only Primary dealers
c) Mutual funds d) all of the above
ANSWERS
1 C 2 B 3 D 4 C 5 A
6 C 7 C 8 D 9 D 10 D
11 C 12 A 13 A 14 C 15 A
16 C 17 B 18 D 19 A 20 D
21 D 22 A 23 D 24 B 25 B
26 B 27 D 28 C 29 C 30 A
31 B 32 C 33 B 34 D 35 D
36 A 37 C 38 C 39 C 40 D

41 C 42 A 43 C 44 A 45 B

PAPER SECOND – AFFB JAIIB SEP 2010

1) When a bill is accepted by drawee, the entry in the book of 'Drawer" shows:
a) Bilis receivable account b) Bills payable account
c) Rebate Account d) Drawee Account e) a and d
2) Bonus shares are issued to
a) Existing shareholders. b) New shareholders
c) Holders of partly paid-up shares d) None of the above
4) Bond holder are ______ of the issuer.
a) Shareholders b) Partner c) Creditors d) Owners
6) DCF methods are superior because:
a) Time value of money is taken into account.

Compiled by Mr. Sanjay Kumar Trivedy, Sr. Mgr., RSTC, mumbai


21
b) Entire cash flows are considered
c) Takes in to account the objective of shareholders d) Both a and b
8) According to which concept future income and income received in advance not accounted:
a) Cost Concept b) Revenue Concept
c) Dual Concept d) Concept of Conservation
9) According to which concept Asset = Liability + Capital:
a) Entity Concept b) Dual Concept
c) Monetary Concept d) Cost Concept
10) What is Owners Equity?
a) Capital only b) Capital + Drawings
c) Capital + Reserves and Surplus d) None of these
11) According to which concept, books of accounts are maintained.
a) Dual Concept b) Cost Conceptc) Monetary Conceptd) a and c
12) B. Ltd purchases on credit. The purchases are not as per specification, the company returns them to supplier, we will record it in
which book:
a) Cash book b} Purchase day book
c) Return outward book d) Return inward book
13) Depreciation appearing in the trial balance should be:
a) Shown as liability in balance sheet b) Debited to P and L account
c) Reduced from related asset d) Shown separately
14) Under which convention, secret reserves are not permitted:
a) Convention of Disclosure b) Convention of Materiality
c) Convention of Conservatism d) Convention of Consistency
15) At what rate % per annum will be the simple interest on a certain sum of money for 20 years will be equal to VP of the total amount at the
end of the period?
a) 5% b) 4% c) 6% d) 41/2%
16) Equal sums of money were lent to two persons at 16% simple interest p.a. for a period of 3 years and 5 years respectively. If the difference
in interest paid by them was Rs. 960, the sum lent to each of them is;
a) Rs.6300 b) Rs. 4200 c) Rs.5000 d) Rs .3000
17) A sum of money amounts to Rs. 1360 in 3 years and to Rs. 1120 in 6 years at simple interest p.a. The rate of interest is:
a) 10% b) 15.1% c) 13% d) 12%
18) The interest on a sum of money at the end of 61/4 years is 5/16 of the sum itself. In what time will the sum doubles itself at the same
rate?
a) 16 years b) 10 years c) 20 years d) 18 years
19)A sum of money amounts to Rs. 2240 at 4% p.a. simple interest in 3 years. The interest on the same sum for 6 months at 31/2% p.a. is:
a) Rs.50 b) Rs.30 c) Rs.35 d) Rs.40
20)Difference between simple interest and compound interest for 3 years at 10% p.a. is Rs. 93, the sum is:
a) Rs.2500 b) Rs.2750 c) Rs.3000 d) Rs.3500

Q A Q A Q A Q A Q
1 E 2 A 3 B 4 C A
5 B
6 D 7 C 8 B 9 B 10 C
11 D 12 C 13 B 14 A 15 B
16
. D 17 D 18 C
• 19 C 20 C

PAPER SECOND – AFFB JAIIB JUNE 2010


31) Cash sales = Rs. 35,000, Credit Sales = Rs. 40,000, Cost of Good Sold = Rs. 52,000 and Expenses on Sale = Rs. 6,700. Find value of
Net Profit.
a) Rs. 15,500 b) Rs. 16,000 c) Rs. 15,000d) Rs. 16,300
32) Discount columns of cash book are:
a) Only totaled b) Balancedc) Not totaled d) Not shown
33) Capital at end is Rs 21,500, drawings are Rs 4,000 and profits given to partner is equal to Rs 6,000; then opening capital is equal
to:
a) 14,000 b) 19,500 c) 20,000 d) 14,500
34) Trade discount is given on
a) Purchasing of goods in bulk b) Purchasing of goods in small quantity
c) On selling the goods d) On returning the goods sold
35) Articlesof Association must be submitted to registrar office within:
a) 15 days b) 30 days 030 days of incorporationd) No limit
36) Profit Sharing Ratio, if not stated in partnership deed is:
a) 2:3 b)equal c) 4 2 d) 5.4
37) Which Act makes sin* provision for the disclosure of essential information in account?
a, The income Tax Act b) The Companies Act
c)TheContractAct d)NegotiableInstrumentsAct
38)ClassifieddetailsofParticular transactioniscalled?
a)Cash book Ledger c) Journald) Trial Balance
39)Cost of goods sold is equal to :-
a) Opening stock+Purchas+Direct b) Closing Stock-Purchse+Opening Stock c) Closing Stock- Openind Stock+ Purchases
40) An entry of Rs 320 has been debited in books as Rs 250. It is an Error of :
a) Commission b) Omission c) Principle d) Compensating
41) If BRS is prepared with balance as per Passbook on 31 Dec., then out of Rs 20,500 paid by X by cheques on 31 Dec., a cheque of Rs. 7,500 was collected on 7th
jan, the effect will be:
a)Rs13,0000willbedeductedfrombalance
b)OnlyRs.7,500bededucted frombalance
c) Rs. 20.500 added is the balance d) None of these
42) When there is a positive balance, it appears on the in the---------------- in the cash book.
a) Cdt ride b) Debit side c) Totally ignored d) None
43) A business transaction is directly entered into :
a General Ledger b) Journal c) Sales Bookd) Trial Balance
44) Discount is an issue of Debenture is:
a) Capital Loss b) Capital gain c) Revenue loss d) Revenue gain
45)A.B C are 3 partners sharing profits in 4:3:2. D is admitted for 1/3' share of milts. New ratio is :
a) 4:3:2:1 b) 8:6:4:9 c) 5:3:3:2 d) 9:5:3.2
46) Revaluation account is to be credited when:
a) Receiptofamountfromdebtorwrittenoffearlierasbaddebt
b) Receipt ofamountonmaturitydaleofabillreceivable
c)Decrease in the value of realizable debts d) None of the above
47) If athbill made on 25th May matures after 4 months, then maturity date all be:
a) 26 Sept b) 25th Sept. c) 28th Septd) 1th Sept.
48) Goodwill means
a)Valueatreputationofthefirmb)Goodwishesofthepartners
c)Goodwishesof thecustomers
d)Itisatypeofwill madewithgoodintentions.
49)In whichmethodamountofdepreciationdecreaseseveryyear:
Compiled by Mr. Sanjay Kumar Trivedy, Sr. Mgr., RSTC, mumbai
22
a) Straight line method b) Written down value
c)Annuitymethod d)Sinkingfundethod
50)Debitsideofpartnerscapitalaicconsistsof:
a) Drawing b) Interest on drawing c) Loss transferred d) Above All
51 A Joint Venture should have at least :
a) 10 Co. Ventures b) 2 Co. Ventures
c) 12 Co. Ventures d) No limit
52) Double entry system is based on: a)Somebody loss equal to some body gain b) Every debit equal to every credit
c) what comes in is equal to what goes out d) All of the above
53) If goods of Rs. 4,000 were sold in auction at discount of 10%, then sale book will Include:
a) Rs. 4000 13) Rs. 3600 c) No entry d) Rs. 4400
54)In petty cash books, imprest money means:
a)Payment in advance b) Expenses in advance
c) Balance b/d d) Expenses of year
55) Depreciation account is a __
a)Real Account b) Nominal Account
c) Personal Acccont dl None of the above
56)Revaluationaccountistobedebitedwhen
a) Dishonour of a bill receivable and not expected to be realized.
b) Receipt of amount from debtor written off earrer as bad debt
c) Discount received from a creditor for Rs. 50- no recorded in the books
d) Noneofthese
57) Profit and loss account is a:
a) Real Account b) Nominal Account
c) Personal Account d) None of the above
58)Where doesnet profit appear in thebalance sheet?
a) Liabilities side b) Asset side c) Contingent uactiity d) Fund Flow
59) Amount of Rs. 1,000 debited in cash account for cash sales ------------- will be in sales account:
a) Debited b) Added c} Credited 0) Not shown
60) Incorrect accounting equation is:
a) Asset = Liability + Capital b) Capital = Asset + Liabilities
c) Liability = Asset + Capital d) Capital = Asset — Liability

31 D 32 A 33 B 34 A 35 C i36 B
37 A 38 B 39 A 40 A 41 B 42 B
43 B 44 B 45 B 46 A 47 C 48 A
49 B 50 D 51 B 52 D 53 C 54 A
55 B 56 D 57 B 58 A 59 C 60 C

PAPER SECOND – AFFB JAIIB DEC 2010

51) Manufacturing Trading account is prepared by:


a) Trading concern b) Manufacturing concern
c) Company d) Sole Trader
52) Bill of exchange cannot be_
a) Retained till maturity b) Discounted with bank
c) Endorsed to anybody d) None of these
53) What is 'Tangible Net Worth of a Firm?
a) Capital only b) Bank OD
c) Capital + Reserves & Surplus - Intangibles
d) Debtors
54) In a Banks balance sheet bad debts and provisions for doubtful debts are shown under:
a) Loans and Advances b) Provisions
c) Reserves d) Contingent Liabilities
55) Bill receivable endorsed are debited to:
a) Debtor a/c b) Creditor a/c
c) Bill payable a/c d) Bill receivable a/c
56) Straight-line method charges depreciation:
a) Same amount b) Amount is reduced every year
c) Same rate d) Amount is increased every year
57) Amount received from Insurance Company on maturity of joint life policy is distributed among partners:
a) Equally b) In profit sharing ratio
c) Capital ratio d) Not mentioned
58) At time of dissolution, an asset was taken by partner should be debited to:
a) Partner capital a/c b) Realisation a/c
c) Asset a/c d) None
59) Which of the following is a fixed Asset?
a) Goodwill b) Land & building c) Cash d) Capital
60) Registration of partnership is:
a) Compulsory b) Optional
c) Compulsory for partners d) none
Amount of depreciation provided in sinking fund method when compared to annuity method is:
a) Higher b) Lower c) Equal d) Unpredictable
62) Clerical error includes:
a) Error of omission b) Principal error
c) Error of commission d) Both a & c
63) Capital expenditure relate to:
a) Wages of construction b) Repair of machinery
c) Payment of wages to Mr X d) Both a & b
64) _____ is an example of contingent liability
a) Bank Guarantee b) Deposits
c) Loans and Advances d) Provisions
65) Revenue expenditure are:
a) Day to day expenditure b) Monthly
c) Yearly expenditure d) Half yearly
66) Drawings do not include:
Payment of Rent for properitor residence
Goods taken by properitor
Amount withdrawn from bank
Amount withdrawn from bank for domestic use.
67) Nominal Capital is:
Part of authorised capital issued
Maximum amount of share capital which company is authorized to issue
Amount of capital actually applied for
Amount of capital actually paid
68) Profit & loss a/c shows:
Compiled by Mr. Sanjay Kumar Trivedy, Sr. Mgr., RSTC, mumbai
23
a) Nominal a/c b) Real a/c
c) Personal a/c d) Both a & b
69) Bad debts are ____ for business:
a) Loss b) Revenue
c) Expense d) Expenditure
70) Value of asset can become zero in _ method:
a) Straight line b) Written down
c) Sinking fund d) Insurance policy
71) Depreciation is calculated on__of asset.
a) Cost price b) Market price
c) Book value d) Invoice price
72) The amount realised from sale of obsolete asset is:
a) Scrap value b) Residual value
c) Both a & b d) Neither a nor b
73) if last installment is not paid, seller can the goods:
a) Take away b) Not retain
c) Sale d) Return
74) Bonus shares are issued to ____ •
a) Existing shareholders. b) New shareholders
c) Holders of partly paid-up shares d) None of the above
75) Account sale statement shows:
a) Gross sales b) Expenses incurred
c) Bills accepted d) All above
76) Relationship of Consignor and Consignee is:
a) Principal /agent b) Partner
c) Share holder d) Members
77) Parties involved in Joint Venture are called:
a) Co-ventures b) Co-owners
c) Share holder d) Both a & b
78) Net worth of business means:
a) Equity capital b) Total asset
c) Fixed asset d) Total asset — total liability
79) The balance of Petty cash a/c is shown as:
a) Expense b) Asset c) Loss d) Revenue
80) If BRS is prepared with balance of cash book, it will
show____ per pass book.
a) Balance b) Overdraft c) Both a & b d) Either a or b
81) Who is the Debtor?
a) A person from whom the firm has made credit purchases.
b) A person to whom goods have been sold on credit
c) The owner d) Bank
82) Interest allowed on Capital of a partner is calculated on:
a) Opening capital b) Capital at end
c) Average capital d) Capital less drawing
83) Goodwill means _____
a) Value of reputation of the firm
b) Good wishes of the partners
c) Good wishes of the customers
d) Type of will made with good intentions.
84) Revaluation account is to be credited when
a) Dishonour of a bill receivable and not expected to be realized b)Motor car with book value zero valued at Rs. 81,000/-
c) Shares of A Bank Ltd. purchased at par (Face value Rs. 10) were having market value of Rs. 9.50 per share. d)None of the
above
85) Debit side of Partner capital a/c consist of:
a) Drawing b) Interest on drawing c) Lass transferred
d) Above All
86) Iftha bill made onth25th May matures after 451months, then maturity date will be:
a) 26 Sept_ b) 25 Sept. c) 28"' Sept.d) 1 Sept.
87) Double entry system is based on:
a) Somebody loss = to some body gain
b) Every debit equal to every credit
c) What comes in is = to what goes outd) All of the above
88) Capital at end is Rs 21,500, drawings are Rs 4,000 and profits given to partner is equal to Rs 6,000 then opening capital is equal to:
a) 14,000 b) 19.500 c) 20,000 d) 14,500
89) Balance sheet contains only Personal and Real accounts:
a) True b) False c) None of aboved) a and b
90) Goods lost by fire should be credited to:
a) Purchase a/c b) Goods lost by fire a/c
c) Sales a/c d) No entry

51 B 52 D 53 C 54 B
55 B 56 A 57 B 58 A 59 B 60 B
61 B 62 D 63 A 64 A 65 A 66 C
67 B 68 A 69 A 70 A 71 C 72 C
73 A 74 A 75 D 76 A 77 D 78 D,
79 B 80 D 81 C 82 A 83 A 84 B
85 D 86 C 87 D 88 -B 89 A 90 A

PAPER SECOND – AFFB JAIIB MAY 2013

71) A Bonds Redemption value (Par value) is Rs. 1,000 bears a annual coupon rate of 12% and has a term to maturity of 3 years. The
going market rate for similar, new investments is 10%. What is the price of this bond in secondary markets? a) Rs 1049 421 b)
Rs.1059.55 c) Rs.1409.44 d) Rs.999.55
72) What is true about the Duration of a Bond? A)Duration js expressed in terms of years b)Duration of a coupon-paying bond is always
less than its maturity.
C)In Zero-coupon bonds where periodical interests are not paid out, duration will be equal to its maturity d) All of the above
73) The duration of a Perpetual bond is equal to 1 + r/ r, where r = current yield of the bond. a) True b) False
74)Zero Coupon bonds are those ______ a) Which do not make a periodical coupon payment. b)These bonds are bought for less than
their face value (at a discount).
C)Are mostly issued in Auctions by Treasuries d) All of the above
75) Debentures are _________ . A)Normal types of bonds issued by Corporates b)It is unsecured debt, backed only by the name and
goodwill of the Company.
c)In the event of the liquidation of the corporation, holders of debentures are repaid before stockholders, but after other secured creditors.
d)All of the above
76) If the Coupon rate & the Discount rate (Market based) or the expected rates of return are same: a) The bond will be trading at par
b) Bond wilt trade at a discount
c) Bond will trade at a premium d) Coupon rate & discount rate have no connection with each other
Compiled by Mr. Sanjay Kumar Trivedy, Sr. Mgr., RSTC, mumbai
24
77) Manufacturing Trading account is prepared by: a) Trading concern b) Manufacturing concern c)Company d) Sole Trader
78) Bill of exchange cannot be ______ . a) Retained till maturity b) Discounted with bank c) Endorsed to anybody d) None of these
79) Left side of asset a/c is for: a) Recording decrease b) Recording increase c)Recording depreciation d) Sole trader
80) Which of the following have a credit balance? at A/c payable b) A/c receivable c) Current a/c d) Discount a/c
81)Bill receivable endorsed are debited to: a) Debtor at b) Creditor a/ c)Bill payable al d) Bill receivable a/c
82)Straight-rine method charges depreciation: a) Same amount b) Amount is reduced every year c) Same rate d) Amount is increased
every year
83)Amount received from Insurance Company on maturity of joint life policy is distributed among partners: a) Equally b) In profit sharing ratio
c) Capital ratio d) Not mentioned
84)At time of dissolution, an asset was taken by partner should be debited to: a) Partner capital a/c b) Realisation alc c) Asset a/c
d) None
85)Which of the following is a fixed Asset? a) Goodwil b) Land & building c) Cash d) Capital
86)Registration of partnership is: • a) Compulsory b) Optional c) Compulsory for partners d) none
87)Amount of depreciation provided in sinking fund method when compared to annuity method is: a) Higher b) Lower c) Equal d)
Unpredictable
89)Clerical error includes: a) Error of omission b)Principal error c) Error of commission d) both a & c
90)Capital expenditure relate to: a) Wages of construction b)Repair of machinery c) Payment of wages to Mr. X d) both a&b
90) In Partnership aim of Joint Life Policy is: a) Make profit b)Sufficient cash to settle claim of deceased partners c)Strength financial
position d) None
91) Revenue expenditure are:a) Day to day expenditure b) Monthly c) Yearly expenditure d) Half yearly
92) Drawings do not include: a)Payment of Rent for properitor residence b)Goods taken by properitor c)Amount withdrawn from bank
d)Amount withdrawn from bank for domestic use.
93) Nominal Capital is: a)Part of authorised capital issued b)Maximum amount of share capital which company is authorized to issue c)
Amount of capital actually applied for d)Amount of capital actually paid
94) Profit & loss a/c shows: a) Nominal a/c b) Real a/c c) Personal a/cd) Both a & b
95) Bad debts are for business: a) Loss b) Revenue c) Expense d) Expenditure
96) Value of asset can become zero in method:
a) Straight lineb) Written down c) Sinking fund d) Insurance policy
97) Depreciation is calculated on of asset
a) Cost price b) Markel price c) Book value d) Invoice price
98) Financial Leverage Ratio is also called
a) Debt equity ratiob) Capital gearing ratio
c) Debt service coverage ratio d) Assets turnover ratio
99) The amount realised from sale of obsolete asset is:
a) Scrap value b) Residual value c) Both a & b d) Neither a nor b
100) If last installment is not paid, seller can_ the goods:
a) Take away b) Not retain c) Sale d) Return
65 0 66 D 67 B 68 B 69 C 70 D 71 A 72 D
73 A 74 D 75 D 76 A 77 B 78 D 79 B 80 A
81 B 82 A 83 B 84 A 85 B 86 B 87 B 88 D
89 A 90 B 91 A 92 C 93 B 94 A 95 A 96 A
97 C 98 B 99 C 100 A 101 A 102 A 103 B 104 A

PAPER JAIIB – II AFFB DEC-2012


ACCOUNTING & FINANCE FOR BANKERS
71) A Bonds Redemption value (Par value) is Rs. 1,000 bears a annual coupon rate of 12% and has a term to maturity of 3 years. The going
market rate for similar, new investments is 10%. What is the price of this bond in secondary markets? a) Rs.1049.44 b) Rs.1059.55
c) Rs.1409.44 d) Rs.999.55
72) What is true about the Duration of a Bond? Duration is expressed in terms of years. b) Duration of a coupon-paying bond is always less
than its maturity. C) In Zero-coupon bonds where periodical interests are not paid out, duration will be equal to its maturity d) All of
the above
73) The duration of a Perpetual bond is equal to 1 + rl r, where r = current yield of the bond. a) True b) False
74) Zero Coupon bonds are those :
a) Which do not make a periodical coupon payment. b)These bonds are bought for less than their face value (at a discount). c) Are mostly
issued in Auctions by Treasuries d)All of the above
75) Debentures are ____________ • a) Normal types of bonds issued by Corporates b)It is unsecured debt, backed only by the name
and goodwill of the Company. C) In the event of the liquidation of the corporation, holders of debentures are repaid before stockholders,
but after other secured creditors. D) all of the above
76) If the Coupon rate & the Discount rate (Market based) or the expected rates of return are same:
a) The bond will be trading at par b) Bond will trade at a discount c) Bond will trade at a premium d) Coupon rate .& discount rate have
no connection with each other
77) Manufacturing Trading account is prepared by: a) Trading concern b) Manufacturing concern c) Company d) Sole Trader
78) Bill of exchange cannot be a) Retained till maturity b) Discounted with bank c) Endorsed to anybody d) None of these
79) Left side of asset alc is for: a) Recording decrease b) Recording increase c) Recording depreciation d) Recording sale
80) Which of the following have a credit balance? a) A/c payable b) A/c receivable c) Current a/c d) Discount a/c
81) Bill receivable endorsed are debited to: a) Debtor a/c b) Creditor a/c c) Bill payable a/c d) Bill receivable a/c
82) Straight-line method charges depreciation: a) Same amountb) Amount is reduced every year c) Same rate d) Amount is increased
every year
83) Amount received from Insurance Company on maturity of joint life policy is distributed among partners: a) Equally b) In profit
sharing ratio c) Capital ratio d) Not mentioned
84) At time of dissolution, an asset was taken by partner should be debited to: a) Partner capital a/c b) Realisation a/c c) Asset a/c
d) None
85) Which of the following is a fixed Asset? a) Goodwill b) Land & building c) Cash d) Capital
86) Registration of partnership is: • a) Compulsory b) Optional c) Compulsory for partners d) none
87) Amount of depreciation provided in sinking fund method when compared to annuity method is: a) Higher b) Lower c)
Equal d) Unpredictable
88) Clerical error includes: a) Error of omission b) Principal error c) Error of commission d) Both a & c
89) Capital expenditure relate to: a) Wages of construction b) Repair of machinery c) Payment of wages to Mr. X d) Both a & b
- 90) In Partnership aim of Joint Life Policy is:a) Make profit b)Sufficient cash to settle claim of deceased partners c)Strength financial
position d) None
91) Revenue expenditure are: a) Day to day expenditure b) Monthly c) Yearly expenditure d) Half yearly
92) Drawings do not include: a)Payment of Rent for properitor residence b)Goods taken by properitor c) Amount withdrawn from bank d)
Amount withdrawn from bank for domestic use.
93) Nominal Capital is: a)Part of authorised capital issued b) Maximum amount of share capital which company is authorized to issue
c)Amount of capital actually applied for d)Amount of capital actually paid
94) Profit & loss a/c shows: a) Nominal a/c b) Real a/c c) Personal a/cd) Both a & b
95) Bad debts are for business: a) Loss b) Revenue c) Expense d) Expenditure
96) Value of asset can become zero in method: a) Straight line b) Written down c) Sinking fund d) Insurance policy
97) Depreciation is calculated on of asset. a) Cost price b) Market price c) Book value d) Invoice price
98) Financial Leverage Ratio is also called a) Debt equity ratio b) Capital gearing ratio c) Debt service coverage ratio d) Assets turnover
ratio
99) The amount realised from sale of obsolete asset is: a) Scrap value b) Residual value c) Both a & b d) Neither a nor b '
100) If last installment is not paid, seller can the goods: a) Take away b) Not retain c) Sale d) Return

71 A 72 D 73 A 74 D 75 D 76 A 77 B 78 D
79 B 80 A 81 B 82 A 83 B 84. A 85 B 86 B
87 B 88 D 89 A 90• 91 A 92 C 93 B 94 A
95 A 96 A 97 C B
98 B 99 C 10 A
0
Compiled by Mr. Sanjay Kumar Trivedy, Sr. Mgr., RSTC, mumbai
25
PAPER JAIIB – II AFFB JUNE 2013
1) A Bonds Redemption value (Par value) is Rs.1,000 bears an annual coupon rate of 12% and has a term to maturity of 3 years. The going
market rate for similar, new investments is 10%. What is the price of this bond in secondary markets? a) Rs.1049.44 b) Rs.1059.55 c)
Rs.1409.44 d) Rs.999.55
2) What is true about the Duration of a Bond? A) Duration is expressed in terms of years. B)Duration of a coupon-paying bond is always less
than its maturity. C) In Zero-coupon bonds where periodical interests are not paid out, duration will be equal to its maturity.d) All of the
above
3) A Banking company can hold shares in a company as pledgee, mortgagee or absolute owner, of an amount not exceeding__: a)30% of its
own paid up capital b)30% of its reserve c) 30 % of paid up capital of the company d)30% of its paid up capital plus reserve of the bank or
30% paid up capital of that company, whichever is less
4) A Company has been sanctioned overdraft limit of Rs. 5,00,000/- against pledge of shares. In this case: a)The charge must be registered
with the Registrar of Cos. B) The charge must be registered with the Controller of Capital Issues. c) Registration of charge is not necessary. D)
The charge must be registered with the Registrar of Companies if the advance exceed 30% of the paid up share capital & free reserve of the
borrowing company.
5) Zero Coupon bonds are those ________:a) Which do not make a periodical coupon payment b) These bonds are bought for less than their
face value (at a discount). c) Are mostly issued in Auctions by Treasuries.
D)All of the above
15) Zero Coupon bonds are those ________: a) Which do not make a periodical coupon payment b)These bonds are
bought for less than their face value (at a discount). c) Are mostly issued in auctions by Treasuries d)All of the above
16) If the Coupon rate & the Discount rate (Market based) or the expected rates of return are same: a) The bond will be trading at par b)
Bond will trade at a discount C) Bond will trade at a premium
c) Coupon rate & discount rate have no connection with each other
18) Depositories hold shares in _____ form: a) Physical b) Dematerialized c) Either a or bd) Neither
19) _____ provide Clearing House facilities for netting of Payments and Security delivery: a) Primary dealers b) Banks c) Stock exchanges d)
All of the above
20) Leasing, Hire purchase and Bill discounting are the main functional domains of: a) NBFCs b) Banks c) Mutual funds d) none
21) _____ is the regulatory authority for Mutual funds: a) IRDA b) SEBI c) RBI d) Ministry of Corporate affairs
22) Primary dealers deal in : a) Shares b) Insurance c) Government securities d) None
23) The objectives of SLR are: a) To restrict expansion of Credit b) To ensure that Banks remain solvent c) To increase volumes in
Government securities d) All of the above
24) The loan values in Retail banking generally range between: a) Rs.20,000-Rs.One crore b) Rs. One crore -Rs. Ten crore c)
Depends on each Bank d) up to Rs.100, 000 only
25) Participatory notes are Contract notes issued by Foreign Institutional Investors (FII) to other entities that want to invest in Indian markets
but are not interested in registering themselves with :
a) IRDA b) RBI c) SEBI d) All of the above
26) Initial Public Offering means that an existing or a new Company is offering to public / QIBs: a) Existing equity (shares) b) New Equity
c) Either d) Neither
27) The aim of a Balanced Mutual Fund is : a) Growth or Price appreciation b) Regular Income (dividends) c) Both in a balanced manner
d) Investing both in shares & Bank deposits
28) On how many pillars do Basel-02 norms rest? a) One b) Two c) Three d) Four
29) In Call money markets, are allowed to trade: a) Only Banks b) Only Primary dealers c) Mutual funds d) all
37) Banks saddled with very high Non Performing assets may find it difficult to extend fresh loans. They may then try to improve their other
business such as non fund based business (LC, LG) and services (cheque collections) by offering lower service charges. This method of pricing
has the objective of: a) Profit b) Communicating image c) Product quality d) Survival
38) The Grand Bazaar super store sells its very famous soft bread at Rs.20/- as against Rs 18.50 elsewhere. The shop is actually making a loss
in this transaction. However many people in the area go to Grand bazaar to buy bread. While there, they normally buy other household
requirements also. What is the pricing strategy here? a) Special event pricing b) Geographical pricing c) Cash rebate d) Loss leader
pricing
39) When colour television was first introduced in Mumbai, each set was priced at Rs 16000/. After two months of sales, the prices were
seen at Rs 15000/. Two months later, the price went down further to Rs.14000/. In this hypothetical example, what was the pricing
strategy of the TV company? a) Introductory pricing b) Discriminatory pricing c) Market skimming pricing d) None of the above
41) Forecasting demand means: a)Assessing whether the inventories are sufficient b) Transporting the goods to the final destination c)
Assessment of the demand for the products d) None of the above
42) Storage, protective packaging, transportation are the tasks associated with : a) Demand forecasting b) Order processing c)
Inventory Management d) All of the above
43) The Gilt-Edged security market is the market for which of the following? a) Market for trading in gold b) Market for trading in sliver c)
Market for government securities d) Market for industrial securities
44) Which of the following can be called as intermediary in the Financial System? a) Insurance companies b) Banking companies c)
Mutual funds d) Financial institutions e) All the above
45) Cash Reserve Ratio is to be maintained by banks with reference to which of the following? a) Time liabilities b) Demand liabilities c)
Time and demand liabilities d) Gross time & demand liabilities
e) Net demand and time liabilities
46) What is the minimum CRR as per the relevant Act? a) No minimum b) 4% c) 3% d) 6%
47) Which kind of assistance is provided by EXIM Bank to Indian companies? a) Direct loans to exporters b) Technology services c)
Consultancy services d) Credit for export of capital goods
e) All the above
51) Gagan is maintaining a Current account, Savings Bank account and FD account. Bank receives Garnishee order on Current account. It will
apply to: a) CA account b) SF account c) FD accountd) All accounts
52) Acceptance is not required in the case of: a) Usance Promissory Note b) Demand bill of exchange c) Both of these d) None of these
53) Which of the following can endorse a negotiable instrument? a) A minor b) Married woman c) An illiterate person d) Blind
persons e) All the above
54) Matured, but not paid, term deposits are treated as: a) Demand Liability b) Time Liability c) No Liability d) Contingent Liability
62) A hire-purchase contract is a: a) Contract of bailment b) Contract of sale Contract of bailment followed by Contract of sale Contract of
lease followed by Contract of bailment
63) Association, Agreement, Business, sharing of profits and
________ sum up the features of a partnership: a) Consideration b) Mutual agency c) Liability d) Interest
69) Govt. has hiked the monetary ceiling for referring cases for compromise settlement of dues of banks using the forum of Lok Adalat from
Rs. 10 lac to : a) Rs 12.5 lac b) Rs.15 lac c) Rs.17.5 lac d) Rs. 20lac
71) Manufacturing Trading account is prepared by: a) Trading concern b) Manufacturing concern c) Company d) Sole Trader
72) Left side of asset a/c is for: a) Recording decrease b) Recording increase c) Recording depreciation d) Recording sale
73) Which of the following have a credit balance? a) A/c payable b) A/c receivable c) Current a/c d) Discount a/c
74) Cash sales = Rs. 35,000, Credit Sales = Rs. 40,000, Cost of Good Sold = Rs. 52,000 and Expenses on Sale = Rs. 6,700. Find value of Net Profit.
a) Rs. 15,500 b) Rs. 16,000 c) Rs. 15,000 d) Rs. 16,300
75) Discount columns of cash book are: a) Only totaled b) Balanced c) Not totaled d) Not shown
76) Capital at end is Rs 21,500, drawings are Rs 4,000 and profits given to partner is equal to Rs 6,000; then opening capital is equal to: a)
14,000 b) 19,500 c) 20,000 d) 14,500
77) Trade discount is given on ________: a) Purchasing of goods in bulk b) Purchasing of goods in small quantity c) On selling the goods d)
On returning the goods sold
78) Articles of Association must be submitted to registrar office within: a) 15 days b) 30 days c) 30 days of incorporation d) No
limit
79) Profit Sharing Ratio, if not stated in partnership deed is: a) 2:3 b) Equal c) 4:2 d) 5:4
80) Which Act makes ample provision for the disclosure of essential information in accounts? a) The Income Tax Act b) The Companies Act c)
The Contract Act d) Negotiable Instruments Act
81) Classified details of particular transaction is called? a) Cashbook b) Ledger c) Journal d) Trial Balance
82) Cost of Goods sold is equal to:a) Opening stock + Purchase + Direct expenses — Closing stock b) Closing stock— Purchase + Opening stock
c) Closing stock— Opening stock + Purchases
83) An entry of Rs 320 has been debited in books as Rs 250. It is an error of: a) Commission b) Omission c) Principle d) Compensating
84) Bill receivable endorsed are debited to: a) Debtor a/c b) Creditor a/c c) Bill payable a/c d) Bill receivable a/c
85) Straight-line method charges depreciation: a) Same amount b) Amount is reduced every year c) Same rate d) Amount is increased
every year
86) Amount received from Insurance Company on maturity of joint life policy is distributed among partners: a) Equallyb) In profit sharing ratio
c) Capital ratiod) Not mentioned
Compiled by Mr. Sanjay Kumar Trivedy, Sr. Mgr., RSTC, mumbai
26
87) Which of the following is a fixed Asset? a) Goodwill b) Land & building c) Cash d) Capital
88) Revenue expenditure are: a) Day to day expenditure b) Monthly c) Yearly expenditure d) Half yearly
89) Amount of depreciation provided in sinking fund method when compared to annuity method is: a) Higher b) Lower c) Equal d)
Unpredictable
90) Clerical error includes: a) Error of omission b) Principal error c) Error of commission d) Both a & c
91) Capital expenditure relate to: a) Wages of construction b) Repair of machinery c) Payment of wages to Mr. X d) Both a & b
92) In partnership aim of joint life policy is: a) Make profit b) Sufficient cash to settle claim of deceased partners c) Strength financial position
d) None of these
93) Drawings do not include: a)Payment of Rent for properitor residence b) Goods taken by properitor c) Amount withdrawn from bank d)
Amount withdrawn from bank for domestic use.
94) Profit & loss a/c shows: a) Nominal a/c b) Real a/c c) Personal a/c d) Both a & b
95) Bad debts are _______ for business: a) Loss b) Revenue c) Expense d) Expenditure
96) Value of asset can become zero in _____ method: a) Straight line b) Written down c) Sinking fund d) Insurance policy
97) Depreciation is calculated on _____ of asset. a) Cost price b) Market price c) Book valued) Invoice price
98) The amount realised from sale of obsolete asset is: a) Scrap value b) Residual value c) Both a & b d) Neither a nor b
99) If last installment is not paid, seller can___ the goods: a) Take away b) Not retain c) Sale d) Return
100) Interest suspense a/c is shown in: a) Trading a/c b) Balance sheet c) Profit & loan a/c d) Venture a/c
101) Account sale statement shows: a) Gross sales b) Expenses incurred c) Bills accepted d) All
103) Parties involved in Joint Venture are called: a) Co-ventures b) Co-owners c) Share holder d) Both a & b
104) Net worth of business means: a) Equity capital b) Total asset c) Fixed asset d) Total asset—total liability
105) CTDI announces that if 50 students join and place orders for their Marketing text book, their offer price will reduce from Rs.200 to
Rs.175. What pricing method are they using here?
a) Value pricing b) Group pricing c) English auction d) Dutch auction
106) Mr.Gopal goes to purchase white canvas shoes for his son to the local BATA store in the Monsoon season. The price tag says Rs.49.95. He
remembers that the adjacent shoe shop is selling the same shoes for Rs 50/-. He buys from BATA store and feels that he has received a
bargain. What is the pricing strategy adopted by Bata? a) Promotional pricingb) Seasonal discount c) Value pricing d) Psychological pricing
120) Where are American Depository receipts traded? a) In USA b) Outside USA c) Both of the above d) In India only
121) Is it possible to have an IDR for an Indian Company? A) Yes, it is possible b) No, IDRs can be issued only for Foreign Registered
Companies c) Varies from case to case d) None of the above
124) Liabilities of an entity consist of :a) a) The liabilities consist of claims of the owners b) The liabilities consist of claims of the owners and
outsiders c) The liabilities consist of claims of the outsiders d) None of the above
125)X and Y are two partners in a firm sharing profits and losses as 2:1. "Z" is admitted as a partner with 25% share in the profits of the firm.
The
new profit sharing ratio, after admission of Z would be : a) 37.50 : 37.50 :25 b) 20:10:10
c) 1:1:1 d) None of the above
126) THE security interests in the NPAs cannot be sold by a Bank:
a) Without intervention of courts b) After the default occurs
c) After the debt has become time barred under limitation act
d) With prior notice of demand
127) The Securitization act does not cover________:
a) Private Sector Banks b) Consortium of Banks
c) Private Financial Institutions d) Chit Fund Companies
128) Limitation period is case of pledged goods is:
a) 3 years from the date of pledge b) 3 years from the date of documents.
c) 5 years from the date of pledge. d) None of these - no limitation
129) Which amongst the following can not be a source for issue of bonus shares?
a) Share premium b) Capital reserve c) Capital redemption reserve
d) Revaluation reserve created by revaluation of fixed assets
130) Find the "Odd man" out?
a) Income on investments b) Profit on sale of investments
c) Profit on revaluation of investments d) Profit on exchange transactions
131) In Banks, matured (but not paid) term deposits are to be shown under:
a) Demand deposits b) Saving bank deposits
c) Term deposits d) Other liabilities & Provisions
132) Advances given to a staff by a bank as a employer should be included in________:
a) Other assets b) Advances c) Investments d) None of the above
133) If there is no liability then:
a) Asset > Capital b) Asset < Capitalc) Asset = Capital d) None
134) Credit side of cash book (Triple column) consists of:
a) Discount column b) Bank column c) Cash column d) all above
135) Preference shareholders have a right of:
a) Dividend b) Interest c) Claimd) Commission
136) Purchase a/c is credited for:
a) Goods withdrawn b) Goods lost c) Good purchased d) both a & b
137) If there is no liability then:
a) Asset > Capital b) Asset < Capitalc) Asset = Capital d) None
138) Credit side of cash book (Triple column) consists of:
a) Discount column b) Bank column c) Cash column d) all above
139) Preference shareholders have a right of:
a) Dividend b) Interest c) Claimd) Commission
140) Purchase a/c is credited for:
a) Goods withdrawn b) Goods lost c) Good purchased d) both a & b
ANSWER (BRAIN STORMING)

1 A 2 D 3 D 4 C 5 D 6 D 7 A 8 C
9 C 10 B 11 C 12 B 13 D 14 C 15 D 16 A
17 D 18 D 19 C 20 A 21 B 22 C 23 D 24 D
25 A 26 C 27 C 28 C 29 D 30 C 31 A 32 A
33 B 34 A 35 B 36 D 37 D 38 D 39 C 40 B
41 C 42 C 43 C 44 E 45 E 46 A 47 E 48 D
49 A 50 C 51 D 52 A 53 E 54 A 55 D 56 B
57 A 58 B 59 A 60 D 61 D 62 C 63 B 64 B
65 B 66 A 67 A 68 E 69 D 70 D 71 B 72 B
73 A 74 D 75 A 76 B 77 A 78 C 79 B 80 A
81 B 82 A 83 A 84 B 85 A 86 B 87 B 88 A
89 B 90 D 91 A 92 B 93 C 94 A 95 A 96 A
97 C 98 C 99 A 100 B 101 D 102 A 103 D 104 D
105 B 106 D 107 B 108 D 109 A 110 A 111 C 112 D
113 D 114 B 115 A 116 A 117 D 118 E 119 B 120 A
121 B 122 D 123 D 124 B 125 B 126 B 127 C 128 D
129 D 130 D 131 A 132 A 133 A 134 C 135 B 136 A
137 C 138 C 139 C 140 D 141 A 142 D 143 B 144 D

Compiled by Mr. Sanjay Kumar Trivedy, Sr. Mgr., RSTC, mumbai


27
PAPER JAIIB – II AFFB JUNE 2014
91) A Bonds Redemption value (Par value) is Rs. 1,000 bears a annual coupon rate of 12% and has a term to maturity of 3 years. The going market rate for similar, new
investmentsis10%. What istheprice of thisbond in secondarymarkets?
a) Rs.1049.44b) Rs.1059.55 c) Rs.1409.44 d) Rs.999.55
92) What is true about the Duration of a Bond?
a) Duration is expressed in terms of years.
b) Duration of a coupon-paying bond is always less than its maturity.
c) In Zero-coupon bonds where periodical interests are not paid out, duration will be equal to its maturity
d) All of the above
93) The duration of a Perpetual bond is equal to 1 + r/ r,where r = current yield of the bond.
a) True b) False
94)Zero Couponbondsarethose________:
a) Which do not make a periodical couponpayment.
b) These bonds are bought for less than their face value (at a discount).
c) Are mostly issued in Auctions by Treasuries d) All of the above
95)Debenturesare ________:
a) Normal typesofbondsissued byCorporates
b) It is unsecured debt, backed only by the name and goodwill of the Company.
c) In the event of the liquidation of the corporation, holdersof debentures are repaid before stockholders, but after other secured creditors.
d) d)All ofthe above
96) If the Coupon rate and the Discount rate (Market based) or the expected rates of return are same:
a) The bond willbe tradingat par b) Bond willtrade at adiscount
c) Bond will trade at a premium
d) Coupon rate & discount rate have no connection with each other
97) Bill of exchange cannot be______:
a) Retained till maturityb) Discounted with bank
c) Endorsed to anybodyd) None of these
98) Left side of Asset a/c is for:
a) Recording decrease b) Recording increase
c) Recording depreciation d) Recording sale
99)Which of the followinghave a Credit balance?
a) A/c payable b) A/c receivable c) Current a/c d) Discount a/c
100) Bill Receivable endorsed are debited to:
a) Debtor a/c b) Creditor a/c c) Bill payable a/c d) Bill receivable a/c
101)Straight-line method chargesdepreciation:
a) Same amount b) Amount is reduced every year
c) Same rate d) Amount is increased every year
102)AmountreceivedfromInsurance CompanyonmaturityofJointLifePolicyisdistributedamongpartners:
a)Equally b)Inprofitsharingratio c)Capitalratio d)Notmentioned
103) At time of dissolution, an asset was taken by partner should be debited to:
a) Partner capital a/c b) Realisation a/c c) Asset a/c d) None
104)Which of the followingis a FixedAsset?
a) Goodwillb) Land & Building c) Cash d) Capital
105) Amount of depreciation provided in sinking fund method when compared to annuity method is:
a) Higher b) Lower c) Equal d) Unpredictable
106) Clerical error includes:
a) Error of omission b) Principal error
c) Error of commission d) Both a & c
107) Capital expenditure relate to:
a) Wages of construction b) Repair of machinery
c) Payment of wages to Mr. X d) Both a & b
108) In Partnership aim of joint life policy is:
a) Make profitb) Sufficient cash to settle claim of deceased partners
c) Strength financial position d) None
a) 3 years from the date of pledge b) 3 years from the date of documents
c) 5 years from the date of pledge d) None of these - no limitation
90)Garnishee Order attachesdebtsdue or accruingdue.Thismeans:
a) Loanspayablebythe customer to Bank.
b) Deposits payable on demand and payable at fixed futuredate.
c) Credit balances in cash credit account. d) None of these
109)Revenueexpenditureare:
a) Day to day expenditure b) Monthly
c) Yearly expenditure d) Half yearly
110)Drawingsdo not include:
a) Payment of Rent for properitor residence
b) Goodstaken byproperitor
c) Amountwithdrawnfrombank
d) Amount withdrawnfrom bankfor domesticuse.
111)Nominal Capitalis:
a) Part of authorised capitalissued
b) Maximum amount of share capital which companyisauthorizedto issue
c) Amount of capital actually applied for
d) Amountof capitalactuallypaid
112)Profit &lossa/c shows:
a) Nominal a/c b) Real a/c c) Personal a/c d) Both a & b
113) Baddebtsare_______for business:
a) Loss b) Revenue c) Expensed) Expenditure
114)Value of asset can become zero in _____ method:
a) Straight line b) Written down c) Sinking fund d) Insurance policy
115)Depreciation iscalculated on _____of asset.
a) Cost price b) Market price c) Book value d) Invoice price
116)The amount realised from sale of obsolete asset is:
a) Scrap value b) Residual value c) Both a & b d) Neither a nor b
117) If last installment is not paid, seller can___ the goods:
a) Take away b) Not retainc) Sale d) Return
118) Interest suspense a/cisshown in:
a) Trading a/c b) Balance sheet c) Profit & loan a/c d) Venture a/c
119)Accountsalestatement shows:
a) Gross sales b) Expenses incurred c) Bills acceptedd) All above
120) Relationshipof Consignor and Consignee is:
a) Principal /agent b) Partnerc) Share holder d) Members
121) Parties involved in JointVenture are called:
a) Co-venturesb) Co-ownersc) Share holder d) Both a & b
122)Networthofbusinessmeans:
a) Equity capital b) Total asset c) Fixed asset
d) Total asset – total liability
Compiled by Mr. Sanjay Kumar Trivedy, Sr. Mgr., RSTC, mumbai
28
123)What isTangible Net Worthof a Firm?
a) Capital only b) Bank OD
c) Capital+ Reserves &Surplus– Intangibles
d) Debtors
124) In a Banks balance sheet bad debts and provisions for doubtful debts are shown under:
a) Loans and Advances b) Provisions
c) Reserves d) Contingent Liabilities
125) Capital expenditure relate to:
a) Wages of construction b) Repair of machinery
c) Payment of wages to Mr. X d) Both a & b
126) ______ is an example of contingent liability:
a) Bank Guarantee b) Deposits
c) Loans and Advances d) Provisions
127)Nominal Capitalis:
a) Part of authorised capitalissued
b) Maximum amount of share capital which companyisauthorizedto issue
c) Amount of capital actually applied for
d) Amountof capitalactuallypaid
128)Profit &lossa/c shows:
a) Nominal a/cb) Real a/c c) Personal a/cd) Both a & b
129) Baddebtsare_______for business:
a) Loss b) Revenue c) Expense d) Expenditure
130) Bonussharesare issuedto _______:
a) Existing shareholders. b) New shareholders
c) Holders of partly paid-up shares d) None of the above
131)Accountsalestatement shows:
a) Gross sales b) Expenses incurred
c) Bills accepted d) All above
132) RelationshipofConsignor andConsigneeis:
a)Principal/agent b)Partner c)Shareholder d)Members
133) Partiesinvolved in Joint Venture are called:
a) Co-ventures b) Co-owners c) Share holder d) Both a & b
134) Networthofbusinessmeans:
a) Equity capital b) Total asset c) Fixed asset
d) Total asset – total liability
135)The balanceofPetty cash a/cisshown as:
a) Expense b) Asset c) Loss d) Revenue
136)IfBRSispreparedwithbalanceofcashbook,itwillshow_____perpassbook.
a) Balanceb) Overdraftc) Both a & bd) Either a or b
137)Who istheDebtor?
a) A person from whom thefirm hasmade credit purchases.
b) A person to whom goodshave been sold on credit
c) The owner d) Bank
138) Interest allowed on Capital of a partner is calculated on:
a) Opening capital b) Capital at end
c) Average capital d) Capital less drawing
139) Goodwill means __:
a) Value of reputation of the firm b) Good wishes of the partners
c) Good wishes of the customersd) Type of will made with good intentions.
140) Debit side of Partner capital a/c consist of:
a) Drawing b) Interest on drawing c) Loss transferred d) Above All
141) If ha bill made onh25th May matures afterh4 months,then maturity date will be:
a) 26t Sept.b) 25t Sept. c) 28t Sept. d) 1st Sept.
142)Double entry system isbased on:
a) Somebody loss = to some body gain b) Every debit equal to every credit
c) What comes in is = to what goes out d) All of the above
143) Capital at end is Rs 21,500, drawings are Rs 4,000 and profits given to partner is equal to Rs 6,000 then opening capital is equal to:
a) 14,000 b) 19,500 c) 20,000 d) 14,500
144) Balance sheet containsonlyPersonaland Realaccounts:
a) True b) False c) None of above d) a and b
145) Goods lost by fire should be credited to:
a) Purchase a/c b) Goods lost by fire a/c
c) Sales a/c d) No entry
89 D 90 B 91 A 92 D 93 A 94 D 95 D 96 A
97 D 98 B 99 A 100 B 101 A 102 B 103 A 104 B
10 B 106 D 107 A 108 B 109 A 110 C 111 B 112 A
5 A 114 A 115 C 116 C 117 A 118 B 119 D 120 A
11
3 122 D 123 C 124 B 125 A 126 A 127 B 128 A
121D
12 A 130 A 131 D 132 A 133 D 134 D 135 B 136 D
9 B 138 A 139 A 140 D 141 C 142 D 143 B 144 A
13
7 A Great dreams of great dreamers are always
14
5 transcended.

PRACTICE TEST NO. : 1


01 Which of the following accounting standards do not match in the context of banking companies' accounts: a)AS-21 : consolidated
statements b)AS-17 : Segment reporting c)AS-18 : Related party disclosure d)AS-15 : payment of compensation to employees
02 In the income and expenditure account, transactions relating to are included: a)current year only b)previous year only c)
subsequent year only d) current, previous and subsequent years
03 When the drawee pays the amount of the bill before its due date it is called: a)endorsing the bill b)honouring the bill c)retirement of the bill
d)none of the above
04 When a bill is paid before due date and the drawer gives some allowance to the drawee, this is called: a)discount on the bill B) retirement of
the bill at a discount
c)rebate d)any of the above
05 The books of a firm are closed but it is found that a purchase of Rs.1000 from Atma Ram Sons has been passed through the sale book for
Rs.100. What is the rectification entry, after preparing the trial balance . A)sales account debit Rs.900 to Atma Ram Sons b)purchase account
debit Rs.900 to Atma Ram Sons
c)sales account debit Rs.100, purchase account debit Rs.1000 to Atma Ram Sons Rs.1100 d)sales account debit Rs.1000, purchase account
debit Rs.100 to Atma Ram Sons Rs.1100
06 For which of the following, an adjustment entry will be required: a)expenses paid but not due b) expenses paid which were due
c)income received which was due
d) all the above
07 Which of the following statement is correct regarding single entry system: a)it is a system in which only one entry is made b)it is a system
where the principles of double entry are not being followed c) it is a system that takes into account only cash transactions d)none of the above
08 A machinery is purchased for a total price of Rs.1 lacndwith expected useful life of 5 years. What is the amount of depreciation at double
declining balance method for calculation of depreciation, in the 2 year: a)Rs.20000 b ) R s . 2 4 0 0 0 c ) r s.30000 d)Rs.40000
(Hint-Rate of depreciation at double rate is 40%. Depreciation is calculated on the balance value of the assets in the beginning of 2nd year i.e. on

Compiled by Mr. Sanjay Kumar Trivedy, Sr. Mgr., RSTC, mumbai


29
Rs.60000)
09 A car is purchased for Rs.310000 with a scrap value of Rs.60000 at the end of 5 years' useful life during Sept 2008. 1/hat is the written down
value as on Mar 31, 2009 (which is closing date of financial year). A)310000 b)260000 c) 250000 d) 200000
Problem: Ramesh and Dinesh are two partners in the profit and loss sharing ratio of 3:2. Their capital is Rs.120000 and Rs.80000. Interest on capital
is 5% p.a. During the year, the withdrawal was Rs.30000 and Rs.20000 and interest allowed was Rs.1600 and Rs.1000 respectively. Profit before
interest was Rs.40000 and Mr. Dinesh is entitled to salary of Rs.12000 p.a. On the basis of this information.
10 If the accounts are fixed capital account, what is the closing balance of capital account of these partners. A)Rs.133240 and Rs.88240 b)
Rs.120000 and Rs.80000 c)Rs.100000 and Rs.60000 d) Rs.90000 and Rs.50000
11 If the accounts' are fixed capital account, what is the closing balance of current accounts of two:a)13000 and 3000 b)13100 and 3100 c)13240
and 3240
d)13540 and 3540
12 If a two-sided error has taken place, it will be corrected: a)debiting the suspense account b)by correcting the positing c) by passing a journal entry
d)none of the above
13 After preparation of trial balance it is observed that the goods amounting to Rs.10000 sold to Ashok & Sons were posted as Rs.1000 to
their account. The rectification entry will be:a)suspense account debit to Ashok & Sons Rs.9000 b) Ashok & sons debit to sales account Rs.9000 c)
sales account debit to suspense account Rs.10000 d)Ashok & sons debit to suspense account Rs.9000
14 When a company issues shares and the calls amount in instalments i.e. application money, allotment money, call money, the amount of
single call :
a should not be less than 25% of the face value b)should not be less than 20% of the face value c)should not be more than 25% of the face value
d should not be more than 20% of the face value
15 The shares of Rs.100 each, are issued at 10% discount and are forfeited for non payment of call money of Rs.20 each. Which of the
following journal entry will not be correct: a)forfeited shares account-credit Rs.60 b) discount on issue of shares — cr Rs.10 c)share capital
account debit Rs.100 d)share 1st and final call — debit Rs.30
16 What adjustment entry will be passed for the goods taken by the promoter out of the goods purchased for the business:
a purchase account debit to credit the drawing account b)capital account debit to credit the purchase account
c drawing account debit to credit the purchase a/c d) purchase account debit to credit the capital account
17 Forex rate in Delhi is 1 US $ = 48.80/90. In London the 1 Euro = US $ 1.60/65 pound sterling. What is the cross rate for Euro. a) 78.08 b)77.92
c)77.65 d) 77.02
18 Spot rate is 1 US $ = 48.10 and 2 months forward is available at 1US = 48.50. a)Forward is at a premium b)Forward is at a discount c)Spot is at a
premium
d Spot is at a discount
19 Which of the following does not match: a)purchase of machinery for sale — revenue expenditure b)advertisement expenses — deferred revenue
expenses
c)payment of wages to labourer — revenue expenditure d)purchase of goods for trade — capital expenditure
20 If there is overdraft with the bank, the bank column of the cash book will show: a)a credit balance b)a debit balance c)either a debit balance or a
credit balance
d neither a debit balance or a credit balance
21 The trial balance in which the total of debit side and total of credit side of each ledger account is taken into account is called:a)main trial balance
b)net trial balance
c trial balance d)gross trial balance
22 The pricing method under which the price is determined by subtracting an appropriate gross mark up from the sale price, to an unrelated
3rd party, with the appropriate gross margin being determined by examining the conditions, under which the goods are sold and comparing the
said transaction to other 3rd party transaction, is called:a) comparable uncontrolled price method b) cost plus method c)re-sale price method
d)non-traditional method
23 A firm has been following the practice of not charging annual depreciation. At the time of sale of assets, it shows the difference between sale
price and original cost as gain or loss in the year of sale. Which accounting concept is being violated. A)consistency concept b)conservatism concept
c)money value concept d)periodicity concept
24 Which of the following is correct formula for calculation of future value of an annuity due (beginning of period).a)C x {(1+r -1/r) b)C x {(1+r )11 -
1/r} x (1+r) 1
c) C / {(1-11 ) ' -1/r} x (1+r) d) C x {(1+r -1/r) / (1+r)
25 X is to receive Rs.5000 at beginning of every year for 5 years at 5%. How much amount he is required to invest now (Hint : calculate the
present value of annuity due).a) 23630.50 b)22729.60 c)21942.15 d)21308.75
26 When a firm makes payment to another firm by way of a cheque, the bank debits the- same, after a time lag when cheque is presented. In the
meantime, if cash book balance is to be reconciled with the pass-book balance, in the balance of cash book, the amount of cheque:
a is added if the firm has a current account b) is deducted if the firm has a current account c) is added if the firm has a overdraft account d)is
added in all circumstances
27 Balance in cash book of a firm is Rs.9500. It is noticed that (a) thA payment side of the cash book is undercast by Rs.200 (b) a cheque of Rs.5000
issued by the firm is not entered in the bank column of the cash book but bank has paid it. The balance in the pass-book should be: a ) 9 5 0 0
b)9300c) 4300d) 4500
28 A firm purchased new machinery for Rs.3.20 lac. It is to charge depreciation @ 6%, 5% and 4% for 1st, 2nd and 3rd year respectively. At the end of 3rd
year, its written down value will be: a ) 2 6 9 5 0 0 b ) 2 7 1 9 4 0 c ) 2 7 4 4 0 0 d ) 2 7 5 9 0 0
29For computing operational risk under Basel II, which are the three approaches, suggested by Basel Committee:
A basic indicator approach, standard rating based approach, standard advance approach
b basic indicator approach, Internal rating based-foundation approach, internal risk based — advance approach
c basic indicator approach, Internal rating based- approach, advance measurement approach
d basic indicator approach, standard approach, advance measurement approach
30 RBI has decided to implement the revised CRAR Basel II guidelines with the following time frame. Which of these does not match:
a Foreign banks having branches in India — March 31, 2008 b ) Indian banks with operations abroad — March 31, 2008
c Other Indian banks excluding RRBs / LABs— March 31, 2009 d) RRBs / Local Area Banks — 31.3.2010
31 Tier I capital fund includes (a) paid up capital (b) statutory and capital reserves (c) Innovative Perpetual debt instruments (d) Noncumulative
perpetual preference shares. A)a to d is correct b) b to d is correct c) a, c and d is correct d)a, b and d is correct
32 The depreciation has been charged and it has been debited to fixed asset account. It is an: a)error of omission b) error of commission c)
error of principle
d compensating error
33 A bond has face value of Rs.100 and coupon of 10%. Its remaining maturity period is 6 years. At 11% YTM, its market value is: a)96.10 b)95.95
c)95.80 d)94.90
34 The liabilities that are repayable only when business will be terminated are called: a)fixed liabilities b)long term liabilities c) contingent
liabilities
d) current liabilities
35 For a given difference between YTM and coupon rate of a bond: a ) shorter the maturity, greater will be change in price with change in
YTM
b)longer the maturity, greater will be change in price with change in YTM c)longer the maturity, shorter will be change in price with change in
YTM d)no change
36 A bond with a face value of Rs.100 has a maturity period of 5 years. If the YTM decreases to 9%, what will be the change in the price: a)
increase 3.9
b) decrease 3.9% c) increase 2.7% d) decrease 2.7%
Problem: Bond-A with a face value of Rs.100 has a coupon of 12% with 6 years' maturity. At YTM of 10%, its market value is Rs.108.70. Bond-B with a
face value of Rs.100, coupon of 12% with a 6 years' maturity carries value of Rs.73.40 for YTM of 20%.
37 If YTM increases by 20%, the market price of Bond-A will change to:a)110 b)106 c)100 d)98.40
(Hint: 12 PVIFA (12%,6) + 100 PVIF (12%, 6)
38 The drawing made by the partners falls under which of the following category of accounts: a)real account b)impersonal account c) nominal
account d) personal account
39 On a bond, the investor is subject to interest rate risk on account of
(a) re-investment of annual interest (b) capital loss on account of change in the value of bond due to increase in market interest rates.
Compiled by Mr. Sanjay Kumar Trivedy, Sr. Mgr., RSTC, mumbai
30
a only (a) b)only (b) c)a and b both d)none of these
40 If duration of a bond is 5 years, which of the following statement would be true: a)its payable after 5 years b) interest rate risk will disappear on
the bond, if the holding of bond is for 5 years c) no change in value of bond will take place within 5 years d)change in price of bond within 5 years
will not affect the investor.
41 Which of the following expenses by a firm cannot be taken as part of capital expenditure: a)land & building, b)plant and machinery c)technical know-
how d)none of the above
42 The appraisal method for capital expenditure, under which it is examined, how much period the invested funds will come back as cash
inflow is called:
a)pay-back method b) rate of return method c) net present value method c) internal rate of return
43 A firm has taken term loan from the bank for construction of building. It will: a)increase the asset and increase the liability b) increase the asset
and decrease the liability c)increase one asset and decrease another asset d) decrease one asset and decrease one liability
44: The process under which the future value of present cash flows is determined is called: a) discounting b)compounding c)net present value
d)internal rate of return
45 Which of the following is an error of omission. A)wages account debited although services of labour /used for building construction
b goods purchased for cash from XYZ and their account credited c)sale of goods made to ABC, but not entered . d)an entry of Rs.927 posted as
Rs.972.
46: Which of the• following is not a source of finance for capital expenditure project: a)bank cash credit and trade creditors b)bank term loan and
subsidy from govt.
a: institutional term loan and promoters' equity d) long term unsecured loans and debentures
Problem: A company invested Rs.5 lac in a project in the year 2000. The project earns the profits as under:
Project-A Project-B Discount Factor @
Investment 5,00,000 5,00,000 5%
ls`dyear profit Nil 30000 0.95238
2"(6 year 60000 50000 0.90703
3profit
th
year 80000 160000 0.86384
4profityear 120000' 170000 0.82270
5thth year
profit 120000 110000 0.78353
6profit
th
year 110000 20000 0.74622
7profit
_year 90000 Nil 0.71068
profit
_Total 580000 540000
profit
47 Based on the above inforrhation, what is the payback period of Project- A) 5 years b)5 years and 2.5 months c)6 years and 1.33
months d)6 years and 8.12 months
48 Based on the payback period, what is the payback period of Project-B- a)4 years and 7.63 months b)4 years and 9.12 months c)5
years and 2 months
d)5 years and 4.33 month
49 Based on the above information, which project is preferable for investment and why: a)Project A with higher payback period b)
Project A with lower payback period
c Project B with higher payback period d)Project B with lower payback period
50 In the above case, what is the accounting rate of return of Project A: a ) 2 8 % b ) 3 1 % c ) 3 3 % d ) 3 5 %
51 In the above case, what is the accounting rate of return of Project B: a ) 2 8 % b ) 3 1 % c ) 3 3 % d ) 3 5 %
52 Based on the accounting rate of return, which project is preferable for investment and why: a)Project A with higher accounting rate of
return b)Project A with lower accounting rate of return c) Project B with higher accounting rate of return d) Project B with lower
accounting rate of return
53 X wants Rs.340000 to replace his car after 5 years. He wants to save and invest in equal monthly instalments at 12% p.a. How much
money will be deposited every month. A) 4136.80 b)4163.20 c)4182.20 d) 4203.40
54 X has been saving Rs.2000 at 10%p.a. compounded annually. What will be amount at the end of 3rd year. A ) 6 5 4 0 b)6620 c)6690 d) 6710
55 The depreciation is calculated as: a) original cost / percentage rate of depreciation b)original cost / no. of years of useful life c)original
cost / total life of the asset
d invoice price / total life of the asset
56 If scrap value of the machinery is given, the amount of depreciation is worked out as: a) original cost / percentage rate of depreciation
b)original cost / no. of years of useful life c)original cost — scrap value / no. of years of useful life d) invoice price — scrap value / total life
of the asset
57 The scrap value of a machinery with original costopf. Rs.1 lac is estimated at Rs.10000. Its useful life is 6 years. The depreciated value at
the end of 4th year is:
a 30000 b) 35000c) 40000d) 45000
58 A person wants to remit Euro and there is no quotation with the bank for Euro. Bank works out the rate through Re/$ rate and $/Euro rate.
This is called:
a bid rate b) offer rate c)cross rate d) floating rate
59 On a particular amount, the simple interest is Rs.306 for 2 years and the compound interest is 450 for 3 years. What is the principal
amount.a ) 3 7 5 0 b ) 3 7 7 5
c)3820 d ) 3 8 6 0
60 X had borrowed Rs.65600 at 5% interest for 2 years to be payable in 2 annual instalment. What is the amount of annual instalment.a )
3 3 4 5 0 b ) 3 3 9 8 0 c)34890
d)35280
54 X has been saving Rs.2000 at 10%p.a. compounded annually. What will be amount at the end of 3rd year.a ) 6 5 4 0 b)6620 c)6690 d) 6710
55 The depreciation is calculated as: a) original cost / percentage rate of depreciation b)original cost / no. of years of useful life c)original
cost / total life of the asset
d invoice price / total life of the asset
56 If scrap value of the machinery is given, the amount of depreciation is worked out as: a)original cost / percentage rate of depreciation
b)original cost / no. of years of useful life c) original cost — scrap value / no. of years of useful life d) invoice price — scrap value / total life of
the asset
57 The scrap value of a machinery with original costopf. Rs.1 lac is estimated at Rs.10000. Its useful life is 6 years. The depreciated value at
the end of 4th year is:
a 30000b)35000 c)40000 d) 45000
58 A person wants to remit Euro and there is no quotation with the bank for Euro. Bank works out the rate through Re/$ rate and $/Euro rate.
This is called:
a bid rate b) offer rate c) cross rate d) floating rate
59 On a particular amount, the simple interest is Rs.306 for 2 years and the compound interest is 450 for 3 years. What is the principal
amount.
a 3 7 5 0 b ) 3 7 7 5 c)3820 d ) 3 8 6 0
60 X had borrowed Rs.65600 at 5% interest for 2 years to be payable in 2 annual instalment. What is the amount of annual instalment.a )
3 3 4 5 0 b ) 3 3 9 8 0 c ) 34890
d)35280
61 Spot exchange rate is 1 Euro = $ 1.40 and swap rate (called forward points) is 0.0105. Calculate 90 days interest differential assuming 360 days
in a year.
a 4 % b ) 3 . 5 0 % c) 3.25% d)3%
62 Accounting system which is associated with the need of the business owners to keep record of their transactions, property etc, dues they owe
and debts others owe them, is : a)financial accounting b)steward accounting c)cost accounting d) management accounting
63 Accounting system which is used ascertaining the costs with a view to control them and also make assessment of profitability and efficiency
of the business, is :
a human resources accounting b)social responsibility accounting c) cost accounting d)management accounting
64 As per accounting standard No.22, the aggregate of current tax and deferred tax charged or credited to the statement of profit and loss for the
period, is called: a)taxable income or loss b) tax expenses c) deferred tax d) accounting income
65 A cheque issued by a firm is recorded on side of the :a)payment, cash book b) receipt, cash book c)payment, journal d)
receipt, journal
66 The price at which two unrelated and non-desperate parties would agree to a transactions is called: a)cost pricing b)transfer pricing
Compiled by Mr. Sanjay Kumar Trivedy, Sr. Mgr., RSTC, mumbai
31
c)opportunity pricing
D )arm's length pricing
67 Firm-A sells goods on credit as well cash. For goods worth Rs.50000 the payment is yet to be received although goods were supplied,
two months earlier to closing of the financial year. A)if it is not accounted for in the books it will have no effect and no accounting practice will
be violated.
B )if it is accounted for, the profit will be increased without any reason to do so. C)if it is accounted for, the profit position will be true and fair.
d) No accounting practice will be violated by not doing so.
68 A firm sells goods worth Rs.50000 to M/s XYZ in cash. In this case, which two accounts are involved: a)cash account and XYZ
account b)XYZ account and goods account c)goods account and cash account d)incomplete information
69 A firm sells goods worth Rs.50000 to M/s XYZ in cash. In this case, while crediting the account, the following principle will be
followed:
a credit the giver b)credit the receiver c)credit what comes in d)credit what goes out
70 A persons owes some amount to the firm, is called -----of the firm: a)debtor b)creditor c) customer d)lender
71 Balance as per pass-book is Rs.23000 in the overdraft account as on Sep 30, 2008. It is observed that (a) a cheque of Rs.3000 deposited
by the firm has been received by the bank as dishonoured. (b) bank has debited Rs.50 as cheque returning charges (c) bank has dishonoured a
cheque of Rs.1000 by mistake, which the firm had issued to make payment. The balance as per cash book should be: a ) 2 0 9 5 0 b)21000
c)24050 d) 25050
72 Which of the following statement is not correct in the context of bank reconciliation statement: a)cheque issued by a firm is first
recorded in the cash book
b cheque deposited by a firm is first recorded in the pass-book c) direct deposit by a customer of a firm, in bank account, is credited by the
bank
d)direct debit by the bank is later on credited by the firm to bank account in its books.
73 A company had issued share with face value of Rs.100 + premium of Rs.200. On certain shares, the application money and allotment money of
Rs.40 each has been received but due to non-receipt of 1st call money, few shares have been forfeited. Which of the following statement is correct: a)
debit to share capital Rs.100 per share to be made b)debit to share capital Rs.300 per share to be made c)debit to share capital Rs.80 per share
to be made d)debit to share application at Rs.100 to be made
74 The shareholders of a company have a fixed liability to pay, in case of liquidation of the company. It is called: a)company limited by shares
b)company limited by guarantee c)company with limited liability d) company with unlimited liability
75 When goodwill is not brought in cash by the new partner, it is raised and immediately written off, what journal entry is passel] to write it
off:
a debit goodwill account, credit old partners' account b)debit old partners' account, credit goodwill c)debit all partners' account, credit goodwill
d)debit old partners' account and credit
76 For which of the following, an adjustment entry will not be required: a)salary of Rs.3000, paid in advance b)commission due Rs.1000, but not
received
c commission payable Rs.300 during the year, paid d)rent due Rs.2000 but not paid.
77 All the material information relating to business transactions is required to be given by a company in its balance, due to which of the
following:
a accounting period concept b)full disclosure concept c)materiality concept d)all the above
78 If commission is received in advance and adjustment entry is passed, which of the following will not be true:
a it will reduce the commission income b)it will reduce the profit c)it will reduce the net worth d) it will be shown on the asset side of the balance
sheet at income
received in advance.
79 Which among the following accounting standards deal with inventory valuation: a) AS – 2 b)AS – 4 c)AS – 9 d)AS - 11
80 Under this method, the stock at the close is the stock received as earliest consignment: a)first in first out method b) last in first out method c)base
stock method
d average cost method
Problem: A firm purchased 400 kgs material @ Rs.10 per kg, on April 4 and 600 kgs @ Rs.11 per kg, on Apr 15. During the month of April it used
500 kgs of material and closing stock is 500 kgs at the end of April.
81 In the above problem, if first in first out method is used, the stock in hand would represent: a)400 kg of earlier and 100 kg of later b)100 kg of
earlier and 400 kg of later c) 500 kg of earlier material with deficit of 100 kg d) 500 kg of later material
82 In the above problem, if first in first out method is used, the value of stock in hand would be: a ) R s . 5 5 0 0 b) Rs.5100 c)Rs.5250 d)Rs.5240
83 The net realizable value is the estimated selling price in the ordinary course of business less --------- cost. A ) m a r k e t b)selling
c)completion d)average
84 When loss of stock takes place due to theft or fire etc. it is called: a)total loss b)normal loss c) contingent loss d)abnormal loss
85 Which is the correct classification of a consignee's account, out of the following: a)personal account b) real account c)nominal account d)none
of the above
86 All receipts and payment made in cash, including cheques are recorded in: a ) balance sheet b)income and expenditure account c) receipt
and payment account
d profit and loss account
87 Under average profit method, the goodwill is calculated as under: a)average profit / no. of years b)average profit x no. of years c)average profit x
agreed multiplier factor d)average profit x average capital of existing partners
88 A firm has a capital of Rs.2 lac and its profits are Rs.52000. The normal rate of return in the industry is 10%. What is the amount goodwill as
per capitalization of profits method: a ) 3 2 0 0 0 0 b ) 3 0 0 0 0 0 c ) 2 6 0 0 0 0 d ) 2 3 0 0 0 0
89 If the proprietor has made certain drawings, the profit will be calculated as under: a)amount of drawings will be added back to the capital to
find out the profit
b) amount of drawings will be reduced from the capital to find out the amount of profit c)the amount of drawings has already been accounted for and cash
reduced to that extent
d )none of the above
90 Accounts and auditors' report of a banking company is required to be published (a) in a newspaper circulating in the place where the banking
company is having its principal office (b) it is to be published within 6 months of close of its financial year (c) this is requirement of Rule 15 of Banking
Regulation (Companies) Rules.
a a to c all correct b)only a and c correct c) only b and c correct d) only a and b correct
91 Under a fixed capital account, all the adjustments like interest on capital or drawings, share in profit or loss are recorded in the: a)partner's capital
account
b partner's current account c)partner's floating account d) none of the above
92 A, B and C are three partners with sharing ratio of 5:3:2. A retires and the new ratio of remaining partners is 5:3. What is gain of B. a)17/40 b)13/40
c)9/40 d)7/40
93 A person has lent his name to a firm without actually joining the firm as partner. He is called: a)sleeping partner b)dormant partner c)quasi or
nominal partner
d)accredited partner.
94 A company comes out with an equity share issue having face value of Rs.10 and charges total Rs.30 for the share. The amount will be
credited to:
a entire amount to the share capital account b)entire amount to the share premium account c) Rs.10 to share capital and Rs.20 to share
premium a/c
d Rs.20 to share capital and Rs.10 to share premium a/c
95 A company forfeits 2000 shares of Rs.10 each due to non-payment of 2nd call @ Rs.2. The amount already received is Rs.3 on application, Rs.3 as
allotment, Rs.2 as first call. How much amount will be debited to share capital account:
a ) R s . 2 0 0 0 b)Rs.8000 c)Rs.10000 d)none of the above
96 Which of the following provisions of Companies Act are not correct: a)Section 209 - companies to maintain certain books of account b)Section 210
— preparation of final accounts c)Section 211 — balance sheet should exhibit true and fair view of state of affair of the company d)Part II — Schedule
VI prescribes the format for preparation of profit and loss account.
97 At the end of one year, a sum of Rs.10000 at 6% rate of interest when compounded half yearly becomes:a)10600 b ) 1 0 6 0 9 c) 10613 d)
10616
98: For a capital budgeting expenditure, the net present value of a project at 18% is Rs.15 lac negative, at 10%, it is Rs.12 lac positive and at
Compiled by Mr. Sanjay Kumar Trivedy, Sr. Mgr., RSTC, mumbai
32
14%, it is ZERO. The internal rate of return from the project is: a)18%, as there is deficit b)14%, as it is Zero c) 10% as there is surplus d) none of the
above
99 When a new partner is included in a partnership it will involve (1) change in sharing ratios (2) accounting of goodwill (3) revaluation of assets or
liabilities.
a 1 and 2 only b)2 and 3 only c)1 and 3 only d)1 to 3 all
100 X has been investing Rs.5000 every year at year end, at 5% for 5 years. How much he will get at the end of 5 years (hint calculate the
future value of an ordinary annuity): a) 26778.20 b )27278.90 c) 27628.15 d) 28342 .2 5
Answers - 1
01 d 02 a 03 c 04 c 05 c 06 a
07 b 08 b 09 b 10 b 11 c 12 c
13 d 14 c 15 d 16 c 17 a 18 a
19 d 20 a 21 d 22 c 23 d 24 b
25 b 26 a 27 c 28 c 29 d 30 d
31 a 32 c 33 c 34 a 35 b 36 a
37 c 38 d 39 c 40 b 41 d 42 a
43 a 44 b 45 c 46 a 47 c 48 a
49 d 50 c 51 b 52 a 53 b 54 b
55 b 56 c 57 c 58 c 59 a 60 d
61 d 62 b 63 c 64 b 65 a 66 d
67 c 68 c 69 d 70 a 71 a 72 b
73 a 74 b 75 c 76 c 77 b 78 d
79 a 80 b 81 d 82 a 83 c 84 d
85 a 86 c 87 c 88 a 89 a 90 a
91 b 92 b 93 c 94 c 95 c 96 d
97 b 98 b 99 d 100 c

PRACTICE TEST NO. : 2

01 An entry for a bill of exchange will be required in the books of drawee on happening of which of the following: a)discount of bill by the drawer
b)acceptance of the bill by drawee c)submitting bill for collection to bank by the drawer d)endorsement of the bill by drawer to another party
02: A firm has four projects as alternative projects and wants to take up one. Based on the payback method, it should take up which of the
following:
a) project with 2-yr payback period b) project with 3-yr payback period c)project with 4-yr payback period d ) project with 5-yr payback period
03 Due to fire, the amount of stock that was destroyed was Rs.3000. The insurance company paid claim of Rs.2450. The difference amount shall
be:
a debited to profit and loss account b)debited to stock account c)debited to trading account d) debited to bad account
04 Goods lost in fire will be shown in (1) debit side of trading account (2) credit side of trading account (3) debit side of profit and loss account
(4) credit side of profit and loss account a)1 and 3 b)2 and 3 c) 1 and 4 d) 2 and 4
05 If the outstanding salaries do not appear in the trial balance but information is provided as additional information, while preparing the final
account, it will be reported as: a)liability in the balance sheet and debit side of P & L account b) liability in the balance sheet and credit side of P & L
account c) asset in the balance sheet and debit side of P & L account d) asset in the balance sheet and credit side of P & L account
06 What will be correct order, if the following liabilities are to be shown on the basis orpermanence (1) term loan (2) capital (3) current liabilities
(4) Reserves
a 3,1,4,2 b ) 2,3,1,4 c) 2,4,1,3 d) 2,4,3,1
07 Which among the following transaction will increase the assets and increase the liability: a) drawing by promoter b) credit purchase c)
purchase of machinery by using cash d) repayment of term loan

08 X and Y decide to admit Z as new partner with 114th share. The new shares of X and Y will be equal, while their existing sharing ratio was
3:2. What is the sacrifice ratio of X and Y: a) 3:1 b) 5:1 c)7:1 d) 9:1
09 A partnership firm earns annual profits of Rs.1.20 lac while the average industry return is 10%. The firm has assets base of Rs.14.40 lac
and liabilities of Rs.4.80 lac. On the bas of capitalization method, the goodwill of this firm shall be: a)R s . 9 6 0 0 0 0 b) Rs.480000 c)Rs.240000
d)Rs.120000
10 Which of the following is not an intangible asset in the balance sheet of a firm: a) pre-paid expenses b)preliminary expenses c) pre-
operative expenses
d) accumulated losses
11 Which of the following better explains the term marshalling of the balance sheet: a) audit of the balance sheet by qualified accountants b)
scrutiny of the balance sheet by the shareholders of a company c)grouping and presentation of assets and liabilities d) mixing of assets and
liabilities.
12 A bond has face value of Rs.100 and coupon of 10%. Its remaining maturity period is 6 years. At 11% YTM, i.e. !% change in YTM. What will be
change in its price.
a 4 . 2 0 % b ) 4 . 3 0 % c ) 4 . 5 0 % d)4.75
13)The face value of the bond is Rs.1000 with a coupon of 8% with a maturity of 3years. The required rate of return is 9%. What is the value of the
bond:
a) 974.68 b) 968.74 c) 961.45 d) 952.67
14: Rate of return at which the net present value is ZERO or NIL, is called: a)pay-back method b) rate of return method c) net present value method
d) internal rate of return
15 What will be amount of owners' equity, if the opening balance of capital is Rs.30000, the creditors Rs.34000, income Rs.35000 and
expenses Rs.32500.
a) 65000 b ) 3 2 5 0 0 c ) - 2 5 0 0 d ) 5 6 5 0 0
16 A firm employed a peon at a monthly salary of Rs.5000 in the month of October. The firm closes its accounts as on Mar 31 each year. The salary
expenses for the peon as recorded in the salary account are Rs.25000. What adjustment entry will be required at the time of closing of books:
a no adjustment is required b) provision for outstanding wages for Rs.5000 will be required. C)provision for outstanding salary for Rs.5000 will be
needed
d provision for outstanding wages for Rs.35000 will be needed
17 A profit of 20% on sale price of a commodity amounts to: a)20% of the cost price b)25% of the cost price c)33.3% of the cost price d)none of the
above
18 Firm is to pay Rs.24000 on account of goods purchased to XYZ out of which a payment of Rs.21000 is made. This will be journalized as
under:
a debit cash and Credit XYZ b)debit XYZ and credit purchases c)debit purchases and credit XYZ d)debit XYZ and credit cash
19) For determination of net present value, the risk adjust discount Rate approach' makes a balance between: a)absolute risk and absolute return b)rate
of return and degree of risk c) risk and profit d) average risk and average return
20 Current yield on a bond can be calculated as: a) amount of coupon interest / market price of the bond b)amount of coupon interest / face value of
the bond
C )amount of coupon interest / maturity value of the bond d) coupon rate of the bond / market price of the bond
21 Mr. Dixit obtained a loan of Rs.200000 which is repayable in 10 years at 15% p.a. interest rate to be compounded monthly. The monthly
instalment will be :
a 2 8 2 5 b ) 3 2 2 5 c) 3175 d ) 3 3 8 5
22 PV = FV (l+r)' This can be written as: a)PV = FV x (1+r) n b)PV = FV / (1+r)" c)FV = PV / (1+r) n d) any of the above
23 Mr. X obtained a personal loan of Rs.50000 from the bank at 16% simple rate of interest for 4 years. The total amount to be returned after 4
years would be:
Compiled by Mr. Sanjay Kumar Trivedy, Sr. Mgr., RSTC, mumbai
33
a 5 0 0 0 0 b ) 7 2 0 0 0 c) 82000 d) 18000
24 Group A: (a) receipt and payment account (b) surplus (c) legacies
(d) income and expenditure account. Group B: (i) amount received as per will of a deceased person (ii) summary of cash transactions for the year (iii)
excess of expenditure over income (iv) consists of items of income and expenditure during the year. Which of the items of Group A, do not match
with Group B:
a ii b)iii c)I d) iv
25 The termination of the contract is possible /not possible in hire purchase /instalment sale (which is correct) : a)possible in hire purchase b)not
possible in instalment sale c) not possible in hire purchase or instalment sale d) possible in instalment sale and not possible in hire purchase
26 During the period of rising prices, the----- (LIFO / FIFO) system results in reporting of false inventory----- (profits / losses): a)LIFO, profit
b)FIFO, profit
c LIFO, loss d) FIFO, loss
27 Future value / (1 +r) n = a)discounted value b) present value c)net present value d) net value
28 A sum of Rs.10000 is deposited with the bank by a customer at 6% rate of interest. The bank has various options (intervals) to compound the
interest. Which of the following is correct total amount: a) if the compounding is annual = 10600 b) if the compounding is semi-annual =
10609
c if the compounding is quarterly = 10613 d) if the compounding is monthly = 10618

29 The operating expenses in the profit and loss account of a bank include (a) establishment expenses (b) audit fees (c) rent and taxes (d)
commission paid.
a a to d all b) a, b and c only c) a, c and d only d) b, c and d only
30 In balance sheet of a bank the bills payable are shown under Schedule----- and deposits of branches outside India under Schedule: a)5-other
liabilities and provisions, 2-deposits b) 4-other liabilities and provisions, 2-deposits c) 5-borrowings, 2-deposits d) 5-other liabilities and provisions, 3-
borrowings
31 XYZ has liabilities of Rs.20000 and their cash is Rs.15000 and machinery Rs.25000. The amount of capital + reserves =
a 15000 b) 25000 c ) 4 0 0 0 0 d ) 2 0 0 0 0
32 A truck is purchased for Rs.6 lac on April 01, 2005 with expected useful life of 6 years and sold for Rs.3.25 lac on Mar 12, 2008. What was book
value on date of sale of the truck at straight line method and what is amount of profit or loss on this sale: a Rs.3 lac, profit Rs.25000 b)Rs.2 lac, loss
Rs.75000 c) Rs.3 lac, loss Rs.25000
d Rs.3 lac, no profit no loss
33 The hire purchase price consists of which of the elements (a) cash price (b) interest for the delayed payment (c) following servicing cost (d)
depreciation:
a a to d all b) a and b only c) b and d only d) a to c all
34 In a joint venture, the joint venture expenses are met out of which of the following accounts: a)joint bank account b)co-venturer's account
c)joint venture account
d any of the above
35 Match the terms (a) rebate (b) accommodation bill (c) drawer (d) payee with the terms out of (1) an imaginary transaction (2) creditor (3)
debtor (4) allowance for early payment (5) person entitled to get payment as per order of the drawer. Which of gie following is not correctly
matched:
a a matches 4 b) b matches 1 c) c matches 3 d) d matches 5
36 Govt. promoted a new town whose population has been increasing at the rate of 5% every year with the present population of 1 lac. What will be the
population after 5 years: a ) 1 2 6 7 2 8 b ) 12 8 7 2 6 c)127628 d)127862
37 Which of the following items, cannot appear in the profit and loss account: a) printing and stationery charges b) salary payment c) power and fuel
d)insurance payment
38 Loss incurred by a firm is transferred to which of the following account (which one is not correct) : a)to proprietor's capital account in case of
proprietorship account
b to profit and loss appropriation account in case of accounts of a company c) to partners capital account in case of partnership account d)none of
the above
39 For construction of his house a person raised a loan of Rs.25 lac from a finance company at 18% rate of interest and repayable over 8 years.
The amount of monthly instalment would be: a ) 4 9 3 0 8 b ) 4 8 5 6 4 c ) 4 9 8 0 4 d ) 4 8 9 6 8
40 In which of the following cases a project investment can be undertaken by an investor: a)the IRR is greater than capital cost, as it increases the
investor's
wealth b)the IRR is less than capital cost, as it increases the investor's wealth c) the IRR is greater than capital cost, as it decreases the investor's wealth
d the IRR is less than capital cost, as it decreases the investor's wealth
41 A van is purchased for Rs.2.40 lac. It is expected that after useful
life of 5 years it can be sold for Rs.0.40 lac. Its annual amount of depreciation in straight line method would be: a)Rs.48000 b)Rs.8000 c) Rs.40000
d)Rs.56000
42 Nominal account means (a) an account of each person (b) an account of each firm with whom the business firm as dealings (c) an account of
each head of expense or source of income (d) An account of each property or possession dealt in by the trader in his business a) a and c only b)b only
c) only d d)c only
43 For debit and credit certain rules are followed. Which of the following rules has not been appropriately applied: a)purchase of machinery
— debit what comes in b) cash received — debit the giver c)salary paid — debit all expenses d) goods purchased on credit from XYZ — debit what
comes in and credit the giver
43 The capital account of the promoter, the books of business is ------ (asset / liability) due to application of accounting concept
called _____a)asset, entity concept b)liability, money concept c)liability, entity concept d) asset, money concept
45 A company called Rs.3 with application, Rs.3 on allotment and Rs.4 as first call money. On 1000 shares, the first call money is not received.
Later on, the company decides to forfeit the share. What amount will be credited to forfeiture account: a) Rs.3000b)Rs.6000 c) Rs.7000 d) Rs.10000
46 A company called Rs.3 with application, Rs.3 on allotment and Rs.4 as first call money. On 1000 shares, the first call money is not received. Later on,
the company decides to forfeit the share. What amount will be credited to calls in arrear account: a)Rs.10 000 b) Rs.7000 c) Rs.4000
d)Rs.3000
47 Out of the following, find out the feature of the business entity concept: a) assets are recorded at their cost b)only those transactions are recorded
that can be expressed in money terms c) each transaction has two aspects in accounting d) business is treated separate from the owner
48 Income or expenditure are recorded when these become due and not when these are actually received or incurred. This is as per: a) consistency
concept
b)historic concept c) accrual concept d) going concern concept
49 The consistency concept is required to be followed in which of the following: a)charging of depreciation b) inventory valuation
c)classification of debtors
d)all the above
50 The partner of a firm extended a loan of Rs.1 lac to the firm for a period of 3 months. This will be:a) debited to capital account of the partner
b credited to capital account of the partner c) debited to loan account of the partner d) credited to loan account of the partner
51 Which of the following equation is not correct? A) capital = assets — outsiders' liabilities b) outsiders' liabilities = capital – assets c) assets = capital +
outsiders' liabilities
d none of these
52) Which of the following accounts will never show a credit balance: a )cash account b) bank account c) capital account d) provisions
53) The discount that is allowed to a customer by a firm as an incentive for making payment before due date is called: a)discount b)cash discount
c)trade discount d)any of the above
54) A Bonds Redemption value (Par value) is Rs. 1,000 bears a annual coupon rate of 12% and has a term to maturity of 3 years. The going market rate for
similar, new investments is 10%. What is the price of this bond in secondary markets? a) Rs.1049.44 b) Rs.1059.55 c) Rs.1409.44 d) Rs.999.55
55) What is true about the Duration of a Bond? A) Duration is expressed in terms of years. b) Duration of a coupon-paying bond is always less than its maturity. c)
In Zero-coupon bonds where periodical interests are not paid out, duration will be equal to its maturity d) All of the above
Compiled by Mr. Sanjay Kumar Trivedy, Sr. Mgr., RSTC, mumbai
34
56) The duration of a Perpetual bond is equal to 1 + r/ r, where r = current yield of the bond. a) True b) False
57) Zero Coupon bonds are those ________: a) Which do not make a periodical coupon payment. b) These bonds are bought for less than their face value (at a
discount). c) Are mostly issued in Auctions by Treasuriesd d) All of the above
58) Debentures are ________: a) Normal types of bonds issued byCorporates b)it is unsecured debt, backed only by the name and goodwill of the
Company. C) In the event of the liquidation of the corporation, holders of debentures are repaid before stockholders, but after other secured creditors. d) All of the
above
59) If the Coupon rate and the Discount rate (Market based) or the expected rates of return are same:a) The bond will be trading at par
b)Bond will trade at a discount c) Bond will trade at a premium
d) Coupon rate & discount rate have no connection with each other
60) Bill of exchange cannot be______:a) Retained till maturity b) Discounted with bank c) Endorsed to anybody d ) None of these
61) Left side of Asset a/c is for: a) Recording decrease b) Recording increase c) Recording depreciation d) Recording sale
62)One of the following transactions will not result in a higher credit balance in the pass book when compared to cash book? A)interest credited by the
bank
b bank charges debited by the bank c) direct deposit into bank by a customer of the firm d) cheque issued but not presented for payment
63)For the purpose of calculation of depreciation, which of the following is not taken as part of the historical cost
a) purchase price of the asset b)transportation cost and installation cost c) repair and renovation expenses before installation on purchase of 2nd hand
assets
d none of the above
64) Which of the following factors is not taken into account while calculating the amount of depreciation: a) the historical cost b) the scrap value
c)the useful years of life of the asset d) none of the above
65 Which of the following is not a feature of written down value method of depreciation: a)the depreciated value becomes zero at the end of useful
years of
life of the asset b) the amount of depreciation is calculated on the original cost c) for income tax purpose, this method is not recognized d) all the
above
66 A firm purchased goods from another firm but it forgot to record the transaction in the book of original entry. It will be called:a)error of
omission b) error of commission c) error of principle d) compensating error
67 Which among the following error will not be disclosed by the trial balances: a)wrong balancing of account b)posting of an amount on the wrong
side of the account
c) posting of wrong amount d) posting of machinery repair charges to machinery account
68 The sales book has been undercast by Rs.3200. How the error will be rectified: a)the sales book should be credited by Rs.3200 b)the sales book
should be debited by Rs.3200 c) the sales book should be credited by debiting the purchase book d) the sales book should be debited by crediting
the purchase book
69 Identify the revenue expenditure, out of the following transactions: a)import duty paid on import of machinery b) wages paid to labourers,
whose services were used for construction of a factory building c) 2nd hand machinery renovated before installation d) legal expenses on defending a
suit for breach of contract to supply goods
70 Which of the following is a deferred revenue expenditure:a) major repair of old machinery b) heavy marketing expenses by way of
advertisement
c construction of temporary sheds for storage of goods d) all the above
71 Which of the following is taken into account, while calculating the cost of sales: a) opening stock and purchases only b) purchases, closing stock
and direct expenses only c) opening stock, closing stock and purchases only d) opening stocks, purchases, direct expenses and closing stocks
72 Which of the following will not be taken into account, while calculating the cost of sales: a) power and fuel b) carriage outward c) wages to
labour d) octroi
73 Which of the following will be part of the trading account: a) depreciation b)outstanding .rent c)bad debts d)closing stocks
74 If the liabilities are to be arranged in a balance sheet on the basis of liquidity, which of the following order would be appropriate:
a fixed liabilities, current liabilities, long term liabilities b) current liabilities, fixed liabilities, long term liabilities c) current liabilities, long term
liabilities, fixed laibilites
d fixed liabilities, long term liabilities, current liabilities
75 Which of the following cannot be classified as a contingent liability in the books of a firm: a) investment made in partly paid shares b)claims
against the firm not acknowledged as debt by the firm c) guarantee given on behalf of an associate firm d) wages outstanding
76 If the closing stocks do not appear in the trial balance but information is provided as additional information, while preparing the final
account, it will be reported as: a) debit side of trading account b) credit side of profit and loss account c) credit side of trading account d)
debit side of trading account
77 If interest has accrued on certain investments made by a firm but not received so far, while preparing final accounts, it will be shown:
a) on credit side of Profit and loss account and assets side of the balance sheet b) on debit side of Profit and loss account and assets side of the
balance sheet c) on credit side of Profit and loss account and liability side of the balance sheet d) it need not be recorded, as it has not been received
as yet
78 When cash is used to purchase machinery, it will: a) increase the asset and increase the liability b) increase the asset and decrease the
liability
c increase one asset and decrease another asset d) decrease one asset and decrease one liability
79 A firm sold goods costing Rs.20000 for Rs.27500. This would result in:a) increase in the asset and increase in the liability b) increase in the
asset and decrease in the liability c)increase in one asset and decrease in another asset d) decrease in one asset and decrease in one
liability
80: Average capital employed of a firm is Rs.10 lac & average annual profit of Rs.2.05 lac. What is the rate of return on investment: a) 2.05%
b)205% c)20.5% d) 4.85%
81 A bill is drawn by X on Y, who accepts this bill and returns the same to X. Which of the following entries will be passed in the books of
Y:
a) debit X account, debit bills payable account b) debit bills payable account, credit X account c) debit X account, credit bills payable account
d) debit bills receivable account, credit X account
82 Where a partner contributes an amount in excess of his share in the capital, he is entitled to interest (1) at 12% if there is 'no agreement (2) no interest,
if there is no agreement (3) as agreed between the partners (4) at 6% if there is no agreement. A) 1 and 3 b)1 and 2 c)3 and 4 d) 2 and 3
83 In a partnership firm, the partners are maintaining fixed capital. In this case, if the drawings are made, this will be: a)debited to current account of the
partner
b debited to capital account of the partner c) debited to loan account of the partner d) debited to drawings account of the partner
84 Firm Z has Y and Z as partners. Z used Rs.25000 belonging to the firm and made a profit of Rs.3000. How much amouvit he will return to
the firm:
a R s . 2 5 0 0 0 b) Rs.25000 + interest at 6% c) Rs.25000 + Rs.3000 d) Rs.25000 + Rs.3000 — his share of profit out of Rs.3000
85 Firm B has been constituted orally and has A and B as partners. B has been devoting double the time compared to A, for the business of the
firm:
a the profit share will be equal b) A will be having half the profit compared to B c) B will be allowed salary for the extra time being devoted d) If
there is loss, it will be borne by A only and if there is profit it will be shared equally
86 If YTM increase by 20%, the market price of Bond-B will change to: a) 75.80 b)68.90 c)66.20 d)63.80
(Hint: 12 PVIFA (24%,6) + 100 PVIF (24%, 6)
87 X and Y are existing partners and they decide to include Z as new partner with 218th share. The new sharing ratio between X and Y will be 4:2. What
will be new sharing ratio amongst all partners: a)4:2:1 b) 3:2:1 c) 2:2:1 d)2:1:1
88 XYZ Limited came out with a public issue of 20 lac shares of Rs.10 each. The amount received from shareholders will be: a) debited to share
application account credited to share capital account b) debited to bank account and credited to share application account c) debited to bank
account and credited to share capital account d) debited to share application and credited to bank account
89) which of the following is the formula for calculation of EMI: A)P x r {(1+r)" / (1+r )" +1) b)P x r {(1+r)" / (1+r )" -1) c)P x r {(1+r )" X (l+r )" -1} d)
none of the above
90) Which of the following organizations in US, does not influences the development of GAAP: a) US Securities Exchange Commission b) American
Institute of Certified Public Accountants c) Financial Accounting Standard Board d) Federal Reserve of US (US Central Bank)
91) A company had a public issue of 20000 shares of Rs.10 each. X did not pay the first and final call amount of Rs.3 per share for 300 shares. In this
Compiled by Mr. Sanjay Kumar Trivedy, Sr. Mgr., RSTC, mumbai
35
connection, which of the following will be correct: a) share capital account will be debited for Rs.3000 b)share allotment and share application will
be credited for Rs.7000
c) forfeiture account shall be credited for Rs.900 d) calls in arrear account shall be credited for Rs.2100
92) A company decides to reward its shareholders by paying dividend. Out of which the following, which account can be debited: a) share
premium account
b) general reserve account c)profit on re-issue of forfeited shares account d) none of these can be debited
93 A company issued 300 shares to X having face value of Rs.10. The called up amount is Rs.5 per share while X has paid Rs.3 per share so far. After
forfeiture, the company decides to re-issue these shares to Y as Rs.5 paid up for Rs.4 per share. What is the amount that would be credited to the
capital reserve account:
a Rs.1500 b)Rs.900 c)Rs.600 d) Incomplete information.
94 Pre-operative expenses are shown by a company as: a) expenditure in the trading account b) expenditure in the profit loss account c) asset in the
balance sheet
d liability in the balance sheet

95 The finance manager of a company receives 10% commission on the profit after charging commission. The profit of the company are Rs.3.30 lac. What
will be amount of commission of the manager: a)Rs.33000 b)Rs.30000 c) Rs.27767 d )Rs.25987
96 Which of the following accounting concept is followed at the reporting stage instead of, at the recording stage: a)matching concept
b)historic concept
c)business entity concept d) going concern concept
97 In. India, the accounting standards are issued by ------- working u n d e r : a ) Accounting Standards Board, Institute of Company Secretaries of
India.
b) Accounting Standards Institute, Institute of Chartered Accountants of India. c)Accounting Standards Board, Institute of Chartered Accountants of India.
d) Accounting Standards Board, Govt. of India, Ministry of Company Affairs.
98 X, Y and Z are three partners with a sharing ratio of 7:5:4. X retires from the firm. The new sharing ratio between Y and Z shall be: a)5:4 b)4:5
c)7:5
d)incomplete information
99 The balance in the pass book is Rs.54000 and it is observed that bank received Rs.2000 through NEFT and also remitted Rs.13000 through NEFT, as
per standing instruction. What is the balance as per cash book: a ) 5 4 0 0 0 b ) 6 5 0 0 0 c ) 4 3 0 0 0 d ) 3 9 0 0 0
100 The balance in the cash book is Rs.30000 overdraft. It is observed that a cheque of Rs.2000 has been debited twice by the bank. Further the bank
credited Rs.3500 to the account of the firm by mistake, while this amount was to be credited to personal account of the partner. What is the balance in
the pass book:
A 35500 credit balance b)28500 debit balance c)31500 credit balance d) 28500 credit balance

ANSWERS - 2
01 b 02 a 03 a 04 b 05 a 06 c
07 b 08 d 09 c 10. a 11 c 12 a
13 a 14 d 15 b 16 c 17 b 18 d
19 b 20 a 21 b 22 b 23 c 24 b
25 d 26 b 27 b 28 d 29 b 30 a
31 d 32 a 33 b 34 c 35 c 36 c
37 c 38 c 39 a 40 a 41 c 42 d
43 b 44 c 45 b 46 c 47 d 48 c
49 d 50 d 51 b 52 b 53 a 54a
55 d 56 a 57 d 58 d 59 a 60 d
61 b 62 b 63 d 64 d 65 d 66
67 d 68 a 69 d 70 b 71 d 72 b
73 d 74 c 75 d 76 c 77 a 78 c
79 a 80 c 81 c 82 c 83 a 84 c
85 a 86 d 87 d 88 b 89 b 90 d
91 a 92 b 93 c 94 c 95 b 96 a
97 c 98 a 99 c 100 b

Compiled by Mr. Sanjay Kumar Trivedy, Sr. Mgr., RSTC, mumbai


36

You might also like